Sunteți pe pagina 1din 243

fiziks

Institute for NET/JRF, GATE, IIT‐JAM, M.Sc. Entrance, JEST, TIFR and GRE in Physics 
 
NET JUNE 2019
PART- A
Q1. In a bacterial cell, a protein is synthesized at random location in the cytoplasm. The
protein has to reach one pole of the cell for its appropriate function. The protein reaches
the pole by
(a) chemical attraction (b) random movement
(c) enzymatic action (d) attraction between opposite charges
Q2. A precious stone breaks into four pieces having weights in the proportion 1: 2 : 3 : 4 . The
value of such a stone is proportional to the square of its weight. What is the percent loss
in the value incurred due to breaking?
(a) 0 (b) 30 (c) 70 (d) 90
Q3. Two runners starting together run on a circular path taking 6 and 8 minutes, respectively,
to complete one round. How many minutes later do they meet again for the first time on
the start line, assuming constant speeds
(a) 8 (b) 24 (c) 32 (d) 60
Q4. The distribution of grades secured by students in a class is given in the table below.
Grade Fraction of the
Population
A 0.1
B 0.4
C 0.3
D 0.2
What is the least possible population of the class?
Q5. The nine numbers x1 , x2 , x3 ...x9 , are in ascending order. Their average m is strictly greater

than all the first eight numbers. Which of the following is true?
(a) Average  x1 , x2 ...x9 , m   m and Average  x2 , x3 ,...x9   m

(b) Average  x1 , x2 ...x9 , m   m and Average  x2 , x3 ,...x9   m

(c) Average  x1 , x2 ...x9 , m   m and Average  x2 , x3 ,...x9   m

(d) Average  x1 , x2 ...x9 , m   m and Average  x2 , x3 ,...x9   m

H.No. 40-D, Ground Floor, Jia Sarai, Near IIT, Hauz Khas, New Delhi-110016
Phone: 011-26865455/+91-9871145498
Website: www.physicsbyfiziks.com | Email: fiziks.physics@gmail.com
1
fiziks
Institute for NET/JRF, GATE, IIT‐JAM, M.Sc. Entrance, JEST, TIFR and GRE in Physics 
 
Q6. Which among the following diagrams represents women, mothers, human beings?
(a) (b)

(c) (d)

Q7. A boy and a girl make the following statements, of which at most one is correct:
The one in a white shirt says: “I am a girl” (statement – I)
The one in a blue shirt says: “I am a boy” (statement – II)
Which of the following is the correct inference?
(a) Statement – I is correct but statement – II is incorrect
(b) Statement – II is correct but statement – I is incorrect
(c) Both statement I and II are incorrect
(d) The correctness of the statements I and II cannot be ascertained
Q8. How many quadrilaterals does the following figure have?

(a) 17 (b) 18 (c) 19 (d) 20


Q9. 12 balls, 3 each of the colours red, green, blue and yellow are put in a box and mixed. If
3 balls are picked at random, without replacement, the probability that all 3 balls are of
the same colour is
1 1 1 1
(a) (b) (c) (d)
4 12 36 55

H.No. 40-D, Ground Floor, Jia Sarai, Near IIT, Hauz Khas, New Delhi-110016
Phone: 011-26865455/+91-9871145498
Website: www.physicsbyfiziks.com | Email: fiziks.physics@gmail.com
2
fiziks
Institute for NET/JRF, GATE, IIT‐JAM, M.Sc. Entrance, JEST, TIFR and GRE in Physics 
 
Q10. Some aliens observe that roosters call before sunrise every day. Having no other
information about roosters and sunrises, which of the following inferences would NOT
be valid?
(a) Rooster-call and sunrise may be independent cyclic events with the same periodicity
(b) Both may be triggered by a common cause
(c) Rooster-call may be causing the sunrise
(d) Sunrise cannot be the cause of rooster call as the rooster-call precedes sunrise
Q11. Twenty-one litres of water in a tank is to be divided into three equal parts using only 5,8
and 12 litre capacity cans. The minimum number of transfers needed to achieve this is
(a) 3 (b) 4 (c) 5 (d) 7
Q12. Of four agents Alpha, Beta, Gamma and Delta, three have to be sent together on a
mission. If Alpha and Beta cannot go together, Beta and Gamma cannot go together and
Gamma and Delta cannot go together, then which of the following holds?
(a) Any three agents can be sent.
(b) Alpha, Delta and any one out of Beta and Gamma can be sent
(c) Beta, Gamma and any one out of Alpha and Delta can be sent
(d) The mission is impossible.
Q13. An open rectangular box is made by excluding the four identical corners of a piece of
paper as shown in the diagram and folding it along the dotted lines
The capacity of the box (in cm3 ) is 10 cm
10 cm
(a) 8000

(b) 1000 40 cm
(c) 4000

(d) 6000
40 cm
Q14. Which of the following is the largest?
250 ,340 , 430 ,520

(a) 250 (b) 340 (c) 430 (d) 520

H.No. 40-D, Ground Floor, Jia Sarai, Near IIT, Hauz Khas, New Delhi-110016
Phone: 011-26865455/+91-9871145498
Website: www.physicsbyfiziks.com | Email: fiziks.physics@gmail.com
3
fiziks
Institute for NET/JRF, GATE, IIT‐JAM, M.Sc. Entrance, JEST, TIFR and GRE in Physics 
 
Q15. A monkey climbs a tree to eat fruits. The amount of energy gained from eating fruits and
the energy spent in climbing on different branches have a relationship shown in the figure.

Energy gained
0, 0 Energy spent

The ratio of energy gained to energy spent will be the maximum


(a) at a point where the slope of the curve is the maximum
(b) at a point where the slope of the curve is unity
(c) at a point on the curve where the tangent passes through the origin
(d) at the highest point on the curve
Q16. The length of a cylinder is measured 10 times yielding 10 distinct values. For this set of
values, consider the following statements
A. Five of these values will lie above the mean and five below it
B. Five of these values will lie above median and five below it
C. At least one value will lie above the mean
D. At least one value will lie at the median
Which of the statements are necessarily correct?
(a) B and C (b) A and C (c) B and D (d) A,C and D
Q17. In the given circle, O is the centre,
P Q
PAO  40o , PBQ  30o and outer angle AOB  220o .
Then AQB is
0
300
0
40 220
o o O
(a) 70 (b) 80 B
A
(c) 60o (d) 110o

H.No. 40-D, Ground Floor, Jia Sarai, Near IIT, Hauz Khas, New Delhi-110016
Phone: 011-26865455/+91-9871145498
Website: www.physicsbyfiziks.com | Email: fiziks.physics@gmail.com
4
fiziks
Institute for NET/JRF, GATE, IIT‐JAM, M.Sc. Entrance, JEST, TIFR and GRE in Physics 
 
Q18. A canal system is shown in the figure
G1

A B

G2
Water flows from A to B through two channels. Gates G1 and G2 , are operated

independently to regulate the flow. Probability of G1 to be open is 10% while that of G2

is 20% . The probability that water will flow from A to B is


(a) 10% (b) 20% (c) 28% (d) 30%
Q19. A long ream of paper of thickness t is rolled tightly. As the roll becomes larger, the
length of the paper wrapped in one turn exceeds the length in the previous turn by
(a) t (b) 2t (c)  t (d) 2 t
Q20. Point A on a wheel of radius r touches the horizontal plane at point P . It rolls without
slipping, till point A is at the highest position in the first turn. What is the final distance
AP ? A

A
P
initial final

(a) 2r (b) r 1   
2
(c) r 4   
2
(d) 2r 1   
2

H.No. 40-D, Ground Floor, Jia Sarai, Near IIT, Hauz Khas, New Delhi-110016
Phone: 011-26865455/+91-9871145498
Website: www.physicsbyfiziks.com | Email: fiziks.physics@gmail.com
5
fiziks
Institute for NET/JRF, GATE, IIT‐JAM, M.Sc. Entrance, JEST, TIFR and GRE in Physics 
 
PART- B
Q21. An object is dropped on a cushion from a height 10 m above it. On being hit, the cushion
is depressed by 0.1 m . Assuming that the cushion provides a constant resistive force, the
deceleration of the object after hitting the cushion, in terms of the acceleration due to
gravity g is
(a) 10 g (b) 50 g (c) 100 g (d) g
Q22. A turn-table is rotating with a constant angular velocity 0 . In the rotating frame fixed to

the turntable, a particle moves radially outwards at a constant speed v0 . The acceleration

of the particle in the r coordinates, as seen from an inertial frame, the origin of which is
at the centre of the turntable, is
(a) r02 rˆ (b) 2r02 rˆ  v00ˆ

(c) r02 rˆ  2v00ˆ (d)  r02 rˆ  2v00ˆ


Q23. Assume that the earth revolves in a circular orbit around the sun. Suppose the
gravitational constant G varies slowly as a function of time. In particular, it decreases to
half its initial value in the course of one million years. Then during this time the
(a) radius of the earth’s orbit will increase by a factor of two
(b) total energy of the earth remains constant
(c) orbital angular momentum of the earth will increase
(d) radius of the earth’s orbit remains the same.
Q24. A particle of mass m moves in One dimension in the potential V  x   kx 4 ,  k  0  . at

time t  0 the particle starts from rest at x  A .


For bounded motion, the time period of its motion is
(a) proportional to A1/ 2
(b) proportional to A1
(c) independent of A
(d) not well-defined (the system is chaotic)

H.No. 40-D, Ground Floor, Jia Sarai, Near IIT, Hauz Khas, New Delhi-110016
Phone: 011-26865455/+91-9871145498
Website: www.physicsbyfiziks.com | Email: fiziks.physics@gmail.com
6
fiziks
Institute for NET/JRF, GATE, IIT‐JAM, M.Sc. Entrance, JEST, TIFR and GRE in Physics 
 
Q25. The infinite square-well potential of a particle in a box of size a is modified as shown in
the figure below (assume   a )
 

a




The energy of the ground state, compared to the ground state energy before the
perturbation was added
(a) increases by a team of order  (b) decreases by a term of order 
(c) increases by a term of order  2 (d) decreases by a term of order  2
Q26. A quantum particle of mass m in one dimension, confined to a rigid box as shown in the
figure, is in its ground state. An infinitesimally thin wall is very slowly raised to infinity
at the centre of the box, in such a way that the system remains in its ground state at all
times. Assuming that no energy is lost in raising the wall, the work done on the system
when the wall is fully raised, eventually separating the original box into two
compartments, is     

L O L L O L

3 2  2  22  22
(a) (b) (c) (d) 0
8mL2 8mL2 2mL2
Q27. The wavefunction of a free particle of mass m , constrained to move in the interval
 L  x  L , is   x   A  L  x  L  x  , where A is the normalization constant. The

 22
probability that the particle will be found to have the energy is
2mL2
1 1 1
(a) 0 (b) (c) (d)
2 2 3 

H.No. 40-D, Ground Floor, Jia Sarai, Near IIT, Hauz Khas, New Delhi-110016
Phone: 011-26865455/+91-9871145498
Website: www.physicsbyfiziks.com | Email: fiziks.physics@gmail.com
7
fiziks
Institute for NET/JRF, GATE, IIT‐JAM, M.Sc. Entrance, JEST, TIFR and GRE in Physics 
 
Q28. A particle moving in a central potential is described by a wavefunction   r   zf  r 

where r   x, y, z  is the position vector of the particle and f  r  is a function of r  r .

If L is the total angular momentum of the particle, the value of L2 must be


3 2
(a) 2 2 (b)  2 (c) 4 2 (d) 
4
Q29. A particle of mass in and energy E  0 . in one dimension is scattered by the potential
below. V  x

a b x
V1

V2

If the particle was moving from x   to x   , which of the following graphs gives
the best qualitative representation of the wavefunction of this particle?

  x   x
(a) (b)

x x
a b a b

(c)   x  (d)   x 

x x
a b a b

H.No. 40-D, Ground Floor, Jia Sarai, Near IIT, Hauz Khas, New Delhi-110016
Phone: 011-26865455/+91-9871145498
Website: www.physicsbyfiziks.com | Email: fiziks.physics@gmail.com
8
fiziks
Institute for NET/JRF, GATE, IIT‐JAM, M.Sc. Entrance, JEST, TIFR and GRE in Physics 
 
Q30. Consider a planar wire loop as an n -sided regular polygon, in which R is the distance
from the centre to a vertex. If a steady current I flows through the wire, the magnitude of
the magnetic field at the centre of the Loop is
0 I  2  0 nI  
(a) sin   (b) sin  
2R  n  4 R n
0 nI  2  0 nI  
(c) tan   (d) tan  
2 R  n  2 R n
Q31. Two coherent plane electromagnetic waves of wavelength 0.5  m (both have the same
amplitude and are linearly polarized along the z -direction) fall on the y  0 plane. Their
wave vectors k1 and k1 are as shown in the figure
y
 
k1 k 2

 

If the angle  is 30o , the fringe spacing of the interference pattern produced on the plane
is
(a) 1.0 m (b) 0.29  m (c) 0.58  m (d) 0.5  m
Q32. Which of the following is not a correct boundary condition at an interface between two
homogeneous dielectric media? (In the following n̂ is a unit vector normal to the

interface,  and js , are the surface charge and current densities, respectively.)
    
 
(a) nˆ  D1  D2  0 
(b) nˆ  H1  H 2  js 
   
 
(c) n̂  D1  D2    
(d) nˆ  B1  B2  0

H.No. 40-D, Ground Floor, Jia Sarai, Near IIT, Hauz Khas, New Delhi-110016
Phone: 011-26865455/+91-9871145498
Website: www.physicsbyfiziks.com | Email: fiziks.physics@gmail.com
9
fiziks
Institute for NET/JRF, GATE, IIT‐JAM, M.Sc. Entrance, JEST, TIFR and GRE in Physics 
 
Q33. The permittivity tensor of a uniaxial anisotropic medium, in the standard Cartesian basis,
 4 0 0 0 
 
is  0 4 0 0  where  0 is a constant. The wave number of an electromagnetic
 0 9 0 
 0

plane wave polarized along the x -direction, and propagating along the y -direction in
this medium (in terms of the wave number k0 of the wave in vacuum) is

(a) 4k0 (b) 2k0 (c) 9k0 (d) 3k0

Q34. The element of a 3  3 matrix A are the products if its row and column indices Aij  ij

(where i, j  1, 2,3 ). The eigenvalues of A are

(a)  7, 7, 0  (b)  7, 4,3

 14 14 14 
(c) 14, 0, 0  (d)  , , 
 3 3 3
Q35. In the following circuit, each device D may be an insulator with probability p or a

conductor with probability 1  p  .


D
D
D D

The probability that a non-zero current flows through the circuit is

(b) 1  p 
4
(a) 2  p  p 3

(c) 1  p  p 2
2

(d) 1  p  1  p 3 
dy
Q36. The solution of the differential equation x  1  x  y  e  x with the boundary
dx
condition y  x  1  0 , is

(a)
 x  1 e x (b)
 x  1 e x
x x2

(c)
1  x  e x (d)  x  1 e  x
2
2
x

H.No. 40-D, Ground Floor, Jia Sarai, Near IIT, Hauz Khas, New Delhi-110016
Phone: 011-26865455/+91-9871145498
Website: www.physicsbyfiziks.com | Email: fiziks.physics@gmail.com
10
fiziks
Institute for NET/JRF, GATE, IIT‐JAM, M.Sc. Entrance, JEST, TIFR and GRE in Physics 
 
 d
Q37. The value of the definite integral 
0 5  4 cos 
is

4 2 
(a) (b) (c)  (d)
3 3 3
Q38. In a system comprising of approximately 1023 distinguishable particles, each particle may
occupy any of 20 distinct states. The maximum value of the entropy per particle is
nearest to
(a) 20k B (b) 3k B

(c) 10  ln 2  k B (d) 20  ln 2  k B

Q39. Consider a classical gas in thermal equilibrium at temperatures T1 and T2 where T1  T2 .


Which of the following graphs correctly represents the qualitative behaviour of the
probability density function of the x -component of the velocity?

T1 T1 T2
(a) (b)
P(Vx)

P(Vx)

T2

0 0
Vx Vx

(c) (d)
P(Vx)

P(Vx)

T1 T2 T1 T2
0 0
Vx Vx
Q40. The equation of state of an ideal gas is pV  RT . At very low temperatures, the volume
1 V
expansion coefficient at constant pressure
V T
1
(a) diverges as 2
T
1
(b) diverges as
T
(c) vanishes as T
(d) is independent of the temperature

H.No. 40-D, Ground Floor, Jia Sarai, Near IIT, Hauz Khas, New Delhi-110016
Phone: 011-26865455/+91-9871145498
Website: www.physicsbyfiziks.com | Email: fiziks.physics@gmail.com
11
fiziks
Institute for NET/JRF, GATE, IIT‐JAM, M.Sc. Entrance, JEST, TIFR and GRE in Physics 
 
1 2
Q41. The Hamiltonian of a classical nonlinear one dimensional oscillator is H  p   x4 ,
2m
where   0 is a constant. The specific heat of a collection of a collection of N
independent such oscillators is
3Nk B 3Nk B
(a) (b)
2 4
Nk B
(c) Nk B (d)
2
Q42. In an experiment to measure the acceleration due to gravity g using a simple pendulum,
the length and time period of the pendulum are measured to three significant figures. The
mean value of g and the uncertainty  g of the measurements are then estimated using a

calculator from a large number of measurements and found to be 9.82147 m / s 2 and

0.02357m / s 2 , respectively. Which of the following is the most accurate way of


presenting the experimentally determined value of g ?

(a) 9.82  0.02 m / s 2 (b) 9.8215  0.02 m / s 2

(c) 9.82147  0.02357 m / s 2 (d) 9.82  0.02357 m / s 2


Q43. An ac signal of the type as shown in the figure, is applied across a resistor R  1 .
8
V(Volts)

3
0
2 t

The power dissipated across the resistor is


(a) 12.5W (b) 9W (c) 25 W (d) 21.5W

H.No. 40-D, Ground Floor, Jia Sarai, Near IIT, Hauz Khas, New Delhi-110016
Phone: 011-26865455/+91-9871145498
Website: www.physicsbyfiziks.com | Email: fiziks.physics@gmail.com
12
fiziks
Institute for NET/JRF, GATE, IIT‐JAM, M.Sc. Entrance, JEST, TIFR and GRE in Physics 
 
Q44. An npn -transistor is connected in a voltage divider configuration as shown in the figure
below. 20V

RC  5 k 
R1  80 k 
C
  50
B
R2  20 k 
E
RE 1k 

If the resistor R2 is disconnected, the voltages VB at the base and VC at the collector
change as follows.
(a) both VB and VC increase (b) both VB and VC decrease

(c) VB decreases, but VC increases (d) VB increases, but VC decreases


Q45. Let Y denote the output in the following logical Circuit.
A
G1
B

G2 Y
C
D

If Y  AB  C D , the gates G1 and G2 must, respectively, be


(a) OR and NAND (b) NOR and OR
(c) AND and NAND (d) NAND and OR

H.No. 40-D, Ground Floor, Jia Sarai, Near IIT, Hauz Khas, New Delhi-110016
Phone: 011-26865455/+91-9871145498
Website: www.physicsbyfiziks.com | Email: fiziks.physics@gmail.com
13
fiziks
Institute for NET/JRF, GATE, IIT‐JAM, M.Sc. Entrance, JEST, TIFR and GRE in Physics 
 
PART - C
Q46. A solid spherical Cork of radius R and specific gravity 0.5 floats on water. The cork is
pushed down so that its centre of mass is at a distance h (where 0  h  R ) below the
surface of water, and Then released. The volume of the part of the cork above water level
2 1 
is  R 3   cos  0  cos3  0  where  0 is the angle as shown in the figure.
3 3 

Water level
h 0
R

At the moment of release, the dependence of the upward force on the cork on h is
3 3 3 3
h 1 h  h 1 h  h 2 h  h 2 h 
(a)    (b)    (c)    (d)   
R 3 R  R 3 R  R 3 R R 3 R
Q47. Two particles of masses m1 and m2 are connected by a massless thread of length l as
shown in figure below.
m1
r0

g m2
The particle of mass in on the plane undergoes a circular motion with radius r0 and

angular momentum L . When a small radial displacement  (whew  r0 ) is applied,

its radial coordinate is found to found to oscillate about r0 . The frequency of the
oscillations is
7 m2 g 7m2 g
(a) (b)
 m2   m1  m2  r0
 m1   r0
 2 

3m2 g 3m2 g
(c) (d)
 m2   m1  m2  r0
 m1   r0
 2 

H.No. 40-D, Ground Floor, Jia Sarai, Near IIT, Hauz Khas, New Delhi-110016
Phone: 011-26865455/+91-9871145498
Website: www.physicsbyfiziks.com | Email: fiziks.physics@gmail.com
14
fiziks
Institute for NET/JRF, GATE, IIT‐JAM, M.Sc. Entrance, JEST, TIFR and GRE in Physics 
 
Q48. The time evolution of a coordinate x of a particle is described by the equation
d 2x 2

dt 4
 2 2 d x

dt 2 
  4  A4 x  0 
For   A , the particle will
(a) eventually come to rest at the origin
(b) eventually drift to infinity  x   

(c) oscillate about the origin


 
(d) eventually come to rest at or 
A A
p2
The Hamiltonian of a quantum particle of mass m is H    x , where  and r
r
Q49.
2m
are positive constants. The energy En of the nth level for large n , depends on n as

(a) n 2 r (b) n r  2 (c) n1/  r  2 (d) n 2 r /  r  2


Q50. In the partial wave expansion, the differential scattering cross-section is given by
2
d
d  cos  
   2l  1 e
l
i i
sin  l Pl  cos  

where  is the scattering angle. For a certain neutron-nucleus scattering. it is found that
the two lowest phase shifts  0 and 1 corresponding to s -wave and p -wave,

0
respectively, satisfy 1  . Assuming that the other phase shifts are negligibly small,
2
the differential cross-section reaches its minimum for cos  equal to
2 1
(a) 0 (b) 1 (c)  cos 2 1 (d) cos 2 1
3 3
Q51. A charged, spin-less particle of mass m is subjected to an attractive potential

V  x, y , z  
1
2
 
k x 2  y 2  z 2 , where k is a positive constant. Now a perturbation in the

form of a weak magnetic field B  B0 kˆ (where B0 is a constant is switched on. Into how
many distinct levels will the second excited state of the unperturbed Hamiltonian split?
(a) 5 (b) 4 (c) 2 (d) 1

H.No. 40-D, Ground Floor, Jia Sarai, Near IIT, Hauz Khas, New Delhi-110016
Phone: 011-26865455/+91-9871145498
Website: www.physicsbyfiziks.com | Email: fiziks.physics@gmail.com
15
fiziks
Institute for NET/JRF, GATE, IIT‐JAM, M.Sc. Entrance, JEST, TIFR and GRE in Physics 
 
Q52. The elastic scattering of a charged particle of mass m off an atom can be approximated

by the potential V  r   e  r / R where  and R
r
are positive constants. If the wave number of the incoming particle is k and the
scattering angle is 2 , the differential cross-section in the Born approximation is
m 2 2 R 4 m 2 2 R 4
(a) (b)

4 4 1  k 3 R 2 sin 2   
 4 2k 2 R 2 sin 2  
2

2m 2 2 R 4 4m 2 2 R 4
(c) 4 (d)

 2k 2 R 2 2sin 2   
 4 1  4k 2 R 2 sin 2  
2

Q53. The wave number k and the angular frequency  of a wave are related by the dispersion
relation  2   k   k 3 where  and  are positive constants. The wave number for
which the phase velocity equals the group velocity, is
  1  1 
(a) 3 (b) (c) (d)
  2  3 

Q54. A inertial observer A at rest measures the electric and magnetic field E   , 0, 0  and

B   , 0, 2  in a region, where a is a constant. Another inertial observer B , moving

with a constant velocity with respect to A , measures the fields as E    Ex ,  , 0 

and B   , By ,   . Then in units c  1, Ex and By are given, respectively, by

(a) 2 and  (b) 2 and 


(c)  and 2 (d)  and 2

H.No. 40-D, Ground Floor, Jia Sarai, Near IIT, Hauz Khas, New Delhi-110016
Phone: 011-26865455/+91-9871145498
Website: www.physicsbyfiziks.com | Email: fiziks.physics@gmail.com
16
fiziks
Institute for NET/JRF, GATE, IIT‐JAM, M.Sc. Entrance, JEST, TIFR and GRE in Physics 
 
Q55. A point charge is moving with a uniform velocity  C along the positive x -direction,
parallel to and very close to a corrugated metal sheet (see the figure below).
c metal
sheet

L L

The wavelength of the electromagnetic radiation received by an observer along the


direction of motion is
1
(a) 1  2 (b) L 1   2

(c) L 1   2 (d) L
Q56. If the Newton-Raphson method is used to find the positive root of the equation
x  2sin x , the iteration equation is
2 xn  2  sin xn  xn cos xn  2  sin xn  xn cos xn 
(a) xn 1  (b) xn 1 
1  2 cos xn 1  2 cos xn

xn2  1  2  cos xn  xn sin xn  xn2  1  2  cos xn  sin xn 


(c) xn 1  (d) xn 1 
xn  2sin xn xn  2sin xn

Q57. x   2 x  A cos t ,
The equation of motion of a forced simple harmonic oscillator is 
where A is a constant. At resonance    the amplitude of oscillations at large times
(a) saturates to a finite value (b) increases with time as t
(c) increases linearly with time (d) increases exponentially with time
2 1 1
Q58. The operator A has a matrix representation   in the basis spanned by   and
1 2 0
0 1 1 1 1
  . In another basis spanned by   and   , the matrix representation of A
1 2 1 2  1
is
2 0  3 0  3 1 3 0
(a)   (b)   (c)   (d)  
 0 2 0 1  0 1 1 1

H.No. 40-D, Ground Floor, Jia Sarai, Near IIT, Hauz Khas, New Delhi-110016
Phone: 011-26865455/+91-9871145498
Website: www.physicsbyfiziks.com | Email: fiziks.physics@gmail.com
17
fiziks
Institute for NET/JRF, GATE, IIT‐JAM, M.Sc. Entrance, JEST, TIFR and GRE in Physics 
 
d
Q59. The operator x   x  , where   x  is the Dirac delta function, acts on the space of real
dx
-valued square-integrable functions on the real line. This operator is equivalent to
(a)   x  (b)   x  (c) x (d) 0

Q60. At each time step, a random walker in one dimension either remains at the same point
1 3
with probability , or moves by a distance  to the right or left with probabilities
4 8
each. After N time steps, its root mean squared displacement is

9N 3N 3N
(a)  N (b)  (c)  (d) 
16 4 8
Q61. The Hamiltonian of three Ising spins S1 , S 2 and S3 , each taking values 1 , is

H   J  S1S 2  S 2 S3   hS1 , where J and h are positive constants. The mean value of

S3 in equilibrium at a temperature T  1/(k B) , is

(a) tanh 3   J  (b) tan   h  tanh 2   J 

(c) sinh   h  sinh 2   J  (d) 0

Q62. The free energy of a magnetic system, as a function of its magnetisation m , is


1 2 1 4 1 6
F am  bm  m . where a and b are positive constants .
2 4 6
At a fixed value of a , the critical value of b ,above which the minimum of F will be at a
non-zero value of magnetisation, is

10a 16a 10 16
(a) (b) (c) a (d) a
3 3 3 3
Q63. For optimal performance of an op-amp based current-to-voltage converter circuit, the
input and output impedance should be
(a) Low input impedance and high output impedance
(b) low input impedance and low output impedance
(c) high input impedance and high output impedance
(d) high input impedance and low output impedance

H.No. 40-D, Ground Floor, Jia Sarai, Near IIT, Hauz Khas, New Delhi-110016
Phone: 011-26865455/+91-9871145498
Website: www.physicsbyfiziks.com | Email: fiziks.physics@gmail.com
18
fiziks
Institute for NET/JRF, GATE, IIT‐JAM, M.Sc. Entrance, JEST, TIFR and GRE in Physics 
 
The forward diode current is given by I  kT  e g B  exp  eV / k BT   1 , where Eg is
E / k T
Q64.

the band gap of the semiconductor, V is the voltage drop across the diode, T is the
temperature of the diode operating near room temperature and,  and K are constants.
A diode is used as a thermal sensor in the circuit shown below.
V
I

If V is measured using an ideal voltmeter to estimate T , the variation of the voltage V


as a function of T is best approximated by (in the following a and b are constants)
(a) aT 2  b (b) aT  b (c) aT 3  b (d) aT  bT 2
Q65. A circuit constructed using op-amp, resistor R1  1 k  and capacitors C1  1  F and

C2  0.1  F is shown in the figure below. R1


This circuit will act as a
C2
(a) high pass filter C1
Vi 
(b) low pass filter  Vo
(c) band pass filter
(d) band reject filter
Q66. The third-nearest neighbour distance in a BCC (Body Centered Cubic) crystal with lattice
constant a0 is

3a0
(a) a0 (b) (c) 3a0 (d) 2a0
2
Q67. A bound electron and hole pair interacting via Coulomb interaction in a semiconductor is
called an exciton. The effective masses of an electron and a hole are about 0.1 me and

0.5me respectively, where me is the rest mass of the electron. The dielectric constant of

the semiconductor is 10 . Assuming that the energy levels of the excitons are hydrogen-
like, the binding energy of an exciton (in units of the Rydberg constant) is closest to
(a) 2 103 (b) 2 104 (c) 8 104 (d) 3 103

H.No. 40-D, Ground Floor, Jia Sarai, Near IIT, Hauz Khas, New Delhi-110016
Phone: 011-26865455/+91-9871145498
Website: www.physicsbyfiziks.com | Email: fiziks.physics@gmail.com
19
fiziks
Institute for NET/JRF, GATE, IIT‐JAM, M.Sc. Entrance, JEST, TIFR and GRE in Physics 
 
Q68. Consider an array of atoms in one dimension with an ensemble averaged periodic density
distribution as shown in the figure.
  x

1/

a 2a 3a 4a x

If k is the wave number and S  k ,   denotes the Fourier transform of the density-

S k, 
density correlation function, the ratio is
S  k , 0

 k   k 
(a) cos   (b) cos 2  
 2   2 
2  k  4  k 
(c) sin   (d) sin 2  
k  2  k  2
2
 2 
Q69. A doubly charged ion in the angular momentum state  J  2, J 3  1 meets a gas of

 1
polarized electrons  S3   and gets neutralized. If the orbital angular momentum
 2
transferred in the process is zero, the probability that the neutral atom is in the
 J  2, J 3  2  state is

2 2 1 1
(a) (b) (c) (d)
5 3 5 3
o
Q70. The range of the inter-atomic potential in gaseous hydrogen is approximately 5 A . In
thermal equilibrium, the maximum temperature for which the atom-atom scattering is
dominantly s -wave, is
(a) 500 K (b) 100 K (c) 1 K (d) 1mK

H.No. 40-D, Ground Floor, Jia Sarai, Near IIT, Hauz Khas, New Delhi-110016
Phone: 011-26865455/+91-9871145498
Website: www.physicsbyfiziks.com | Email: fiziks.physics@gmail.com
20
fiziks
Institute for NET/JRF, GATE, IIT‐JAM, M.Sc. Entrance, JEST, TIFR and GRE in Physics 
 
Q71. The energy levels corresponding to the rotational motion of a molecule are
EJ  BJ  J  1 cm 1 where J  0,1, 2,... , and B is a constant. Pure rotational Raman

transitions follow the selection rule J  0, 2 . When the molecule is irradiated, the

separation between the closest Stokes and anti-Stokes lines (in cm 1 ) is


(a) 6B (b) 12 B (c) 4 B (d) 8B
Q72. The cavity of a He-Ne laser emitting at 632.8 nm , consists of two mirrors separated by a
distance of 35 cm . If the oscillations in the laser cavity occur at frequencies within the
gain bandwidth of 1.3GHz , the number of longitudinal modes allowed in the cavity is
(a) 1 (b) 2 (c) 3 (d) 4
Q73. An excited state of a 84 Be nucleus decays into two  -particles which are in a spin-

parity 0 state. If the mean life-time of this decay is 1022 s , the spin-parity of the excited
state of the nucleus is
(a) 2 (b) 3 (c) 0 (d) 4
Q74. The elastic scattering of a neutrino  e by an electron e  , i.e. the reaction

 e  e   e  e can be described by the interaction Hamiltonian

GF d 3 x  e  x     e  x     e  x    e  x  
1
H int 
2 
The cross-section of the above process depends on the centre of mass energy E , as
1
(a) (b) E 2 (c) E (d) E
E2
Q75. The mean life-time of the following decays:
0       ,  0     ,    e  e  v , are   ,   and   respectively.
They satisfy
(a)         (b)        

(c)         (d)        

H.No. 40-D, Ground Floor, Jia Sarai, Near IIT, Hauz Khas, New Delhi-110016
Phone: 011-26865455/+91-9871145498
Website: www.physicsbyfiziks.com | Email: fiziks.physics@gmail.com
21
fiziks
Institute for NET/JRF, GATE, IIT‐JAM, M.Sc. Entrance, JEST, TIFR and GRE in Physics 
 
NET DECEMBER 2018
BOOKLET CODE ‘A’

INSTRUCTIONS
1. You have opted for English as medium of question. This Test Booklet contains seventy five (20 Part ‘A’
+25 Part ‘B’ +30 Part ‘C’). Multiple Choice Questions (MCQs).
You are required to answer a maximum of 15, 20 and 20 questions from part ‘A’ ‘B’ and ‘C’

respectively. If more than required number of questions are answered, only first 15, 20, 20 questions in
Part ‘A’ ‘B’ and ‘C’ respectively, will be taken up for evaluation.
2. OMR answer sheet has been provided separately. Before you start filling up your particulars, please
ensure that the booklet contains requisite number of pages and that these are not torn or mutilated. If it is
so, you may request the invigilator to change the booklet of the same code. Likewise check the OMR
answer sheet also, Sheets for rough work have been appended to the test booklet.
3. Write your Roll No, Name and Serial Number of this Test Booklet on the OMR Answer sheet in the
space provided. Also put your signatures in the space earn marked.
4. You must darken the appropriate circles with a black ball pen related to Roll Number, Subject
Code, Booklet Code and Centre Code on the OMR answer sheet. It is the sole responsibility of the
candidate to meticulously follow the instructions given on the answer sheet, failing which, the
computer shall not be able to decipher the correct detail which may ultimately result in loss,
including rejection of the OMR answer sheet.
5. Each question in Part ‘A’ carries 2 marks, Part ‘B’ 3.5 marks, Part ‘C’ 5 marks respectively. There will be
negative marking @ 25% (Part ‘A’ 0.50 marks, Part ‘B’ 0.875 marks and Part ‘C’ 1.25 marks) for each
wrong answer.
6. Below each question in Part ‘A’, ‘B’ and ‘C’ four alternative or responses are given. Only one of these
alternatives is the “correct” option to the question. You have to find for each question, the correct or the
best answer.
7. Candidate found copying or resorting to any unfair means are liable to be disqualified rough work.
8. Candidate should not write anything anywhere except on answer sheet of sheet for rough work.
9. Use of calculator is not permitted.
10. After the test is over, at the perforation point, tear the OMR answer sheet, hand over the original
OMR answer sheet to the Invigator and retain the carbonless copy for your record.
11. Candidates who sit for the entire duration of the exam will only be permitted to carry their Test Booklet.

H.No. 40-D, Ground Floor, Jia Sarai, Near IIT, Hauz Khas, New Delhi-110016
Phone: 011-26865455/+91-9871145498
Website: www.physicsbyfiziks.com | Email: fiziks.physics@gmail.com

fiziks
Institute for NET/JRF, GATE, IIT‐JAM, M.Sc. Entrance, JEST, TIFR and GRE in Physics 
 
PART A
Q1. A rectangular photo frame of size 30 cm  40 cm has a photograph mounted at the centre leaving
a 5cm border all around. The area of the border is

(a) 600 cm 2 (b) 350 cm 2 (c) 400 cm 2 (d) 700 cm 2


Q2. At a birthday party, every child gets 2 chocolates, every mother gets 1 chocolate, while no father
gets a chocolate. In total 69 persons get 70 chocolates. If the number of children is half of the
number of mothers and fathers put together, then how many fathers are there?
(a) 22 (b) 23 (c) 24 (d) 69
Q3. What is the value of 12  22  32  42  52  ....  17 2  182  192 ?
(a) 5 (b) 12 (c) 95 (d) 190
Q4. The curves of y  2 x 2 and y  4 x intersect each other at
(a) only one point (b) exactly two points
(c) more than two points (d) no point at all
Q5. The diameters of the pinholes of two otherwise identical cameras A and B are 500  m and
200  m , respectively. Then the image in camera A will be
(a) sharper than in B (b) darker than in B
(c) less sharp and brighter than in B (d) sharper and brighter than in B
Q6. If D  ABC BCA  CAB , where A, B and C are decimal digits, then D is divisible by
(a) 37 and 29 (b) 37 but not 29
(c) 29 but not 37 (d) neither 29 nor 37
Q7. For the following set of observed values
60, 65, 65, 70, 70, 70, 70,82,85,90,95,95,100,160,160
which of the statements is true?
(a) mode < median < mean (b) mode < mean < median
(c) mean < median < mode (d) median < mode < mean
Q8. A circular running track has six lanes, each 1 m wide. How far ahead (in meters) should the
runner in the outermost lane start from, so as to cover the same distance in one lap as the runner
in the innermost lane?
(a) 6  (b) 10  (c) 12  (d) 36 

H.No. 40-D, Ground Floor, Jia Sarai, Near IIT, Hauz Khas, New Delhi-110016
Phone: 011-26865455/+91-9871145498
Website: www.physicsbyfiziks.com | Email: fiziks.physics@gmail.com

fiziks
Institute for NET/JRF, GATE, IIT‐JAM, M.Sc. Entrance, JEST, TIFR and GRE in Physics 
 
Q9. In an examination 100 questions of 1 mark each are given. After the examination, 20 questions
are deleted from evaluation, leaving 80 questions with a total of 100 marks. Student A had
answered 4 of the deleted questions correctly and got 40 marks, whereas student B had
answered 10 of the deleted questions correctly and got 35 marks. In this situation
(a) A and B were equally benefited (b) A and B lost equally
(c) B lost more than A (d) A lost more than B
Q10. A tourist drives 20 km towards east, turns right and drives 6 km , then drives 6 km towards west.
He then turns to his left and drives 4 km and finally turns right and drives 14 km . Where is he
from his starting point?
(a) 6 km towards east (b) 20 km towards west
(c) 14 km towards north (d) 10 km towards south
Q11. If ‘SELDOON’ means ‘NOODLES’ then what does ‘SPUOS’ mean?
(a) SALAD (b) SOUPS (c) RASAM (d) ONION
Q12. An ideal pendulum oscillates with angular amplitude of 300 from the vertical. If it is observed at
a random instant of time, its angular deviation from the vertical is most likely to be
(a) 00 (b) 100 (c) 200 (d) 300
Q13. In the context of tiling a plane surface, which of the following polygons is the odd one out?
(a) Equilateral triangle (b) Square
(c) Regular pentagon (d) Regular hexagon
Q14. Scatter plots for pairs of observations on the variables x and y in samples A and B are shown
in the figure.

y y

A B
x x
Which of the following is suggested by the plots?
(a) Correlation between x and y is stronger in A than in B
(b) Correlation between x and y is absent in B
(c) Correlation between x and y is weaker in A than in B
(d) y and x have a cause - effect relationship in A but not in B

H.No. 40-D, Ground Floor, Jia Sarai, Near IIT, Hauz Khas, New Delhi-110016
Phone: 011-26865455/+91-9871145498
Website: www.physicsbyfiziks.com | Email: fiziks.physics@gmail.com

fiziks
Institute for NET/JRF, GATE, IIT‐JAM, M.Sc. Entrance, JEST, TIFR and GRE in Physics 
 
Q15. Two solutions X and Y containing ingredients A, B and C in proportions a : b : c and c : b : a ,
respectively, are mixed. For the resultant mixture to have A, B and C in equal proportion, it is
necessary that
ca ab a b ca
(a) b  (b) c  (c) c  (d) b 
2 2 2 2
Q16. Find the missing figure in the following sequence.

     
?
  

 
(a) (b)

 
(c) (d)

Q17. In triangle ABC , AB  11, BC  61, AC  60 , and O is the mid-point of BC . Then AO is


A

B C
O
(Not to scale)
(a) 18.5 (b) 24.0 (c) 30.5 (d) 36.0

Q18. Areas of three parts of a rectangle are given in unit of cm 2 . What is the total area of the rectangle?
(a) 18
3 9
(b) 24
(c) 36 6

(d) 108

H.No. 40-D, Ground Floor, Jia Sarai, Near IIT, Hauz Khas, New Delhi-110016
Phone: 011-26865455/+91-9871145498
Website: www.physicsbyfiziks.com | Email: fiziks.physics@gmail.com

fiziks
Institute for NET/JRF, GATE, IIT‐JAM, M.Sc. Entrance, JEST, TIFR and GRE in Physics 
 
Q19. A student is free to choose only Chemistry, only Biology or both. If out of 32 students,
Chemistry has been chosen by 16 and Biology by 25, then how many students have chosen
Biology but not Chemistry?
(a) 9 (b) 16 (c) 25 (d) 7
Q20. The lift (upward force due to air) generated by the wings and engines of an aircraft is
(a) positive (upwards) while landing and negative (downwards) while taking off.
(b) negative (downwards) while landing and positive (upwards) while taking off
(c) negative (downwards) while landing as well as while taking off
(d) positive (upwards) while landing as well as while taking off

H.No. 40-D, Ground Floor, Jia Sarai, Near IIT, Hauz Khas, New Delhi-110016
Phone: 011-26865455/+91-9871145498
Website: www.physicsbyfiziks.com | Email: fiziks.physics@gmail.com

fiziks
Institute for NET/JRF, GATE, IIT‐JAM, M.Sc. Entrance, JEST, TIFR and GRE in Physics 
 
PART B
a 0 0
Q21. One of the eigenvalues of the matrix e is e , where A   0 0 a  . The product of the other
A a

0 a 0
 
two eigenvalues of e A is
(a) e 2a (b) e  a (c) e 2a (d) 1
Q22. The polynomial f  x   1  5 x  3 x 2 is written as linear combination of the Legendre

polynomials
 

1
2
 2



 P0  x   1, P1  x  , P2  x   3 x  1  as f  x    n cn Pn  x  . The value of c0 is

1 1
(a) (b) (c) 2 (d) 4
4 2
dz tanh 2 z 
Q23. The value of the integral  C z sin  z
, where C is a circle of radius , traversed counter-
2
clockwise, with centre at z  0 , is
(a) 4 (b) 4i (c) 2i (d) 0
Q24. A particle of mass m , moving along the x - direction, experiences a damping force  v 2 , where
 is a constant and v is its instantaneous speed. If the speed at t  0 is v0 , the speed at time t is
 v0t
 v0 mv0 2v0
m
(a) v0 e (b) (c) (d)
 v t  m   v0t  v0t
1  ln  1  0  1 e m
 m 

Q25. The integral I   e z dz is evaluated from the point  1, 0  to 1, 0  Im z


C

along the contour C , which is an arc of the parabola y  x 2  1 , as  1, 0  1, 0 


Re z
shown in the figure.
C
The value of I is
(a) 0 (b) 2sinh1 (c) e 2i sinh1 (d) e  e 1

H.No. 40-D, Ground Floor, Jia Sarai, Near IIT, Hauz Khas, New Delhi-110016
Phone: 011-26865455/+91-9871145498
Website: www.physicsbyfiziks.com | Email: fiziks.physics@gmail.com

fiziks
Institute for NET/JRF, GATE, IIT‐JAM, M.Sc. Entrance, JEST, TIFR and GRE in Physics 
 
Q26. In terms of arbitrary constants A and B , the general solution to the differential equation
d2y dy
x2 2
 5 x  3 y  0 is
dx dx
A B
(a) y   Bx 3 (b) y  Ax 
x x3
A B
(c) y  Ax  Bx 3 (d) y  
x x3
k
Q27. In the attractive Kepler problem described by the central potential V  r   (where k is a
r
positive constant), a particle of mass m with a non-zero angular momentum can never reach the
centre due to the centrifugal barrier. If we modify the potential to
k 
V r     3
r r
one finds that there is a critical value of the angular momentum  c below which there is no

centrifugal barrier. This value of  c is


1/ 2 1/ 2 1/ 4 1/ 4
(a) 12km 2   (b) 12km 2   (c) 12km 2   (d) 12km 2  

Q28. The time period of a particle of mass m , undergoing small oscillations around x  0 , in the
x
potential V  V0 cosh   , is
L

mL2 mL2 mL2 2mL2


(a)  (b) 2 (c) 2 (d) 2
V0 2V0 V0 V0

1
Q29. Consider the decay A  B  C of a relativistic spin- particle A . Which of the following
2
statements is true in the rest frame of the particle A ?
1
(a) The spin of both B and C may be
2
(b) The sum of the masses of B and C is greater than the mass of A
(c) The energy of B is uniquely determined by the masses of the particles
(d) The spin of both B and C may be integral

H.No. 40-D, Ground Floor, Jia Sarai, Near IIT, Hauz Khas, New Delhi-110016
Phone: 011-26865455/+91-9871145498
Website: www.physicsbyfiziks.com | Email: fiziks.physics@gmail.com

fiziks
Institute for NET/JRF, GATE, IIT‐JAM, M.Sc. Entrance, JEST, TIFR and GRE in Physics 
 
Q30. Two current-carrying circular loops, each of radius R , are placed z
perpendicular to each other, as shown in the figure.
2I0
The loop in the xy - plane carries a current I 0 while that in the
 y
xz -plane carries a current 2I 0 . The resulting magnetic field B
I0
at the origin is x
0l0  ˆ ˆ  0l0  ˆ ˆ 
(a) 2jk (b) 2jk
2R   2R  

0l0  ˆ ˆ  0l0  ˆ ˆ 
(c) 2 j  k (d) 2 j  k
2R   2R  

Q31. An electric dipole of dipole moment P  qbiˆ is placed at

origin in the vicinity of two charges  q and  q at  L, b 


Y

and  L, b  , respectively, as shown in the figure.


 q ( L, b )
L 
The electrostatic potential at the point  , 0  is 
2  P A
x
O ( L /2, 0)
qb  1 2  4qbL
(a)  2 2 2 
(b)
 0  L L  4b   0  L2  4b 2 
3/ 2

 q ( L,-b)
qb 3qb
(c) (d)
 0 L2  0 L2
Q32. A monochromatic and linearly polarized light is used in a Young’s double slit experiment. A
linear polarizer, whose pass axis is at an angle 450 to the polarization of the incident wave, is
placed in front of one of the slits. If I max and I min , respectively, denote the maximum and
minimum intensities of the interference pattern on the screen, the visibility, defined as the ratio
I max  I min
, is
I max  I min

2 2 2 2 2
(a) (b) (c) (d)
3 3 3 3

H.No. 40-D, Ground Floor, Jia Sarai, Near IIT, Hauz Khas, New Delhi-110016
Phone: 011-26865455/+91-9871145498
Website: www.physicsbyfiziks.com | Email: fiziks.physics@gmail.com

fiziks
Institute for NET/JRF, GATE, IIT‐JAM, M.Sc. Entrance, JEST, TIFR and GRE in Physics 
 
Q33. An electromagnetic wave propagates in a nonmagnetic medium with relative permittivity   4 .
The magnetic field for this wave is

H  x, y   kH 
ˆ cos t   x   3 y
0 

where H 0 is a constant. The corresponding electric field E  x, y  is

(a)
1
4
  
0 H 0 c  3iˆ  ˆj cos t   x   3 y 
(b)
1
4
0 H 0 c   
3iˆ  ˆj cos t   x   3 y 
(c)
1
4
0 H 0 c   
3iˆ  ˆj cos t   x   3 y 
(d)
1
4
  
0 H 0 c  3iˆ  ˆj cos t   x   3 y 
Q34. The ground state energy of an anisotropic harmonic oscillator described by the potential
1
V  x, y, z   m 2 x 2  2m 2 y 2  8m 2 z 2 (in units of  ) is
2
5 7 3 1
(a) (b) (c) (d)
2 2 2 2
Q35. The product x p of uncertainties in the position and momentum of a simple harmonic

oscillator of mass m and angular frequency  in the ground state 0 , is . The value of the
2
product x p in the state, e  ipˆ  /  0 (where  is a constant and p̂ is the momentum operator)

is

 m  2  2
(a) (b)  (c) (d)
2  2 m  2
Q36. Let the wavefunction of the electron in a hydrogen atom be

 1  2  1 
 r   200  r   211  r   100  r 
6 3 6

where nlm  r  are the eigenstates of the Hamiltonian in the standard notation. The expectation

value of the energy in this state is


(a) 10.8 eV (b) 6.2 eV (c) 9.5 eV (d) 5.1 eV

H.No. 40-D, Ground Floor, Jia Sarai, Near IIT, Hauz Khas, New Delhi-110016
Phone: 011-26865455/+91-9871145498
Website: www.physicsbyfiziks.com | Email: fiziks.physics@gmail.com

fiziks
Institute for NET/JRF, GATE, IIT‐JAM, M.Sc. Entrance, JEST, TIFR and GRE in Physics 
 
1
Q37. Three identical spin particles of mass m are confined to a one-dimensional box of length L ,
2
but are otherwise free. Assuming that they are non-interacting, the energies of the lowest two
 22
energy eigen states, in units of , are
2mL2
(a) 3 and 6 (b) 6 and 9 (c) 6 and 11 (d) 3 and 9
Q38. The heat capacity CV at constant volume of a metal, as a function of temperature, is  T   T 3 ,

where  and  are constants. The temperature dependence of the entropy at constant volume is
1
(a)  T   T 3 (b)  T   T 3
3
1 1 1 1
(c) T   T 3 (d) T   T 3
2 3 2 4
2
Q39. The rotational energy levels of a molecule are E      1 , where   0,1, 2,... and I 0 is its
2I0
moment of inertia. The contribution of the rotational motion to the Helmholtz free energy per
molecule, at low temperatures in a dilute gas of these molecules, is approximately
2
 2  
(a)  k BT 1   (b) k BTe I 0 k BT

 I 0 k BT 
2

(c) k BT (d) 3k BTe I 0 k BT

Q40. The vibrational motion of a diatomic molecule may be considered to be that of a simple
harmonic oscillator with angular frequency  . If a gas of these molecules is at temperature T ,
what is the probability that a randomly picked molecule will be found in its lowest vibrational
state?
 
 
(a) 1  e k BT
(b) e 2 k BT

   1   
(c) tanh   (d) cosec h  
 k BT  2  2 k BT 

H.No. 40-D, Ground Floor, Jia Sarai, Near IIT, Hauz Khas, New Delhi-110016
Phone: 011-26865455/+91-9871145498
Website: www.physicsbyfiziks.com | Email: fiziks.physics@gmail.com
10 
fiziks
Institute for NET/JRF, GATE, IIT‐JAM, M.Sc. Entrance, JEST, TIFR and GRE in Physics 
 
Q41. Consider an ideal Fermi gas in a grand canonical ensemble at a constant chemical potential. The
variance of the occupation number of the single particle energy level with mean occupation
number n is
1
(a) n 1  n  (b) n (c) n (d)
n
Q42. Consider the following circuit, consisting of an RS flip-flop and two AND gates.

J 1 A S Q
CLK
K 2 B R Q

Which of the following connections will allow the entire circuit to act as a JK flip-flop?
(a) connect Q to pin 1 and Q to pin 2

(b) connect Q to pin 2 and Q to pin 1

(c) connect Q to K input and Q to J input

(d) connect Q to J input and Q to K input

Q43. The truth table below gives the value Y  A, B, C  where A, B and C are binary variables.

The output Y can be represented by A B C Y


(a) Y  ABC  ABC  ABC  ABC 0 0 0 1
0 0 1 0
(b) Y  ABC  ABC  ABC  ABC
0 1 0 0
(c) Y  ABC  ABC  ABC  ABC 0 1 1 1
1 0 0 1
(d) Y  ABC  ABC  ABC  ABC 1 1 0 0
1 1 1 1

H.No. 40-D, Ground Floor, Jia Sarai, Near IIT, Hauz Khas, New Delhi-110016
Phone: 011-26865455/+91-9871145498
Website: www.physicsbyfiziks.com | Email: fiziks.physics@gmail.com
11 
fiziks
Institute for NET/JRF, GATE, IIT‐JAM, M.Sc. Entrance, JEST, TIFR and GRE in Physics 
 
Q44. A sinusoidal signal is an input to the following circuit
VCC

v R R
Input Vo
C
 0, 0  t C
R

Which of the following graphs best describes the output wave function?

V V
(a) (b)

 0, 0 t  0, 0 t

V V
(c) (d)
 0, 0 t  0, 0 t

Q45. A sinusoidal voltage having a peak value of V p is an input to the following circuit, in which the

DC voltage is Vb
R
V
Vp
Input D Output
t
Vb

H.No. 40-D, Ground Floor, Jia Sarai, Near IIT, Hauz Khas, New Delhi-110016
Phone: 011-26865455/+91-9871145498
Website: www.physicsbyfiziks.com | Email: fiziks.physics@gmail.com
12 
fiziks
Institute for NET/JRF, GATE, IIT‐JAM, M.Sc. Entrance, JEST, TIFR and GRE in Physics 
 
Assuming an ideal diode which of the following best describes the output waveform?

Vp
(a) (b) Vb

t  0, 0  t
Vb

(c) (d)
Vb Vb

 0, 0  t  0, 0  t
Vb
Vp

H.No. 40-D, Ground Floor, Jia Sarai, Near IIT, Hauz Khas, New Delhi-110016
Phone: 011-26865455/+91-9871145498
Website: www.physicsbyfiziks.com | Email: fiziks.physics@gmail.com
13 
fiziks
Institute for NET/JRF, GATE, IIT‐JAM, M.Sc. Entrance, JEST, TIFR and GRE in Physics 
 
PART C
d 2 y  x
Q46. The Green’s function G  x, x  for the equation  f  x  , with the boundary values
dx 2
y  0   0 and y 1  0 , is
1
 2 x 1  x  , 0  x  x  1  x  x  1 , 0  x  x  1
(a) G  x, x    (b) G  x, x   
1
 x 1  x  0  x  x  1  x 1  x  0  x  x  1
2

 1
  x 1  x  , 0  x  x  1  x  x  1 , 0  x  x  1
(c) G  x, x    2 (d) G  x, x   
1
 x 1  x  0  x  x  1  x  x  1 0  x  x  1
 2
Q47. A 4  4 complex matrix A satisfies the relation A† A  4 I , where I is the 4  4 identity matrix.
The number of independent real parameters of A is
(a) 32 (b) 10 (c) 12 (d) 16
Q48. The contour C of the following integral

 z  1 z  3
 dz
z 
3
C
2
 25
in the complex z plane is shown in the figure below.
C

1 3

This integral is equivalent to an integral along the contours

(a) (b)
5 1 3 5 5 1 3 5

(c) (d)
5 1 3 5 5 1 3 5

H.No. 40-D, Ground Floor, Jia Sarai, Near IIT, Hauz Khas, New Delhi-110016
Phone: 011-26865455/+91-9871145498
Website: www.physicsbyfiziks.com | Email: fiziks.physics@gmail.com
14 
fiziks
Institute for NET/JRF, GATE, IIT‐JAM, M.Sc. Entrance, JEST, TIFR and GRE in Physics 
 
1
Q49. The value of the integral 
0
x 2 dx , evaluated using the trapezoidal rule with a step size of 0.2 , is

(a) 0.30 (b) 0.39 (c) 0.34 (d) 0.27

1   dx  
2

Q50. The motion of a particle in one dimension is described by the Langrangian L      x 2 


2   dt  

in suitable units. The value of the action along the classical path from x  0 at t  0 to x  x0 at
t  t0 , is
x02 1 2 1 2 x02
(a) (b) x0 tan t0 (c) x0 cot t0 (d)
2sin 2 t0 2 2 2 cos 2 t0

Q51. The Hamiltonian of a classical one-dimensional harmonic oscillator is H 


2

1 2

p  x 2 , in

suitable units. The total time derivative of the dynamical variable p  2 x is  


(a) 2p  x (b) p  2 x (c) p  2 x (d) x  2 p
Q52. A relativistic particle of mass m and charge e is moving in a uniform electric field of strength
c
 . Starting from rest at t  0 , how much time will it take to reach the speed ?
2
1 mc mc mc 3 mc
(a) (b) (c) 2 (d)
3 e e e 2 e
 
Q53. In an inertial frame uniform electric and magnetic field E and B are perpendicular to each other
2 2
and satisfy E  B  29 (in suitable units). In another inertial frame, which moves at a constant

velocity with respect to the first frame, the magnetic field is 2 5kˆ . In the second frame, an
electric field consistent with the previous observations is

(a)
2

7 ˆ ˆ
ij  (b) 7 iˆ  kˆ   (c) 
7 ˆ ˆ
2
i k  
(d) 7 iˆ  ˆj 
Q54. Electromagnetic wave of angular frequency  is propagating in a medium in which, over a band
2
  vg
of frequencies the refractive index is n    1    , where 0 is a constant. The ratio of
 0  vp
0
the group velocity to the phase velocity at   is
2
1 2
(a) 3 (b) (c) (d) 2
4 3

H.No. 40-D, Ground Floor, Jia Sarai, Near IIT, Hauz Khas, New Delhi-110016
Phone: 011-26865455/+91-9871145498
Website: www.physicsbyfiziks.com | Email: fiziks.physics@gmail.com
15 
fiziks
Institute for NET/JRF, GATE, IIT‐JAM, M.Sc. Entrance, JEST, TIFR and GRE in Physics 
 
Q55. A rotating spherical shell of uniform surface charge and mass density has total mass M and

charge Q . If its angular momentum is L and magnetic moment is  , then the ratio is
L
Q 2Q Q 3Q
(a) (b) (c) (d)
3M 3M 2M 4M
Q56. Consider the operator Ax  Ly pz  Lz p y , where Li and pi denote, respectively, the components

of the angular momentum and momentum operators. The commutator  Ax , x  , where x is the

x - component of the position operator, is


(a) i  zpz  yp y  (b) i  zpz  yp y  (c) i  zpz  yp y  (d) i  zpz  yp y 

p2
Q57. A one-dimensional system is described by the Hamiltonian H    x (where   0 ). The
zm
ground state energy varies as a function of  as
(a)  5/ 3 (b)  2 / 3 (c)  4 / 3 (d)  1/ 3
Q58. If the position of the electron in the ground state of a Hydrogen atom is measured, the probability
that it will be found at a distance r  a0 ( a0 being Bohr radius) is nearest to

(a) 0.91 (b) 0.66 (c) 0.32 (d) 0.13


1
Q59. A system of spin particles is prepared to be in the eigenstate of  z with eigenvalue 1 . The
2
system is rotated by at angle of 600 about the x -axis. After the rotation, the fraction of the
particles that will be measured to be in the eigenstate of  z with eigenvalue 1 is

1 2 1 3
(a) (b) (c) (d)
3 3 4 4

Q60. The Hamiltonian of a one-dimensional Ising model of N spins ( N large) is


N
H   J   i i 1
i 1

1
where the spin  i  1 and J is a positive constant. At inverse temperature   , the
k BT
correlation function between the nearest neighbor spins  i i 1  is
e  J
(a) (b) e2  J (c) tanh   J  (d) coth   J 
 e  J  e  J 

H.No. 40-D, Ground Floor, Jia Sarai, Near IIT, Hauz Khas, New Delhi-110016
Phone: 011-26865455/+91-9871145498
Website: www.physicsbyfiziks.com | Email: fiziks.physics@gmail.com
16 
fiziks
Institute for NET/JRF, GATE, IIT‐JAM, M.Sc. Entrance, JEST, TIFR and GRE in Physics 
 
Q61. At low temperatures, in the Debye approximation, the contribution of the phonons to the heat
capacity of a two dimensional solid is proportional to
(a) T 2 (b) T 3 (c) T 1/ 2 (d) T 3/ 2
Q62. A particle hops on a one-dimensional lattice with lattice spacing a . The probability of the
particle to hop to the neighboring site to its right is p , while the corresponding probability to

hop to the left is q  1  p . The root-mean squared deviation x  x2  x


2
in displacement

after N steps, is q p

O
(a) a Npq (b) aN pq (c) 2a Npq (d) a N

Q63. The energy levels accessible to a molecule have energies E1  0, E2   and E3  2 (where 
is a constant). A gas of these molecules is in thermal equilibrium at temperature T . The specific
heat at constant volume in the high temperature limit  k BT    varies with temperature as

1 1 1 1
(a) 3/ 2
(b) (c) (d)
T T3 T T2
Q64. The input Vi to the following circuit is a square wave as shown in the following figure.

Vi
T /2 T
Vi C R
 0,0  t 
Vo

which of the waveforms best describes the output?


Vo Vo
(a) (b)
 0, 0   0, 0  T /2 3T /2 5T /2 t
T /2 3T /2 5T /2 t

H.No. 40-D, Ground Floor, Jia Sarai, Near IIT, Hauz Khas, New Delhi-110016
Phone: 011-26865455/+91-9871145498
Website: www.physicsbyfiziks.com | Email: fiziks.physics@gmail.com
17 
fiziks
Institute for NET/JRF, GATE, IIT‐JAM, M.Sc. Entrance, JEST, TIFR and GRE in Physics 
 

Vo
(c) (d) Vo

 0, 0  T /2 3T /2 t  0, 0  T /2 3T /2 5T /2 t

Q65. The amplitude of a carrier signal of frequency f 0 is sinusoidally modulated at a

frequency f   f 0 . Which of the following graphs best describes its power spectrum?

Power Power
(a) (b)

f f
(f0 - 3f ')
(f0 - 2f ')
(f0 - 4f')

(f0 + 2f ')

(f0 + 4f')
(f0 + 3f ')
(f0 + f ')

(f0 + f ')
(f0 - f ')
f0

(f0 - f ')
f0
Power Power
(c) (d)

f f
(f0 - 2f ')

(f0 + 2f ')
(f0 + f ')
(f0 - f ')
f0

(f0 + f ')
f0

Q66. The standard deviation of the following set of data


10.0,10.0,9.9,9.9,9.8,9.9,9.9,9.9,9.8,9.9 is nearest to
(a) 0.10 (b) 0.07 (c) 0.01 (d) 0.04
Q67. The diatomic molecule HF has an absorption line in the rotational band at 40 cm 1 for the

isotope 18 F . The corresponding line for the isotope 19


F will be shifted by approximately
(a) 0.05 cm 1 (b) 0.11 cm 1 (c) 0.33 cm 1 (d) 0.01 cm 1

H.No. 40-D, Ground Floor, Jia Sarai, Near IIT, Hauz Khas, New Delhi-110016
Phone: 011-26865455/+91-9871145498
Website: www.physicsbyfiziks.com | Email: fiziks.physics@gmail.com
18 
fiziks
Institute for NET/JRF, GATE, IIT‐JAM, M.Sc. Entrance, JEST, TIFR and GRE in Physics 
 
Q68. The excited state  n  4, l  2  of an election in an atom may decay to one or more of the lower

energy levels shown in the diagram below.

n4
l2

n3
l  0 l 1 l  2

n2
l 1
1
Of the total emitted light, a fraction comes from the decay to the state ( n  2, l  1 ). Based on
4
selection rules, the fractional intensity of the emission line due to the decay to the state
 n  3, l  1
3 1 1
(a) (b) (c) (d) 0
4 2 4
Q69. The volume of an optical cavity is 1 cm3 . The number of modes it can support within a
bandwidth of 0.1 nm , centered at   500 nm , is of the order of

(a) 103 (b) 105 (c) 1010 (d) 107


Q70. Barium Titanate  BaTiO3  crystal has a cubic perovskite structure,

where the Ba 2 ions are at the vertices of a unit cube, the O 2 ions
are at the centres of the faces while the Ti 2 is at the centre. The
number of optical phonon modes of the crystal is
(a) 12 (b) 15
(c) 5 (d) 18
Ti 4 Ba 2  O 2

Q71. The dispersion relation of optical phonons in a cubic crystal is given by   k   0  ak 2 where

0 and a are positive constants. The contribution to the density of states due to these phonons
with frequencies just below 0 is proportional to

(a) 0    (b) 0    (c) 0    (d) 0   


1/ 2 3/ 2 2

H.No. 40-D, Ground Floor, Jia Sarai, Near IIT, Hauz Khas, New Delhi-110016
Phone: 011-26865455/+91-9871145498
Website: www.physicsbyfiziks.com | Email: fiziks.physics@gmail.com
19 
fiziks
Institute for NET/JRF, GATE, IIT‐JAM, M.Sc. Entrance, JEST, TIFR and GRE in Physics 
 
Q72. A silicon crystal is doped with phosphorus atoms. (The binding energy of a H atom is 13.6 eV ,
the dielectric constant of silicon is 12 and the effective mass of electrons in the crystal is 0.4 me ).
The gap between the donor energy level and the bottom of the conduction band is nearest to
(a) 0.01 eV (b) 0.08 eV (c) 0.02 eV (d) 0.04 eV
Q73. Assume that pion-nucleon scattering at low energies, in which isospin is conserved is described
 
by the effective interaction potential Veff  F  r  I .I N , where F  r  is a function of the radial
 
separation r and I and I N denote, respectively, the isospin vectors of a pion and the nucleon.

 I 3/ 2 3 1
The ratio of the scattering cross-sections corresponding to total isospins I  and is
 I 1/ 2 2 2
3 1 5 1
(a) (b) (c) (d)
2 4 4 2
Q74. A nucleus decays by the emission of a gamma ray from an excited state of spin parity 2 to the
ground state with spin-parity 0 what is the type of the corresponding radiation?
(a) magnetic dipole (b) electric quadrupole
(c) electric dipole (d) magnetic quadrupole
Q75. The low lying energy levels due to the vibrational excitations of an even-even nucleus are shown
in the figure below.
0 E4  1.35 MeV
2 E3  1.25 MeV
4 E2  1.17 MeV
 2E

jp E1  0.56 MeV

E

0 E0  0 MeV
The spin-parity j p of the level E1 is

(a) 1 (b) 1 (c) 2 (d) 2

H.No. 40-D, Ground Floor, Jia Sarai, Near IIT, Hauz Khas, New Delhi-110016
Phone: 011-26865455/+91-9871145498
Website: www.physicsbyfiziks.com | Email: fiziks.physics@gmail.com
20 
fiziks
Institute for NET/JRF, GATE, IIT‐JAM, M.Sc. Entrance, JEST, TIFR and GRE in Physics 
 
NET June 2018 (Code B)
PART A
Q1. When a farmer was asked as to how many animals he had, he replied that all but two
were cows, all but two were horses and all but two were pigs. How many animals did he
have?
(a) 3 (b) 6 (c) 8 (d) 12
Q2. Nine eleventh of the members of a parliamentary committee are men. Of the men, two-
thirds are from the Rajya Sabha. Further, 7 /11 of the total committee members are from
the Rajya Sabha. What fraction of the total number are women from the Lok Sabha?
(a) 1/11 (b) 6/11 (c) 2/11 (d) 3/11
Q3. A librarian is arranging a thirteen-volume encyclopedia on the shelf from left to right in
the following order of volume numbers: 8,11,5, 4,9,1, 7, 6,10,3,12, 2 . In this pattern,
where should the volume 13 be placed?
(a) Leftmost (b) Rightmost
(c) Between 10 and 3 (d) Between 9 and 1
Q4. Pick the correct statement:
(a) The sky is blue because Sir C.V. Raman gave the correct explanation.
(b) Copernicus believed that the Sun, and not the Earth, was at the centre of the Solar
system.
(c) The sky appears blue when seen from the Moon..
(d) No solar eclipse is visible for an astronaut standing on the Moon.
What is the last digit of  2017 
2017
Q5. ?

(a) 1 (b) 3 (c) 7 (d) 9


Q6. What is the value of 1  3  5  7  ......  4033  7983  2017 ?

(a) 20170000 (b) 20172017 (c) 20171720 (d) 20172020

H.No. 40‐D, Ground Floor, Jia Sarai, Near IIT, Hauz Khas, New Delhi‐110016 
Phone: 011‐26865455/+91‐9871145498
Website: www.physicsbyfiziks.com  | Email: fiziks.physics@gmail.com  

fiziks
Institute for NET/JRF, GATE, IIT‐JAM, M.Sc. Entrance, JEST, TIFR and GRE in Physics 
 
Q7. Path of a ray of light between two mirrors is shown in the diagram. If the length of each
mirror is ‘ l ’, what is the total path length of the ray between the mirrors?

300

l
3 4 3
(a) l (b) l (c) l (d) 2l
4 3 2
Q8. In a group of 11 persons, each shakes hand with every other once and only once. What is
the total number of such handshakes?
(a) 110 (b) 121 (c) 55 (d) 66
Q9. Suppose (i) “ A  B ” means “ A is the father of B ”, (ii) “ A  B ” means “ A is the husband
of B ”, (iii) “ A B ” means “ A is the wife of B ”, (iv) “ A  B ” means “ A is the sister of
B ”. Which of the following represents “ C is the father-in-law of the sister of D ”
(a) C  E  F  D (b) C  E F  D (c) C  E * F  D (d) C  E F  D
Q10. In a 100 m race A beats B by 10 m . B beats C by 5 m . By how many meters does A
beat C ?
(a) 15.0 m (b) 5.5 m (c) 10.5 m (d) 14.5 m
Q11. If all the angles of a triangle are prime numbers, which of the following could be one
such angle?
(a) 89o (b) 79o (c) 59o (d) 29o
Q12. A water tank that is 40% empty holds 40 L more water than when it is 40% full. How
much water does it hold when it is full?
(a) 100 L (b) 75 L (c) 120 L (d) 200 L
Q13. How much gold and copper (in g), respectively, are required to make a 120 g bar of 22
carat gold?
(a) 90 and 30 (b) 100 and 20
(c) 110 and 10 (d) 120 and 0

H.No. 40‐D, Ground Floor, Jia Sarai, Near IIT, Hauz Khas, New Delhi‐110016 
Phone: 011‐26865455/+91‐9871145498
Website: www.physicsbyfiziks.com  | Email: fiziks.physics@gmail.com  

fiziks
Institute for NET/JRF, GATE, IIT‐JAM, M.Sc. Entrance, JEST, TIFR and GRE in Physics 
 
Q14. Which of the following be the correct pattern in the empty square?

(a) (b) (c) (d)

Q15. Areas of the three rectangles inside the full rectangle are given in the diagram
8
12 4
What is the area of the full rectangle?
(a) 36 (b) 48 (c) 72 (d) 96
Q16. The university needs to appoint a new Vice Chancellor which will be based on seniority.
Ms. West is less senior to Mr. North but more senior to Ms. East. Mr. South is senior to
Ms. West but junior to Mr. North. If the senior most declines the assignment, then who
will be the new vice Chancellor of the University?
(a) Mr. North (b) Mr. East (c) Ms. West (d) Mr. South
Q17. The prices of diamonds having a particular colour and clarity are tabulated below:
Weight of diamond (in carats) Price of diamond (in rupees / carat)
0.25 1 lakh
0.5 2 lakh
1 4 lakh
2 8 lakh
How many 0.25 carat diamonds can be purchased for the price of a 2 carat diamond?
(a) 8 (b) 16 (c) 32 (d) 64

H.No. 40‐D, Ground Floor, Jia Sarai, Near IIT, Hauz Khas, New Delhi‐110016 
Phone: 011‐26865455/+91‐9871145498
Website: www.physicsbyfiziks.com  | Email: fiziks.physics@gmail.com  

fiziks
Institute for NET/JRF, GATE, IIT‐JAM, M.Sc. Entrance, JEST, TIFR and GRE in Physics 
 
Q18. In a sequence of 24 positive integers, the product of any two consecutive integer is 24 .
If the 17th member of the sequence is 6 , the 7th member is
(a) 24 (b) 4 (c) 6 (d) 17
Q19. Mohan lent Geeta as much money as she already had, she then spent Rs. 10 . Next day, he
again lent as much money as Geeta now had, and she spent Rs. 10 again. On the third
day, Mohan again lent as much money as Geeta now had, and she again spent Rs. 10 .If
Geeta was left with no money at the end of third day, how much money did she have
initially?
(a) Rs. 11.25 (b) Rs. 10 (c) Rs. 7.75 (d) Rs. 8.75
Q20. The distribution of marks of students in a class is given by the following chart:
100
90
No. of Students

80
70
60
50
40
30
20
10

1 2 3 4 5 6 7 8 9 10
Marks
If 3.30 marks is the passing score in a 10 mark question paper, which of the following is
false?
(a) Majority of the students have scored above the pass mark
(b) mode of the distribution is 3
(c) Average marks of passing students is above 55%
(d) Average marks of students who have failed is below 20%

H.No. 40‐D, Ground Floor, Jia Sarai, Near IIT, Hauz Khas, New Delhi‐110016 
Phone: 011‐26865455/+91‐9871145498
Website: www.physicsbyfiziks.com  | Email: fiziks.physics@gmail.com  

fiziks
Institute for NET/JRF, GATE, IIT‐JAM, M.Sc. Entrance, JEST, TIFR and GRE in Physics 
 
PART B
Q21. Consider the following ordinary differential equation
2
d 2 x 1  dx  dx
    0
dt 2 x  dt  dt

with the boundary conditions x  t  0   0 and x  t  1  1 . The value of x  t  at t  2 is

(a) e 1 (b) e2  1 (c) e 1 (d) e2  1

Q22. What is the value of a for which f  x, y   2 x  3  x 2  y 2   2i  3xy  ay  is an analytic

function of complex variable z  x  iy


(a) 1 (b) 0 (c) 3 (d) 2
Q23. Two particles A and B move with relativistic velocities of equal magnitude v , but in
opposite directions, along the x -axis of an inertial frame of reference. The magnitude of
the velocity of A , as seen from the rest frame of B , is
2v 2v cv 2v
(a) (b) (c) 2v (d)
 v2   v2  cv v2
1  2  1  2  1
 c   c  c2
Q24. Which of the following figures best describes the trajectory of a particle moving in a
a
repulsive central potential V  r   ( a  0 is a constant)?
r
y y
(a) (b)

x x
O O

y y

(c) (d)
x x
O O

H.No. 40‐D, Ground Floor, Jia Sarai, Near IIT, Hauz Khas, New Delhi‐110016 
Phone: 011‐26865455/+91‐9871145498
Website: www.physicsbyfiziks.com  | Email: fiziks.physics@gmail.com  

fiziks
Institute for NET/JRF, GATE, IIT‐JAM, M.Sc. Entrance, JEST, TIFR and GRE in Physics 
 
  
Q25. Consider the three vectors v1  2iˆ  3kˆ, v2  iˆ  2 ˆj  2kˆ and v3  5iˆ  ˆj  akˆ where iˆ, ˆj

and k̂ are the standard unit vectors in a three-dimensional Euclidean space. These vectors
will be linearly dependent if the value of a is
31 23 27
(a) (b) (c) (d) 0
4 4 4

 dxf  x  eikx of the function f  x   e
x
Q26. The Fourier transform


2 1 2 2
(a)  (b)  (c) (d)
1 k 2 2 1  k 2  1 k 2 2  k 2 
Q27. The value of the integral
 /2 1
 dx  dy.  sin 2 x    x  y  is
 /2 1

1 1
(a) 0 (b) (c) (d) 1
2 2
Q28. A particle moves in the one-dimensional potential V  x    x 6 , where a  0 is a constant.

If the total energy of the particle is E , its time period in a periodic motion is proportional
to
(a) E 1/ 3 (b) E 1/ 2 (c) E1/ 3 (d) E1/ 2
Q29. Two point charges 2Q and Q are kept at point with Cartesian coordinates 1, 0, 0  ,

respectively, in front of an infinite grounded conducting plate at x  0 . The potential at


 x, 0, 0  for x  1 depends on x as

(a) x 3 (b) x 5 (c) x 2 (d) x 4


Q30. Two Stern-Gerlach apparatus S1 and S2 are kept in a line ( x -axis). The directions of

their magnetic fields are along the positive z and y -axes, respectively. Each apparatus
only transmits particles with spins aligned in thee direction of its magnetic field. If an
1
initially unpolarized bearn of spin particles passes through this configuration, the ratio
2
of intensities l0 : l f of the initial and final beams is

l0 lf
k̂ ĵ

(a) 16 :1 (b) 2 :1 (c) 4 :1 (d) 1: 0


H.No. 40‐D, Ground Floor, Jia Sarai, Near IIT, Hauz Khas, New Delhi‐110016 
Phone: 011‐26865455/+91‐9871145498
Website: www.physicsbyfiziks.com  | Email: fiziks.physics@gmail.com  

fiziks
Institute for NET/JRF, GATE, IIT‐JAM, M.Sc. Entrance, JEST, TIFR and GRE in Physics 
 
Q31. The following configuration of three identical narrow slits are illuminated by
monochromatic light of wavelength  (as shown in the figure below). The intensity is
measured at an angle  (where  is the angle with the incident beam) at a large distance
2 d
from the slits. If   sin  , the intensity is proportional to

(a) 2 cos   2 cos 2
d
1
(b) 3  sin 3
2

2 
(c) 3  2 cos   2 cos 2  2 cos 3 2d

1
(d) 2  sin 2 3
2
1 1
Q32. A particle of mass m , kept in potential V  x    kx 2   x 4 (where k and  are
2 4
positive constants), undergoes small oscillations about an equilibrium point. The
frequency of oscillations is

1 2 1 k 1 2k 1 
(a) (b) (c) (d)
2 m 2 m 2 m 2 m
1   
Q33. The Hamiltonian of a spin particle in a magnetic field B is given by H    .B. ,
2

where  is a real constant and    x ,  y ,  z  are the Pauli spin matrices. If

B   B0 , B0 , 0  and the spin state at time t  0 is an eigenstate of  x , then of the

expectation values  x ,  y and  z

(a) only  x changes with time (b) only  y changes with time

(c) only  z changes with time (d) all three change with time

Q34. A particle of mass m is constrained to move in a circular ring of radius R . When a


a
perturbation V   2
cos 2  (where a is a real constant) is added, the shift in energy of
R
the ground state, to first order in a , is
a 2a a a
(a) (b) (c) (d)
R2 R2 2R2  R 2 
H.No. 40‐D, Ground Floor, Jia Sarai, Near IIT, Hauz Khas, New Delhi‐110016 
Phone: 011‐26865455/+91‐9871145498
Website: www.physicsbyfiziks.com  | Email: fiziks.physics@gmail.com  

fiziks
Institute for NET/JRF, GATE, IIT‐JAM, M.Sc. Entrance, JEST, TIFR and GRE in Physics 
 
Q35. A particle of mass m is confined in a three-dimensional box by the potential
0, 0  x, y, z  a
V  x, y , z   
 otherwise

9 2 2
The number of eigenstates of Hamiltonian with energy is
2ma 2
(a) 1 (b) 6 (c) 3 (d) 4
 
Q36. The electric field E and the magnetic field B corresponding to the scalar and vector
 1
potentials, V  x, y, z , t   0 and A  x, y, z , t   kˆ0 A0  ct  x  , where A0 is a constant,
2
are
  1  1  1
(a) E  0 and B  ˆj 0 A0 (b) E   kˆ0 A0 c and B  ˆj 0 A0
2 2 2
  1  1  1
(c) E  0 and B   iˆ0 A0 (d) E  kˆ0 A0 c and B   iˆ0 A0
2 2 2
Q37. The electric field of a plane wave in a conducting medium is given by

ˆ 0 e z / 3a cos  z  t  ,
E  z , t   iE  
 3a 
where  is the angular frequency and a  0 is a constant. The phase difference between
 
the magnetic field B and the electric field E is
   
(a) 300 and B lags behind B (b) 300 and B lags behind E
   
(c) 600 and E lags behind B (d) 600 and B lags behind E
Q38. Which of the following statements concerning the coefficient of volume expansion 
and the isothermal compressibility  of a solid is true?
(a)  and  are both intensive variables
(b)  is an intensive and  is an extensive variable
(c)  is an extensive and  is an intensive variable
(d)  and  are both extensive variables

H.No. 40‐D, Ground Floor, Jia Sarai, Near IIT, Hauz Khas, New Delhi‐110016 
Phone: 011‐26865455/+91‐9871145498
Website: www.physicsbyfiziks.com  | Email: fiziks.physics@gmail.com  

fiziks
Institute for NET/JRF, GATE, IIT‐JAM, M.Sc. Entrance, JEST, TIFR and GRE in Physics 
 
Q39. A sinusoidal signal with a peak voltage V p and average value zero, is an input to the

following circuit.

Vp
Input voltage RL Output
voltage
C

Assuming ideal diodes, the peak value of the output voltage across the load resistor RL is
V
(a) V p (b) p (c) 2V p (d) 2V p
2
Q40. In the following circuit, the value of the common-emitter forward current amplification
factor  for the transistor is 100 and VBE is 0.7V .

VCC 20.7 V

500 k R1 R2
5k
Ib Output
Input
Ik
The base current I B is

(a) 40  A (b) 30  A (c) 44  A (d) 33  A


Q41. The number of ways of distributing 11 indistinguishable bosons in 3 different energy
levels is

(a) 311 (b) 113 (c)


13! (d)
11!
2!11 ! 3!8!

 a 
Q42. The van der Waals equation for one mole of a gas is  p  2  V  b   RT . The
 V 
corresponding equation of state for n moles of this gas at pressure P , volume V and
temperature T , is
 an 2   a 
(a)  P  2  V  nb   nRT (b)  P  2  V  nb   nRT
 V   V 

 an 2   a 
(c)  P  2  V  nb   nRT (d)  P  2  V  nb   nRT
 V   V 

H.No. 40‐D, Ground Floor, Jia Sarai, Near IIT, Hauz Khas, New Delhi‐110016 
Phone: 011‐26865455/+91‐9871145498
Website: www.physicsbyfiziks.com  | Email: fiziks.physics@gmail.com  

fiziks
Institute for NET/JRF, GATE, IIT‐JAM, M.Sc. Entrance, JEST, TIFR and GRE in Physics 
 
Q43. In a system of N distinguishable particles, each particle can be in one of two states with
energies 0 and  E , respectively. The mean energy of the system at temperature T is
1

(a)  N 1  e / kBT
2
 
(b)  NE 1  e / kBT 
1
(c)  NE
2

(d)  NE 1  e  / kBT 
Q44. In the following JK flip-flop circuit, J and K inputs are tied together to VCC . If the

input is a clock signal of frequency f , the frequency of the output Q is


(a) f VCC
I Q
(b) 2 f
(c) 4 f Clock signal
f K Q
(d)
2
Q45. Which of the following gates can be used as a parity checker?
(a) an OR gate (b) a NOR gate
(c) an exclusive OR (XOR) gate (d) an AND gate

H.No. 40‐D, Ground Floor, Jia Sarai, Near IIT, Hauz Khas, New Delhi‐110016 
Phone: 011‐26865455/+91‐9871145498
Website: www.physicsbyfiziks.com  | Email: fiziks.physics@gmail.com  
10 
fiziks
Institute for NET/JRF, GATE, IIT‐JAM, M.Sc. Entrance, JEST, TIFR and GRE in Physics 
 
PART C
Q46. Which of the following statements is true for a 3  3 real orthogonal matrix with
determinant 1 ?
(a) the modulus of each of its eigenvalues need not be 1 , but their product must be 1
(b) at least one of its eigenvalues is 1
(c) all of its eigenvalues must be real
(d) none of its eigenvalues must be real
k
Q47. A particle of mass m moves in a central potential V  r    in an elliptic orbit
r
a 1  e 2 
r    , where 0    2 and a and e denote the semi-major axis and
1  e cos 
k
eccentricity, respectively. If its total energy is E   , the maximum kinetic energy is
2a
 e  1  e  1
(a) E 1  e 2  (b) E (c) E / 1  e 2  (d) E
 e  1  e  1
xp 2 1
Q48. The Hamiltonian of a one-dimensional system is H   kx , where m and k are
2m 2
positive constants. The corresponding Euler-Lagrange equation for the system is
(a) mx  k  0 (b) mx  2 x  kx 2  0
(c) 2mxx  mx 2  kx 2  0 (d) mxx  2mx 2  kx 2  0
Q49. A hollow waveguide supports transverse electric TE  modes with the dispersion

1
relation k   2  mn
2
, where mn is the mode frequency. The speed of flow of
c
electromagnetic energy at the mode frequency is
(a) c (b) mn / k (c) 0 (d) 
2
Q50. The energy of a free relativistic particle is E  p c 2  m 2 c 4 , where m is its rest mass,

p is its momentum and c is the speed of light in vacuum. The ratio vg / v p of the group

velocity vg of a quantum mechanical wave packet (describing this particle) to the phase

velocity v p is
  2 
(a) p c / E (b) p mc 3 / E 2 (c) p c 3 / E 2 (d) p c / 2 E

H.No. 40‐D, Ground Floor, Jia Sarai, Near IIT, Hauz Khas, New Delhi‐110016 
Phone: 011‐26865455/+91‐9871145498
Website: www.physicsbyfiziks.com  | Email: fiziks.physics@gmail.com  
11 
fiziks
Institute for NET/JRF, GATE, IIT‐JAM, M.Sc. Entrance, JEST, TIFR and GRE in Physics 
 
In the function Pn  x  e  x of a real variable x , Pn  x  is polynomial of degree n . The
2
Q51.

maximum number of extrema that this function can have is


(a) n  2 (b) n  1 (c) n  1 (d) n
d 2 y  x
Q52. The Green’s function G  x, x  for the equation  y  x   f  x  , with the
dx 2
 
boundary values y  0   y    0 , is
2
   
 x  x  2  , 0  x  x 
   2
(a) G  x, x    
 x    x , 0  x  x 


 
2 2

 
 cos x  sin x, 0  x  x   2
(b) G  x, x    
 sin x  cos x, 0  x   x  
 2

 
cos x  sin x, 0  x  x 
(c) G  x, x   
 2
sin x  cos x, 
0  x  x 
 2

   
 x  2  x  , 0  x  x 
   2
(d) G  x, x    
 x    x  , 0  x  x 

  2 
 2
1 1
Q53. The fractional error in estimating the integral 0
xdx using Simpson’s
3
rule, using a step

size 0.1 , is nearest to


(a) 104 (b) 0 (c) 102 (d) 3 104

H.No. 40‐D, Ground Floor, Jia Sarai, Near IIT, Hauz Khas, New Delhi‐110016 
Phone: 011‐26865455/+91‐9871145498
Website: www.physicsbyfiziks.com  | Email: fiziks.physics@gmail.com  
12 
fiziks
Institute for NET/JRF, GATE, IIT‐JAM, M.Sc. Entrance, JEST, TIFR and GRE in Physics 
 
Q54. An inertial frame K  moves with a constant speed v with respect to another inertial
frame K along their common x - direction. Let  x, ct  and  x , ct   denote the space-

time coordinates in the frames K and K  , respectively. Which of the following space-
time diagrams correctly describes the t  - axis  x   0 line  and the x  - axis  t   0 line 

in the x-ct plane? (In the following figures tan   v / c )


ct ct  ct ct 

(a)

(c)
 x

x  x

x

ct  ct ct  ct

(c) x
(d)
 

 x
x 

x

Q55. The loop shown in the figure below carries a steady current I .
I

3a
a
O
The magnitude of the magnetic field at the point O is
0 I 0 I 0 I 0 I
(a) (b) (c) (d)
2a 6a 4a 3a

H.No. 40‐D, Ground Floor, Jia Sarai, Near IIT, Hauz Khas, New Delhi‐110016 
Phone: 011‐26865455/+91‐9871145498
Website: www.physicsbyfiziks.com  | Email: fiziks.physics@gmail.com  
13 
fiziks
Institute for NET/JRF, GATE, IIT‐JAM, M.Sc. Entrance, JEST, TIFR and GRE in Physics 
 
Q56. In the region far from a source, the time dependent electric field at a point  r ,  ,   is

  sin     r 
E  r ,  ,    ˆ E0 2   cos   t   
 r    c 
where  is angular frequency of the source. The total power radiated (averaged over a
cycle) is
2 E02 4 4 E02 4 4 E02 4 2 E02 4
(a) (b) (c) (d)
3 0 c 3 0 c 3 0 c 3 0 c
Q57. The pressure P of a system of N particles contained in a volume V at a temperature T
1 1
is given by P  nk B T  an 2  bn3 , where n is the number density and a and b are
2 6
temperature independent constants. If the system exhibits a gas-liquid transition, the
critical temperature is
a a a2 a2
(a) (b) (c) (d)
bk B 2b 2 k B 2bk B b2 kB
Q58. Consider a particle diffusing in a liquid contained in a large box. The diffusion constant
of the particle in the liquid is 1.0  102 cm 2 / s . The minimum time after which the root-
mean-squared displacement becomes more than 6 cm is

(a) 10 min (b) 6 min (c) 30 min (d) 6 min

Q59. A thermally insulated chamber of dimensions  L, L, 2 L  is partitioned in the middle. One

side of the chamber is filled with n moles of an ideal gas at a pressure P and
temperature T , while the other side is empty. At t  0 , the partition is removed and the
gas is allowed to expand freely. The time to reach equilibrium varies as

L P, T

2L
(a) n1/ 3 L1T 1/ 2 (b) n 2 / 3 LT 1/ 2 (c) n0 LT 1/ 2 (d) n L1T 1/ 2

H.No. 40‐D, Ground Floor, Jia Sarai, Near IIT, Hauz Khas, New Delhi‐110016 
Phone: 011‐26865455/+91‐9871145498
Website: www.physicsbyfiziks.com  | Email: fiziks.physics@gmail.com  
14 
fiziks
Institute for NET/JRF, GATE, IIT‐JAM, M.Sc. Entrance, JEST, TIFR and GRE in Physics 
 
Q60. Two signals A1 sin  t  and A2 cos t  are fed into the input and the reference channels,

respectively, of a lock-in amplifier. The amplitude of each signal is 1 V . The time


constant of the lock-in amplifier is such that any signal of frequency larger than  is
filtered out. The output of the lock-in amplifier is
(a) 2 V (b) 1 V (c) 0.5 V (d) 0 V
o
Q61. The maximum intensity of solar radiation is at the wavelength of s u n ~ 5000 A and

corresponds to its surface temperature Tsun ~ 104 K . If the wavelength of the maximum
o
intensity of an X -ray star is 5 A , its surface temperature is of the order of
(a) 1016 K (b) 1014 K (c) 1010 K (d) 107 K
Q62. The full scale of a 3 -bit digital-to-analog (DAC) converter is 7V . Which of the following
tables represents the output voltage of this 3 -bit DAC for the given set of input bits?

Input bits Output voltage Input bits Output voltage


(a) 000 0 (b) 000 0
001 1 001 1.25
010 2 010 2.5
011 3 011 3.75

Input bits Output voltage Input bits Output voltage


(c) (d)
000 1.25 000 1
001 2.5 001 2
010 3.75 010 3
011 5 011 4

H.No. 40‐D, Ground Floor, Jia Sarai, Near IIT, Hauz Khas, New Delhi‐110016 
Phone: 011‐26865455/+91‐9871145498
Website: www.physicsbyfiziks.com  | Email: fiziks.physics@gmail.com  
15 
fiziks
Institute for NET/JRF, GATE, IIT‐JAM, M.Sc. Entrance, JEST, TIFR and GRE in Physics 
 
Q63. The input Vi to the following circuit is a square wave as shown in the following figure
Vi
T /2 T C R
 0, 0  t 
V

Which of the waveforms V0 best describes the output?

Vo Vo
(a) T /2 T (b)
 0, 0  t  0, 0  T
T /2 t

Vo Vo
T /2 T T /2 T
(c) (d)
 0, 0  t
 0, 0  t

p2
Q64. The nth energy eigenvalues En of a one-dimensional Hamiltonian H    x 4 (where
2m
  0 is a constant) in the WEB approximation, is proportional to
4/3 4/3
 1  1
(a)  n    1/ 3
(b)  n    2/3
 2  2
5/3 5/ 3
 1  1
(c)  n    1/ 3
(d)  n    2/3
 2  2
d 
Q65. The differential scattering cross-section for the central potential V  r   e   r ,
d r
where  and  are positive constants, is calculated in thee first Born approximation. Its
dependence on the scattering angle  is proportional to ( A is a constant below)
1
   
(a)  A2  sin 2  (b)  A2  sin 2 
 2  2
2
 
2
 
(c)  A2  sin 2  (d)  A2  sin 2 
 2  2

H.No. 40‐D, Ground Floor, Jia Sarai, Near IIT, Hauz Khas, New Delhi‐110016 
Phone: 011‐26865455/+91‐9871145498
Website: www.physicsbyfiziks.com  | Email: fiziks.physics@gmail.com  
16 
fiziks
Institute for NET/JRF, GATE, IIT‐JAM, M.Sc. Entrance, JEST, TIFR and GRE in Physics 
 
Q66. At t  0 , the wavefunction of an otherwise free particle confined between two infinite
2  x 3 x 
walls at x  0 and x  L is   x, t  0    sin  sin  . Its wave function at a
L L L 
mL2
later time t  is
4 h
2  x 3 x  i / 6 2  x 3 x   i / 6
(a)  sin  sin e (b)  sin  sin e
L L L  L L L 

2  x 3 x   i / 8 2  x 3 x   i / 6
(c)  sin  sin e (d)  sin  sin e
L L L  L L L 

Q67. Sodium Chloride  NaCl  crystal is a face-centered cubic lattice with a basis consisting of

Na  and Cl  ions separated by half the body diagonal of a unit cube. Which of the
planes corresponding to the Miller indices given below will not give rise to Bragg
reflection of X -rays?
(a)  220  (b)  242  (c)  221 (d)  311

Q68. The dispersion relation for the electrons in the conduction band of a semiconductor is
given by E  E0   k 2 where  and E0 are constants. If c is the cyclotron resonance
frequency of the conduction band electrons in a magnetic field B , the value of  is
c 2 2c  2c  2c
(a) (b) (c) (d)
4eB eB eB 2eB
Q69. Hard disc of radius R are arranged in a two-dimensional triangular lattice. What is the
fractional area occupied by the discs in the closest possible packing?

 3   2 2
(a) (b) (c) (d)
6 3 2 5 7
Q70. A photon of energy 115.62 keV ionizes a K -shell electron of a Be atom. One L -shell
electron jumps to the K -shell to fill this vacancy and emits a photon of energy
109.2 keV in the process. If the ionization potential for the L -shell is 6.4 keV , the
kinetic energy of the ionized electron is
(a) 6.42 keV (b) 12.82 keV (c) 20 eV (d) 32 eV
Q71. The value of the Lande g - factor for a fine-structure level defined by the quantum
number L  1, J  2 and S  1 , is
11 4 8 3
(a) (b) (c) (d)
6 3 3 2
H.No. 40‐D, Ground Floor, Jia Sarai, Near IIT, Hauz Khas, New Delhi‐110016 
Phone: 011‐26865455/+91‐9871145498
Website: www.physicsbyfiziks.com  | Email: fiziks.physics@gmail.com  
17 
fiziks
Institute for NET/JRF, GATE, IIT‐JAM, M.Sc. Entrance, JEST, TIFR and GRE in Physics 
 
Q72. The electronic energy level diagram of a molecule is shown in the following figure,
2
23

3
 21 34 Laser light
4

41
1
Let  ij denote the decay rate for a transition from the level i to j . The molecules are

optically pumped from level 1 to 2 . For the transition from level 3 to level 4 to be a
lassing transition, the decay rates have to satisfy
(a)  21   23   41  34 (b)  21   41   23  34

(c)  41   23   21  34 (d)  41   21  34   23

Q73. The reaction 63


Cu29  p  63
Zn30  n is followed by a prompt   decay of zinc
63
Zn30  63Cu29  e  ve . If the maximum energy of the position is 2.4 MeV , the Q -

value of the original reaction in MeV is nearest to


[Take the masses of electron, proton and neutron to be 0.5 MeV / c 2 ,938 MeV / c 2 and

939.5 MeV / c 2 ,respectively.]


(a) 4.4 (b) 2.4 (c) 4.8 (d) 3.4
Q74. A deuteron d captures a charged pion   in the l  1 state, and subsequently decays into
a pair of neutrons  n  via strong interaction. Given that the intrinsic parities of   , d and

n are 1, 1 and 1 respectively, the spin wavefunction of the final state neutrons is
(a) linear combination of a singlet and a triplet
(b) singlet
(c) triplet
(d) doublet
Q75. Which of the following elementary particle processes does not conserve strangeness?
(a)  0  p  k    0 (b)    p  k 0   0

(c)  0   0  n (d) K 0      
H.No. 40‐D, Ground Floor, Jia Sarai, Near IIT, Hauz Khas, New Delhi‐110016 
Phone: 011‐26865455/+91‐9871145498
Website: www.physicsbyfiziks.com  | Email: fiziks.physics@gmail.com  
18 
fiziks
Institute for NET/JRF, GATE, IIT‐JAM, M.Sc. Entrance, JEST, TIFR and GRE in Physics 
 
NET December 2017
PART A
Q1. A leaf appears green in daylight. If this leaf were observed in red light, what colour
would it appear to have?
(a) Green (b) Black-Brown (c) Red (d) Blue
Q2. The distance from Nehrunagar to Gandhinagar is 27 km . A and B start walking from
Nehrunagar towards Gandhinagar at speeds of 5 km / hr and 7 km / hr , respectively. B
reaches Gandhinagar, returns immediately, and meets A at Indiranagar. What is the
distance between Nehrunagar and Indiranagar? (Assume all three cities to be in one
straight line)
(a) 12.5 km (b) 22.5 km (c) 4.5 km (d) 13.5 km
Q3. A sphere G of radius b is fixed mid-air and several spheres identical to the first one are
shot at it with their velocities parallel to each other. If the shot spheres fall within an
imaginary cylinder of radius a  b  a  , then the fraction of spheres that will hit G is

(a) 2b / a (b) 4b 2 / a 2 (c)  a  b  /  a  b  (d) 8b3 / a 3

Q4. Five persons A, B, C , D and E are sifting in a row with C in the middle of the group. If
D is at an extreme end and there are at least two persons between B and E , then which
of the following statements is incorrect?
(a) E can be on extreme left (b) E can be on extreme right
(c) A cannot be on extreme left (d) A is always a neighbour of B or D
Q5. In a group of students, 30 % play only cricket, 20 % play only football and 10 % play
only basketball. 20 % of the students play both football and cricket, 15 % play both
basketball and cricket, 10 % play both football and basketball. 15 students play no
games, while 5 % of the students play all three games. What is the total number of
students?
(a) 300 (b) 250 (c) 350 (d) 400

H.No. 40‐D, Ground Floor, Jia Sarai, Near IIT, Hauz Khas, New Delhi‐110016 
Phone: 011‐26865455/+91‐9871145498
Website: www.physicsbyfiziks.com  | Email: fiziks.physics@gmail.com  

fiziks
Institute for NET/JRF, GATE, IIT‐JAM, M.Sc. Entrance, JEST, TIFR and GRE in Physics 
 
Q6. When Ramesh was at the age of 8 years, he hammered a nail into a large tree to mark his
height. If the tree grows 2 cm/year, how much higher would the nail be after 5 years?
(a) 5cm higher (b) 0 cm higher (c) 10 cm higher (d) 8cm higher
Q7. Find the missing number

17 15 13 12 25 24 41 40
8 5 7 ?

(a) 4 (b) 9 (c) 3 (d) 6

Q8.
A

B won
lost
C

10 20 30 40 50 60 70 80 90 100110120130140150
Number of seats
The bar chart above shows number of seats won by four political parties A, B, C and D .
Which party won the largest proportion of seats it contested?
(a) A (b) B (c) C (d) D
Q9. The molar fraction of hydrochloric acid in an extremely dilute’ aqueous solution is
doubled. The pH of the resulting solution is
(a) approximately doubled (b) approximately halved
(c) increased (d) reduced
Q10. Approximately how much blood flows per day through a normal human heart beating 70
times per minute, having a relaxed volume of 110 cc and compressed volume of 70 cc ?
(a) 7150 litres (b) 4000 litres (c) 28000 litres (d) 11100 litres

H.No. 40‐D, Ground Floor, Jia Sarai, Near IIT, Hauz Khas, New Delhi‐110016 
Phone: 011‐26865455/+91‐9871145498
Website: www.physicsbyfiziks.com  | Email: fiziks.physics@gmail.com  

fiziks
Institute for NET/JRF, GATE, IIT‐JAM, M.Sc. Entrance, JEST, TIFR and GRE in Physics 
 
Q11. The number of three English letter words, having at least one consonant, but not having
two consecutive consonants, is
(a) 2205 (b) 3780 (c) 2730 (d) 3360
Q12. Which one of the following graphs represents f  x   sin x cos x ?
1 0.5
(a) 0.5 (b)
0 0
0.5
1 0.5
0  /2  3 / 2 2 0  /2  3 / 2 2

1 0.5

(c) 0.5 (d)


0 0
0.5
1 0.5
0  /2  3 / 2 2 0  /2  3 / 2 2
Q13. There are two gas parcels of equal volume, A and B at the same temperature and
pressure. Parcel A is one mole of water vapour, while parcel B is one mole of dry air.
Which of the following is TRUE?
(a) Parcel A is heavier then Parcel B
(b) Parcel B is heavier than Parcel A
(c) Both parcels are equally heavy
(d) Without temperature and pressure data, their relative masses cannot be determined
Q14. For which of the following numbers is its positive square root closest to the number
itself’?
(a) 0.33 (b) 0.99 (c) 0.89 (d) 0.10

H.No. 40‐D, Ground Floor, Jia Sarai, Near IIT, Hauz Khas, New Delhi‐110016 
Phone: 011‐26865455/+91‐9871145498
Website: www.physicsbyfiziks.com  | Email: fiziks.physics@gmail.com  

fiziks
Institute for NET/JRF, GATE, IIT‐JAM, M.Sc. Entrance, JEST, TIFR and GRE in Physics 
 
Q15. Find the next pattern in the following sequence:

(a) (b)

(c) (d)

Q16. DRQP is a small square of side a in the corner of a big square D R C


ABCD of side A. What is the ratio of the area of the quadrilateral P Q
PBRQ to that of the square ABCD , given A / a  3 ?
(a) 2 / 9 (b) 1/ 6
(c) 1/ 3 (d) 2 / 7 A B

Q17. A 100 m long fence is to be made by fixing a wire mesh on steel poles. Each pole has a

1 m vertical portion and a 1 m portion tilted at 450 to the vertical. What will be the area
of wire mesh required?
(a) 200 m 2 (b) 241.4 m 2 (c) 400 m 2 (d) 170.7 m 2
Q18. The average staff salary of a company is Rs. 8000 / - . A new guard and a new manager
are recruited with salaries of Rs. 5000 / - and 20000 / - , respectively. What is the current
staff strength if the new average salary is Rs. 4000 / - more than that of the guard?
(a) 7 (b) 9 (c) 10 (d) 11
Q19. A bird flies along the three sides of a field in the shape of an equilateral triangle at speeds
of 2, 4,8 km / hr , respectively. The average speed of the bird is
24 14 22
(a) km / hr (b) km / hr (c) km / hr (d) 4 km / hr
7 3 7

H.No. 40‐D, Ground Floor, Jia Sarai, Near IIT, Hauz Khas, New Delhi‐110016 
Phone: 011‐26865455/+91‐9871145498
Website: www.physicsbyfiziks.com  | Email: fiziks.physics@gmail.com  

fiziks
Institute for NET/JRF, GATE, IIT‐JAM, M.Sc. Entrance, JEST, TIFR and GRE in Physics 
 
Q20. A buys n copies of a book at 20 % discount. B gets the same book at 30 % discount.
What is the minimum value of it for which B can buy one extra copy of the book,
spending the same amount as A ?
(a) 7
(b) 8
(c) 6
(d) This problem cannot be solved unless the marked price of the book is known.

H.No. 40‐D, Ground Floor, Jia Sarai, Near IIT, Hauz Khas, New Delhi‐110016 
Phone: 011‐26865455/+91‐9871145498
Website: www.physicsbyfiziks.com  | Email: fiziks.physics@gmail.com  

fiziks
Institute for NET/JRF, GATE, IIT‐JAM, M.Sc. Entrance, JEST, TIFR and GRE in Physics 
 
PART ‘B’
Q21. Let A be a non-singular 3  3 matrix, the columns of which are denoted by the vectors
     
a , b and c , respectively. Similarly, u , v and w denote the vectors that form the

 
1
corresponding columns of AT . Which of the following is true?
           
(a) u  a  0, u  b  0, u  c  1 (b) u  a  0, u  b  1, u  c  0
           
(c) u  a  1, u  b  0, u  c  0 (d) u  a  0, u  b  0, u  c  0

Q22.  
Consider the real function f  x   1/ x 2  4 . The Taylor expansion of f  x  about x  0

converges
(a) for all values of x (b) for all values of x except x  2
(c) in the region 2  x  2 (d) for x  2 and x  2
Q23. Consider the matrix equation
1 1 1  x  0
    
1 2 3  y   0
 2 b 2c   z   0 
    
The condition for existence of a non-trivial solution and the corresponding normalised
solution (upto a sign) is
1
(a) b  2c and  x, y, z   1, 2, 1
6
1
(b) c  2b and  x, y, z   1, 1, 2 
6
1
(c) c  b  1 and  x, y, z    2, 1, 1
6
1
(d) b  c  1 and  x, y, z   1, 2, 1
6

H.No. 40‐D, Ground Floor, Jia Sarai, Near IIT, Hauz Khas, New Delhi‐110016 
Phone: 011‐26865455/+91‐9871145498
Website: www.physicsbyfiziks.com  | Email: fiziks.physics@gmail.com  

fiziks
Institute for NET/JRF, GATE, IIT‐JAM, M.Sc. Entrance, JEST, TIFR and GRE in Physics 
 
dy
Q24. Consider the differential equation  ay  e bt with the initial condition y  0   0 .
dt
Then the Laplace transform Y  s  of the solution y  t  is

1 1 1 e a  eb
(a) (b) (c) (d)
 s  a  s  b  b s  a a  s  b ba

Q25. A light signal travels from a point A to a point B , both within a glass slab that is moving
with uniform velocity (in the same direction as the light) with speed 0.3c with respect to
an external observer. If the refractive index of the slab is 1.5 , then the observer will
measure the speed of the signal as
(a) 0.67 c (b) 0.81 c (c) 0.97 c (d) c
Q26. A monoatomic gas ‘of volume V is in equilibrium in a uniform vertical cylinder, the
lower end of which is closed by a rigid wall and the other by a frictionless piston. The
piston is pressed lightly and released. Assume that the gas is a poor conductor of heat and
the cylinder and piston are perfectly insulating. If the cross-sectional area of the cylinder
is A , the angular frequency of small oscillations of the piston about the point of
equilibrium, is
5
(a) 5 gA /  3V  (b) 4 gA /  3V  (c) gA / V (d) 7 gA /  5V 
3
Q27. A disc of mass m is free to rotate in a plane parallel to the xy plane with an angular
velocity  ẑ about a massless rigid rod suspended from the roof of a stationary car (as
shown in the figure below). The rod is free to orient itself along any direction.
The car accelerates in the positive x -direction with an acceleration a  0 . Which of the
following statements is true for the coordinates

of the centre of mass of the disc in the reference

frame of the car?
(a) only the x and the z coordinates change x̂
(b) only the y and the z coordinates change
axˆ
(c) only the x and the y coordinates change
(d) all the three coordinates change
 ẑ

H.No. 40‐D, Ground Floor, Jia Sarai, Near IIT, Hauz Khas, New Delhi‐110016 
Phone: 011‐26865455/+91‐9871145498
Website: www.physicsbyfiziks.com  | Email: fiziks.physics@gmail.com  

fiziks
Institute for NET/JRF, GATE, IIT‐JAM, M.Sc. Entrance, JEST, TIFR and GRE in Physics 
 
Q28. A cyclist, weighing a total of 80 kg with the bicycle, pedals at a speed of 10 m / s . She
stops pedalling at an instant which is taken to be t  0 . Due to the velocity dependent
10
frictional force, her velocity is found to vary as v  t   m.s , where t is
 t 
1  
 30 
measured in seconds. When the velocity drops to 8 m / s , she starts pedalling again to
maintain a constant speed. The energy expended by her in 1 minute at this (new) speed, is
(a) 4 kJ (b) 8 kJ (c) 16 kJ (d) 32 kJ
Q29. The number of linearly independent power series solutions, around x  0 , of the second
d 2 y dy
order linear differential equation x   xy  0 , is
dx 2 dx
(a) 0 (this equation does not have a power series solution)
(b) 1
(c) 2
(d) 3
Q30. Let x denote the position operator and p the canonically conjugate momentum operator
of a particle. The commutator
 1 2 2 1 2
 2m p   x , m p   x 
2

where  and  are constants, is zero if

(a)    (b)   2 (c)   2  (d) 2  

Q31. Two point charges 3Q and Q are placed at  0, 0, d  and  0, 0, 2d  respectively,

above an infinite grounded conducting sheet kept in the xy - plane. At a point  0, 0, z  ,

where z  d , the electrostatic potential of this charge configuration would


approximately be
1 d2 1 2d 1 3d 1 d2
(a) Q (b) Q (c) Q (d)  Q
4 0 z 3 4 0 z 2 4 0 z 2 4 0 z 3

H.No. 40‐D, Ground Floor, Jia Sarai, Near IIT, Hauz Khas, New Delhi‐110016 
Phone: 011‐26865455/+91‐9871145498
Website: www.physicsbyfiziks.com  | Email: fiziks.physics@gmail.com  

fiziks
Institute for NET/JRF, GATE, IIT‐JAM, M.Sc. Entrance, JEST, TIFR and GRE in Physics 
 
Q32. A rectangular piece of dielectric material is inserted partially into the (air) gap between
the plates of a parallel plate capacitor. The dielectric piece will
(a) remain stationary where it is placed
(b) be pushed out from the gap between the plates
(c) be drawn inside the gap between the plates and its velocity does not change sign
(d) execute an oscillatory motion in the region between the plates
Q33. An electromagnetic wave is travelling in free space (of permittivity  0 ) with electric field

ˆ cos q  x  ct 
E  kE 0

The average power (per unit area) crossing planes parallel to 4 x  3 y  0 will be
4 1 16
(a)  0 cE02 (b)  0 cE02 (c)  0 cE02 (d)  0 cE02
5 2 25
Q34. A plane electromagnetic wave from within a dielectric medium (with   4 0 and   0 )

is incident on its boundary with air, at z  0 . The magnetic field in the medium is

 
H  ˆjH 0 cos t  kx  k 3 z , where  and k are positive constants.

The angles of reflection and refraction are, respectively,


(a) 450 and 600 (b) 300 and 900 (c) 300 and 600 (d) 600 and 900
Q35. The normalized wavefunction of a particle in three dimensions is given by
1
  r , ,   e  r / 2 a where a  0 is a constant. The ratio of the most probable
8 a 3

distance from the origin to the mean distance from the origin, is

[You may use  dx x n e  x  n ! ]
0

1 1 3 2
(a) (b) (c) (d)
3 2 2 3

H.No. 40‐D, Ground Floor, Jia Sarai, Near IIT, Hauz Khas, New Delhi‐110016 
Phone: 011‐26865455/+91‐9871145498
Website: www.physicsbyfiziks.com  | Email: fiziks.physics@gmail.com  

fiziks
Institute for NET/JRF, GATE, IIT‐JAM, M.Sc. Entrance, JEST, TIFR and GRE in Physics 
 
Q36. The state vector of a one-dimensional simple harmonic oscillator of angular frequency  ,
1
at time t  0 , is given by   0    0  2  , where 0 and 2 are the normalized
2
ground state and the second excited state, respectively. The minimum time t after which
the state vector   t  is orthogonal to   0  , is

 2  4
(a) (b) (c) (d)
2   
Q37. The normalized wavefunction in the momentum space of a particle in one dimension is

  p  , where  and  are real constants. The uncertainty  x in measuring
p 2
2

its position is
    
(a)  (b)  (c) (d)
 2
 3
2  
Q38. The full scale voltage of an n -bit Digital-to-Analog Convener is V . The resolution that
can be achieved in it is
V V V V
(a) (b) (c) (d)
 2 1
n
  2 1
n
 22 n n

Q39. The spring constant k of a spring of mass ms is determined experimentally by loading


the spring with mass M and recording the time period T , for a single oscillation. If the
experiment is carried out for different masses, then the graph that correctly represents the
result is

(a) (b)
T2 T2

 0, 0  M  0, 0  M

(c) T2 (d) T2

 0, 0  M  0, 0  M
H.No. 40‐D, Ground Floor, Jia Sarai, Near IIT, Hauz Khas, New Delhi‐110016 
Phone: 011‐26865455/+91‐9871145498
Website: www.physicsbyfiziks.com  | Email: fiziks.physics@gmail.com  
10 
fiziks
Institute for NET/JRF, GATE, IIT‐JAM, M.Sc. Entrance, JEST, TIFR and GRE in Physics 
 
Q40. A Zener diode with an operating voltage of 10 V at 250 C has a positive temperature

coefficient of 0.07 % per 0 C of the operating voltage. The operating voltage of this

Zener diode at 1250 C is


(a) 12.0 V (b) 11.7 V (c) 10.7 V (d) 9.3 V
1 
Q41. The dispersion relation of a gas of spin fermions in two dimensions is E  v k ,
2

where E is the energy, k is the wave vector and v is a constant with the dimension of
velocity. If the Fermi energy at zero temperature is F , the number of particles per unit
area is
F 3F  3/F 2 2F
(a) (b) (c) (d)
 4 v   6 2 v 33   3v  
3 3
 2 v 2  2 
Q42. The relation between the internal energy U , entropy S , temperature T , pressure p ,
volume V , chemical potential  and number of particles N of a thermodynamic system
is dU  TdS  pdV   dN . That U is an exact differential implies that

p T U U
(a)   (b) p S
S V , N V S ,N T S ,N V S ,

U 1 U p T
(c) p  (d) 
T S ,N T V S , S V , N V S ,N

Q43. The number of microstates of a gas of N particles in a volume V and of internal energy
U , is given by
3N / 2
N  aU 
 U , V , N   V  Nb   
 N 
(where a and b are positive constants). Its pressure P , volume V and temperature T ,
are related by
 aN   aN 
(a)  P   V  Nb   Nk BT (b)  P  2  V  Nb   Nk BT
 V   V 

(c) PV  Nk BT (d) P V  Nb   Nk BT

H.No. 40‐D, Ground Floor, Jia Sarai, Near IIT, Hauz Khas, New Delhi‐110016 
Phone: 011‐26865455/+91‐9871145498
Website: www.physicsbyfiziks.com  | Email: fiziks.physics@gmail.com  
11 
fiziks
Institute for NET/JRF, GATE, IIT‐JAM, M.Sc. Entrance, JEST, TIFR and GRE in Physics 
 
Q44. Consider a system of identical atoms in equilibrium with blackbody radiation in a cavity
at temperature T . The equilibrium probabilities for each atom being in the ground state
0 and an excited state 1 are P0 and P1 respectively. Let n be the average number of

photons in a mode in the cavity that causes transition between the two states. Let W01

and W10 denote, respectively, the squares of the matrix elements corresponding to the

atomic transitions 0  1 and 1  0 . Which of the following equations hold in

equilibrium?
(a) P0 nW01  PW
1 1 0 (b) P0 nW01  PnW
1 1 0

(c) P0 nW01  PW
1 10  PnW
1 10 (d) P0 nW01  PW
1 10  PnW
1 10

Q45. In the circuit below the voltages VBB and VCC are kept fixed, the
VCC
voltage measured at B is a constant, but that measured at A
fluctuates between a few V to a few mV . 9V

From these measurements it may be inferred that the RC

(a) base is open internally B


(b) emitter is open internally VBB RB
(c) collector resistor is open 3V A
(d) base resistor is open

H.No. 40‐D, Ground Floor, Jia Sarai, Near IIT, Hauz Khas, New Delhi‐110016 
Phone: 011‐26865455/+91‐9871145498
Website: www.physicsbyfiziks.com  | Email: fiziks.physics@gmail.com  
12 
fiziks
Institute for NET/JRF, GATE, IIT‐JAM, M.Sc. Entrance, JEST, TIFR and GRE in Physics 
 
PART ‘C’
Q46. Consider a set of particles which interact by a pair potential V  ar 6 where r is the inter-
particle separation and a  0 is a constant. If a system of such particles has reached virial
equilibrium, the ratio of the kinetic to the total energy of the system is
1 1 3 2
(a) (b) (c) (d)
2 3 4 3
Q47. In an inertial frame S , the magnetic vector potential in a region of space is given by

A  az iˆ (where a is a constant) and the scalar potential is zero. The electric and

magnetic fields seen by an inertial observer moving with a velocity viˆ with respect to S ,
1
are, respectively [In the following   ]
v2
1 2
c

(a) 0 and  ajˆ (b) vakˆ and  aiˆ

(c) v akˆ and v ajˆ (d) v akˆ and  ajˆ


Q48. In the rest frame S1 of a point particle with electric charge q1 another point particle with

electric charge q2 moves with a speed v parallel to the x -axis at a perpendicular

distance l . The magnitude of the electromagnetic force felt by q1 due to q2 when the

1
distance between them is minimum, is [In the following   ]
v2
1 2
c
1 q1q2 1  q1q2
(a) (b)
4 0  l 2 4 0 l 2

1  q1q2  v 2  q1q2  v 2 
1
(c) 1   (d) 1  
4 0 l 2  c 2  4 0  l 2  c 2 

Q49. A circular current carrying loop of radius a carries a steady current. A constant electric
 
charge is kept at the centre of the loop. The electric and magnetic fields, E and B
respectively, at a distance d vertically above the centre of the loop satisfy
        
(a) E  B (b) E  0 (c)  E  B  0  
(d)   E  B  0  

H.No. 40‐D, Ground Floor, Jia Sarai, Near IIT, Hauz Khas, New Delhi‐110016 
Phone: 011‐26865455/+91‐9871145498
Website: www.physicsbyfiziks.com  | Email: fiziks.physics@gmail.com  
13 
fiziks
Institute for NET/JRF, GATE, IIT‐JAM, M.Sc. Entrance, JEST, TIFR and GRE in Physics 
 
0
Q50. A phase shift of 30 is observed when a beam of particles of energy 0.1 MeV is scattered

by a target. When the beam energy is changed, the observed phase shift is 600 . Assuming
that only s -wave scattering is relevant and that the cross-section does not change with
energy, the beam energy is
(a) 0.4 MeV (b) 0.3 MeV (c) 0.2 MeV (d) 0.15 MeV

Q51. Consider an element U   of the group SU  2  , where  is any one of the parameters

of the group. Under an infinitesimal change      , it changes as

U    U     U    1  X    U  gj  . To order  , the matrix X   should

always be
(a) positive definite (b) real symmetric (c) hermitian (d) anti-hermitian
dy  x 
Q52. The differential equation   x 2 , with the initial condition y  0   0 , is solved
dx
using Euler’s method. If yE  x  is the exact solution and y N  x  the numerical solution

obtained using n steps of equal length, then the relative error


 y  x   y  x 
N E
is
yE  x 

proportional to
1 1 1 1
(a) (b) (c) (d)
n2 n3 n4 n

Q53. The interval  0,1 is divided into n parts of equal length to calculate the integral
1

e
i 2 x
dx using the trapezoidal rule. The minimum value of n for which the result is
0

exact, is
(a) 2 (b) 3 (c) 4 (d) 

H.No. 40‐D, Ground Floor, Jia Sarai, Near IIT, Hauz Khas, New Delhi‐110016 
Phone: 011‐26865455/+91‐9871145498
Website: www.physicsbyfiziks.com  | Email: fiziks.physics@gmail.com  
14 
fiziks
Institute for NET/JRF, GATE, IIT‐JAM, M.Sc. Entrance, JEST, TIFR and GRE in Physics 
 
Q54. The generating function G  t , x  for the Legendre polynomials Pn  t  is

1
G t, x     x n Pn  t , for x  1
1  2 xt  x 2 n 0

x
If the function f  x  is defined by the integral equation  f  x dx  xG 1, x  , it can be
0

expressed as
 
1
(a) 
n ,m 0
x n  m Pn 1 Pm  
2
(b) 
n ,m 0
x n  m Pn 1 Pm 1

 
(c) 
n ,m 0
x n  m Pn 1 Pm 1 (d) 
n ,m 0
x n  m Pn  0  Pm 1

Q55. A particle moves in one dimension in a potential V  x    k 2 x 4   2 x 2 where k and 

are constants. Which of the following curves best describes the trajectories of this system
in phase space?
p p

(a) (b)
x x

p
p

(c) (d)

x x

H.No. 40‐D, Ground Floor, Jia Sarai, Near IIT, Hauz Khas, New Delhi‐110016 
Phone: 011‐26865455/+91‐9871145498
Website: www.physicsbyfiziks.com  | Email: fiziks.physics@gmail.com  
15 
fiziks
Institute for NET/JRF, GATE, IIT‐JAM, M.Sc. Entrance, JEST, TIFR and GRE in Physics 
 
Q56. Let  x, p  be the generalized coordinate and momentum of a Hamiltonian system. If new

variables  X , P  are defined by X  x sinh   p  and P  x cosh   p  , where  , 

and  are constants, then the conditions for it to be a canonical transformation, are
1 1 1 1
(a)      1 and      1 (b)     1 and     1
2 2 2 2
1 1 1 1
(c)      1 and      1 (d)     1 and     1
2 2 2 2
Q57. A closed system having three non-degenerate energy levels with energies E  0,  , is
at temperature T . For  2k BT , the probability of finding the system in the state with

energy E  0 , is
1 1 1 1
(a) (b) (c) cosh 2 (d)
1  2 cosh 2   2 cosh 2  2 cosh 2

Q58. Two non-degenerate energy levels with energies 0 and  are occupied by N non-
interacting particles at a temperature T . Using classical statistics, the average internal
energy of the system is
N N 3
(a) (b) (c) N  e/ kBT (d) Nk BT

1  e/ kBT  
1  e/ kBT  2

Q59. In the circuit below, D1 and D2 are two silicon diodes with the same characteristics. If

the forward voltage drop of a silicon diode is 0.7 V then the value of the current I1  I D1
I1
is
1k  I D1 ID2
10V D1 D2 Vo

(a) 18.6 mA (b) 9.3 mA (c) 13.95 mA (d) 14.65 mA

H.No. 40‐D, Ground Floor, Jia Sarai, Near IIT, Hauz Khas, New Delhi‐110016 
Phone: 011‐26865455/+91‐9871145498
Website: www.physicsbyfiziks.com  | Email: fiziks.physics@gmail.com  
16 
fiziks
Institute for NET/JRF, GATE, IIT‐JAM, M.Sc. Entrance, JEST, TIFR and GRE in Physics 
 
1 1 1 
Q60. The Hamiltonian of a two-level quantum system is H     possible initial
2 1 1
state in which the probability of the system being in that quantum state does not change
with time, is
       
 cos 4   cos 8   cos 2   cos 6 
(a)   (b)   (c)   (d)  
 sin    sin    sin    sin  
       
 4  8   2  6
Q61. Consider a one-dimensional infinite square well
0 for 0  x  a,
V  x  
 otherwise
If a perturbation
V for 0  x  a / 3,
V  x    0
0 otherwise

is applied, then the correction to the energy of the first excited state, to first order in V ,
is nearest to
(a) V0 (b) 0.16 V0 (c) 0.2 V0 (d) 0.33 V0

Q62. The energy eigenvalues En of a quantum system in the potential V  cx 6 (where c  0 is

a constant), for large values of the quantum number n , varies as


(a) n 4 / 3 (b) n3/ 2 (c) n5/ 4 (d) n6 / 5
Q63. Consider a quantum system of non-interacting bosons in contact with a particle bath. The
probability of finding no particle in a given single particle quantum state is 106 . The
average number of particles in that state is of the order of
(a) 103 (b) 106 (c) 109 (d) 1012
Q64. The sensitivity of a hot cathode pressure gauge is 10 mbar 1 . If the ratio between the
numbers of the impinging charged particles to emitted electrons is 1:10 , then the
pressure
(a) 10 mbar (b) 101 mbar (c) 102 mbar (d) 102 mbar

H.No. 40‐D, Ground Floor, Jia Sarai, Near IIT, Hauz Khas, New Delhi‐110016 
Phone: 011‐26865455/+91‐9871145498
Website: www.physicsbyfiziks.com  | Email: fiziks.physics@gmail.com  
17 
fiziks
Institute for NET/JRF, GATE, IIT‐JAM, M.Sc. Entrance, JEST, TIFR and GRE in Physics 
 
2 M  b
Q65. Two physical quantities T and M are related by the equation T  , where a
a 2
and b are constant parameters. The variation of T as a function of M was recorded in
an experiment to determine the value of a graphically. Let m be the slope of the straight
line when T 2 is plotted vs M , and  m be the uncertainty in determining it. The
uncertainty in determining a is
a m  m  b m  2   m 
(a)   (b) a   (c)   (d)  
2 m   m  2a  m  a  m 
Q66. The circuit below comprises of D -flip flops. The output is taken from Q3 , Q2,Q1 and Q0 as

shown in the figure.

LSB Q Q1 Q2 MSB Q
0 3

D Q D Q D Q D Q

CLR CLR CLR CLR

CLK

RST
the binary number given by the string Q3 , Q2,Q1Q0 changes for every clock pulse that is

applied to the CLK input. If the output is initialized at 0000 , the the corresponding
sequence of decimal numbers that repeats itself, is
(a) 3, 2,1, 0 (b) 1,3, 7,14,12,8
(c) 1,3, 7,15,12,14, 0 (d) 1,3, 7,15,14,12,8, 0

Q67. The spin-parity assignments for the ground and first excited states of the isotope 57
28 Ni , in

the single particle shell model, are


   
1 3 5 7
(a)   and   (b)   and  
2 2 2 2
   
3 5 3 5
(c)   and   (d)   and  
2 2 2 2

H.No. 40‐D, Ground Floor, Jia Sarai, Near IIT, Hauz Khas, New Delhi‐110016 
Phone: 011‐26865455/+91‐9871145498
Website: www.physicsbyfiziks.com  | Email: fiziks.physics@gmail.com  
18 
fiziks
Institute for NET/JRF, GATE, IIT‐JAM, M.Sc. Entrance, JEST, TIFR and GRE in Physics 
 
238
Q68. The first excited state of the rotational spectrum of the nucleus 92 U has an energy

45 keV above the ground state. The energy of the second excited state (in keV) is
(a) 150 (b) 120 (c) 90 (d) 60
Q69. Which of the following process is not allowed by the strong interaction but is allowed by
the weak interaction?
(a) K 0   0  K 0       (b) p  n  d  p  p

(c)    K 0  p  n (d) p     n   
Q70. A crystal of MnO has NaCl structure. It has a paramagnetic to anti- ferromagnetic
transition at 120 K . Below 120 K , the spins within a single 111 planes are parallel but

the spins in adjacent 111 planes are antiparallel. If neutron scattering is used to

determine the lattice constants, respectively, d and d  , below and above the transition
temperature of MnO then
d d
(a) d  (b) d  (c) d  2d  (d) d  2d 
2 2
Q71. A metallic nanowire of length l is approximated as a one-dimensional lattice of N
atoms with lattice spacing a . If the dispersion of electrons in the lattice is given
as E  k   E0  2t cos ka , where E0 and t are constants, then the dnsity of states inside

the nanowire depends on E as

 E  E0 
2
t2
 2t   1
3
(a) N (b)
E  E0  

E  E0 N
(c) N 3 (d)
t2  2t    E  E0 
2 2

H.No. 40‐D, Ground Floor, Jia Sarai, Near IIT, Hauz Khas, New Delhi‐110016 
Phone: 011‐26865455/+91‐9871145498
Website: www.physicsbyfiziks.com  | Email: fiziks.physics@gmail.com  
19 
fiziks
Institute for NET/JRF, GATE, IIT‐JAM, M.Sc. Entrance, JEST, TIFR and GRE in Physics 
 
Q72. Consider a two-dimensional material of length l and width w subjected to a constant
magnetic field B applied perpendicular to it. The number of change carries per unit area
B
may be expressed as n  k q , where k is appositive real number and q is the
 2  
carrier charge. Then the Hall resistivity  xy is

2 k l 2  w 2  2 k
(a) (b) (c) (d)
q2 w kq 2 l kq 2 q2
Q73. The Zeeman shift of the energy of a state with quantum numbers L, S , J and mJ is

mJ  B B  
Hz 
J  J  1
 L.J  g s S .J 
Where B is the applied magnetic field, g s is the g -factor for the spin and

B
 1.4 MHz  G 1 , where h is the Planck constant. The approximate frequency shift of
h
the S  0, L  1 and mJ  1 state, at a magnetic field of 1G , is

(a) 10 MHz (b) 1.4 MHz (c) 5 MHz (d) 2.8 MHz
Q74. The separations between the adjacent levels of a normal multiplet are found to be
22 cm 1 and 33 cm 1 . Assume that the multiplet is described well by the L  S coupling

scheme and the Lande’s interval rule, namely E  J   E  J  1  AJ , where A is a

constant. The term notaions for this multiplet is


(a) 3 P0,1,2 (b) 3 F2,3,4 (c) 3 G3,4,5 (d) 3 D1,2,3

Q75. If the fine structure splitting between the 2 2 P3/ 2 and 2 2 P3/ 2 levels in the hydrogen atom

is 0.4 cm 1 , the corresponding splitting in Li 2 will approximately be

(a) 1.2 cm 1 (b) 10.8 cm 1 (c) 32.4 cm 1 (d) 36.8 cm 1

H.No. 40‐D, Ground Floor, Jia Sarai, Near IIT, Hauz Khas, New Delhi‐110016 
Phone: 011‐26865455/+91‐9871145498
Website: www.physicsbyfiziks.com  | Email: fiziks.physics@gmail.com  
20 
fiziks
Institute for NET/JRF, GATE, IIT‐JAM, JEST, TIFR and GRE in PHYSICAL SCIENCES 
 
NET JUNE 2016
PART A
1
Q1. An infinite number of identical circular discs each of radius are tightly packed such
2
that the centers of the discs are at integer values of coordinates x and y . The ratio of the
area of the uncovered patches to the total area is
 
(a) 1  (b) (c) 1   (d) 
4 4
Q2. It takes 5 days for a steamboat to travel from A to B along a river. It takes 7 days to
return from B to A . How many days will it take for a raft to drift from A to B (all
speeds stay constant)?
(a) 13 (b) 35 (c) 6 (d) 12
Q3. “My friend Raju has more than 1000 books” said Ram “Oh no, he has less than 1000
books”, said Shyam. “Well Raju certainly has at least one book” said Geeta. If only one
of these statements is true, how many books does Raju have?
(a) 1 (b) 1000 (c) 999 (d) 1001
Q4. Of the following, which is the odd one out?
(a) Cone (b) Torus (c) Sphere (d) Ellipsoid
Q5. A student appearing for an exam is declared to have failed the exam if his/her score is
less than half the median score. This implies
1
(a) of the students appearing for the exam always fail.
4
1
(b) if a student scores less than of the maximum score, he/she always fails.
4
1
(c) if a student score more than of the maximum score, he/she always passes
2
(d) it is possible that no one fails

                                                                                
Head office  Branch office 
 
fiziks, H.No. 40 D, G.F, Jia Sarai,  Anand Institute of Mathematics, 
 
Near IIT, Hauz Khas, New Delhi‐16  28‐B/6, Jia Sarai, Near IIT 
 
Phone: 011‐26865455/+91‐9871145498 Hauz Khas, New Delhi‐16 
                                                   
                                             Website: www.physicsbyfiziks.com                                                                                          
                                                           Email: fiziks.physics@gmail.com                                                                     1 
fiziks
Institute for NET/JRF, GATE, IIT‐JAM, JEST, TIFR and GRE in PHYSICAL SCIENCES 
 
Q6. Find the next figure “D”

?
(A) (B) (C) (D)

(a) (b) (c) (d)

Q7. N is a four digit number. If the leftmost digit is removed, the resulting three digit
1
number is th of N . How many such N are possible?
9
(a) 10 (b) 9 (c) 8 (d) 7
Q8. AB and CD are two chords of a circle subtending 60o and 120o respectively at the same
point on the circumference of the circle. Then AB : CD is
(a) 3 :1 (b) 2 :1 (c) 1:1 (d) 3 : 2
Q9. % change
over  2014   2015
previous  10  10
year 10
8
5
5

year

5

10
10 10
Physics Chemistry Biology

                                                                                
Head office  Branch office 
 
fiziks, H.No. 40 D, G.F, Jia Sarai,  Anand Institute of Mathematics, 
 
Near IIT, Hauz Khas, New Delhi‐16  28‐B/6, Jia Sarai, Near IIT 
 
Phone: 011‐26865455/+91‐9871145498 Hauz Khas, New Delhi‐16 
                                                   
                                             Website: www.physicsbyfiziks.com                                                                                          
                                                           Email: fiziks.physics@gmail.com                                                                     2 
fiziks
Institute for NET/JRF, GATE, IIT‐JAM, JEST, TIFR and GRE in PHYSICAL SCIENCES 
 
Which of the following inferences can be drawn from the above graph?
(a) The total number of students qualifying in physics in 2015 and 2014 is the same
(b) The number of students qualifying in Biology in 2015 is less than in 2013
(c) The number of Chemistry students qualifying in 2015 must be more than the number
of students who qualified in Biology in 2014
(d) The number of students qualifying in physics in 2015 is equal to the number of
students in Biology that qualified in 2014

Q10. What is the minimum number of moves required to transform figure1 to figure 2 ? A
move is defined as removing a coin and placing it such that it touches two other coins in
its new position

Fig -1 Fig -2
(a) 1 (b) 2 (c) 3 (d) 4
Q11. The relationship among the numbers in each corner square is the same as that in the
other corner squares. Find the missing number.
9 10
7 13 8 12
15 14

5 
3 11 4 16
25 18
(a) 10 (b) 8 (c) 6 (d) 12
Q12. Which of the following best approximates sin 0.50 ?  
  
(a) 0.5 (b) 0.5  (c) 0.5  (d) 0.5 
90 180 360

                                                                                
Head office  Branch office 
 
fiziks, H.No. 40 D, G.F, Jia Sarai,  Anand Institute of Mathematics, 
 
Near IIT, Hauz Khas, New Delhi‐16  28‐B/6, Jia Sarai, Near IIT 
 
Phone: 011‐26865455/+91‐9871145498 Hauz Khas, New Delhi‐16 
                                                   
                                             Website: www.physicsbyfiziks.com                                                                                          
                                                           Email: fiziks.physics@gmail.com                                                                     3 
fiziks
Institute for NET/JRF, GATE, IIT‐JAM, JEST, TIFR and GRE in PHYSICAL SCIENCES 
 
Q13. What comes next in the sequence?

(a) (b) (c) (d)

Q14. Which of the following statements is logically incorrect?


(a) I always speak the truth
(b) I occasionally lie
(c) I occasionally speak the truth
(d) I always lie
Q15. How many times starting at 1.00 pm would the minute and hour hands of a clock make

an angle of 400 with each other in the next 6 hours?


(a) 6 (b) 7 (c)11 (d)12
Q16. Brother Santa and Chris walk to school from their house. The former takes 40 minutes
while the latter, 30 minutes. One day Santa started 5 minutes earlier than Chris. In how
many minutes would Chris overtake Santa?
(a) 5 (b) 15 (c) 20 (d) 25
Q17. The set of numbers  5, 6, 7, m, 6, 7,8, n  has an arithmetic mean of 6 and mode (most

frequently occurring number) of 7 . Then m  n 


(a) 18 (b) 35 (c) 28 (d)14

Q18. The diagram shows a block of marble having the shape of 50 cm


a triangular prism. What is the maximum number of slabs
of 10  10  5 cm3 size that can be cut parallel to the face on
which the block is resting? 50 cm
(a) 50 (b) 100 900
(c) 125 (d) 250 50 cm
                                                                                
Head office  Branch office 
 
fiziks, H.No. 40 D, G.F, Jia Sarai,  Anand Institute of Mathematics, 
 
Near IIT, Hauz Khas, New Delhi‐16  28‐B/6, Jia Sarai, Near IIT 
 
Phone: 011‐26865455/+91‐9871145498 Hauz Khas, New Delhi‐16 
                                                   
                                             Website: www.physicsbyfiziks.com                                                                                          
                                                           Email: fiziks.physics@gmail.com                                                                     4 
fiziks
Institute for NET/JRF, GATE, IIT‐JAM, JEST, TIFR and GRE in PHYSICAL SCIENCES 
 
Q19. A solid contains a spherical cavity. The cavity is filled with a liquid and includes a
spherical bubble of gas. The radii of cavity and gas bubble are 2 mm and 1 mm ,
respectively. What proportion of the cavity is filled with liquid?
1 3 5 7
(a) (b) (c) (d)
8 8 8 8
Q20. Fill in the blank: F 2, ___, D8, C16, B32, A64
(a) C 4 (b) E 4 (c) C 2 (d) G16

                                                                                
Head office  Branch office 
 
fiziks, H.No. 40 D, G.F, Jia Sarai,  Anand Institute of Mathematics, 
 
Near IIT, Hauz Khas, New Delhi‐16  28‐B/6, Jia Sarai, Near IIT 
 
Phone: 011‐26865455/+91‐9871145498 Hauz Khas, New Delhi‐16 
                                                   
                                             Website: www.physicsbyfiziks.com                                                                                          
                                                           Email: fiziks.physics@gmail.com                                                                     5 
fiziks
Institute for NET/JRF, GATE, IIT‐JAM, JEST, TIFR and GRE in PHYSICAL SCIENCES 
 
PART-B
1
Q21. The radius of convergence of the Taylor series expansion of the function
cosh  x 

around x  0 , is

(a)  (b)  (c) (d) 1
2
e4 z  1 1
Q22. The value of the contour integral  cosh  z   2sinh  z  dz around the unit circle C
2 i 
C

traversed in the anti-clockwise direction, is


8 1
(a) 0 (b) 2 (c) (d)  tanh  
3 2
Q23. The Gauss hypergeometric function F  a, b, c, z  , defined by the Taylor series expansion

around z  0 as F  a, b, c, z  
 a  a  1 ...  a  n  1 b  b  1 ...  b  n  1

n 0 c  c  1 ...  c  n  1 n !
zn ,

satisfies the recursion relation


d c
(a) F  a, b, c; z   F  a  1, b  1, c  1; z 
dz ab
d c
(b) F  a, b, c; z   F  a  1, b  1, c  1; z 
dz ab
d ab
(c) F  a, b, c; z   F  a  1, b  1, c  1; z 
dz c
d ab
(d) F  a, b, c; z   F  a  1, b  1, c  1; z 
dz c

                                                                                
Head office  Branch office 
 
fiziks, H.No. 40 D, G.F, Jia Sarai,  Anand Institute of Mathematics, 
 
Near IIT, Hauz Khas, New Delhi‐16  28‐B/6, Jia Sarai, Near IIT 
 
Phone: 011‐26865455/+91‐9871145498 Hauz Khas, New Delhi‐16 
                                                   
                                             Website: www.physicsbyfiziks.com                                                                                          
                                                           Email: fiziks.physics@gmail.com                                                                     6 
fiziks
Institute for NET/JRF, GATE, IIT‐JAM, JEST, TIFR and GRE in PHYSICAL SCIENCES 
 
Q24. Let X and Y be two independent random variables, each of which follow a normal
distribution with the same standard deviation  , but with means   and   ,
respectively. Then the sum X  Y follows a
(a) distribution with two peaks at   and mean 0 and standard deviation  2
(b) normal distribution with mean 0 and standard deviation 2
(c) distribution with two peaks at   and mean 0 and standard deviation 2

(d) normal distribution with mean 0 and standard deviation  2


Q25. Using dimensional analysis, Planck defined a characteristic temperature TP from powers

of the gravitational constant G , Planck’s constant h , Boltzmann constant k B and the

speed of light c in vacuum. The expression for TP is proportional to

hc 5 hc3 G hk B2
(a) (b) (c) (d)
k B2G k B2G hc 4 k B2 Gc 3

Q26. Let  x, t  and  x, t  be the coordinate systems used by the observers O and O ,

respectively. Observer O moves with a velocity v   c along their common positive x -


axis. If x  x  ct and x  x  ct are the linear combinations of the coordinates, the

Lorentz transformation relating O and O takes the form

x   x x   x 1  1 
(a) x  and x  (b) x  x and x  x
1  2 1  2 1  1 

x   x x   x 1  1 
(c) x  and x  (d) x  x and x  x
1  2 1  2 1  1 

Q27. A ball of mass m , initially at rest, is dropped from a height of 5 meters. If the coefficient
of restitution is 0.9 , the speed of the ball just before it hits the floor the second time is
approximately (take g  9.8 m / s 2 )
(a) 9.80 m / s (b) 9.10 m / s (c) 8.91 m / s (d) 7.02 m / s

                                                                                
Head office  Branch office 
 
fiziks, H.No. 40 D, G.F, Jia Sarai,  Anand Institute of Mathematics, 
 
Near IIT, Hauz Khas, New Delhi‐16  28‐B/6, Jia Sarai, Near IIT 
 
Phone: 011‐26865455/+91‐9871145498 Hauz Khas, New Delhi‐16 
                                                   
                                             Website: www.physicsbyfiziks.com                                                                                          
                                                           Email: fiziks.physics@gmail.com                                                                     7 
fiziks
Institute for NET/JRF, GATE, IIT‐JAM, JEST, TIFR and GRE in PHYSICAL SCIENCES 
 
Q28. Four equal charges of Q each are kept at the vertices of a square of side R . A particle of

mass m and charge Q is placed in the plane of the square at a short distance a   R 

from the centre. If the motion of the particle is confined to the plane, it will undergo
small oscillations with an angular frequency

Q2 Q2
(a) (b)
2 0 R 3 m  0 R 3m

2Q 2 Q2
(c) (d)
 0 R 3 m 4 0 R 3m

Q29. The Hamiltonian of a system with generalized coordinate and momentum  q. p  is

H  p 2 q 2 . A solution of the Hamiltonian equation of motion is (in the following A and


B are constants)
A 2 At A 2 At
(a) p  Be2 At , q e (b) p  Ae2 At , q e
B B
A  At A A2t
(c) p  Ae At , q (d) p  2 Ae A t , q
2
e e
B B
Q30. Two parallel plate capacitors, separated by distances x and 1.1x respectively, have a
dielectric material of dielectric constant 3.0 inserted between the plates and are
connected to a battery of voltage V . The difference in charge on the second capacitor
compared to the first is
(a) 66% (b) 20% (c) 3.3% (d) 10%
Q31. The half space region x  0 and x  0 are filled with dielectric
x0 x0
media of dielectric constants 1 and  2 respectively. There is a 2 1 
uniform electric field in each part. In the right half, the electric field 1 E1

makes an angle 1 to the interface. The corresponding angle  2 in


the left half satisfies 2
(a) 1 sin  2   2 sin 1 (b) 1 tan  2   2 tan 1 
E2
(c) 1 tan 1   2 tan  2 (d) 1 sin 1   2 sin  2

                                                                                
Head office  Branch office 
 
fiziks, H.No. 40 D, G.F, Jia Sarai,  Anand Institute of Mathematics, 
 
Near IIT, Hauz Khas, New Delhi‐16  28‐B/6, Jia Sarai, Near IIT 
 
Phone: 011‐26865455/+91‐9871145498 Hauz Khas, New Delhi‐16 
                                                   
                                             Website: www.physicsbyfiziks.com                                                                                          
                                                           Email: fiziks.physics@gmail.com                                                                     8 
fiziks
Institute for NET/JRF, GATE, IIT‐JAM, JEST, TIFR and GRE in PHYSICAL SCIENCES 
 
Q32. The x - and z -components of a static magnetic field in a region are Bx  B0 x 2  y 2  
and Bz  0 , respectively. Which of the following solutions for its y -component is
consistent with the Maxwell equations?
(a) By  B0 xy (b) By  2 B0 xy

1 
(c) By   B0 x 2  y 2   (d) By  B0  x3  xy 2 
3 
Q33. A magnetic field B is Bzˆ in the region x  0 and zero elsewhere. A rectangular loop, in
the xy -plane, of sides l (along the x -direction) and h (along the y - direction) is
inserted into the x  0 region from the x  0 region at constant velocity v  vxˆ . Which of
the following values of l and h will generate the largest EMF?
(a) l  8, h  3 (b) l  4, h  6
(c) l  6, h  4 (d) l  12, h  2
Q34. The state of a particle of mass m in a one dimensional rigid box in the interval 0 to L is

2  3  2 x  4  4 x  
given by the normalized wavefunction   x    sin    sin    . If its
L  5  L  5  L 
energy is measured the possible outcomes and the average value of energy are,
respectively
h 2 2h 2 73 h 2 h2 h2 19 h 2
(a) , and (b) , and
2mL2 mL2 50 mL2 8mL2 2mL2 40 mL2
h 2 2h 2 19 h 2 h 2 2h 2 73 h 2
(c) , and (d) , and
2mL2 mL2 10 mL2 8mL2 mL2 200 mL2
Q35. If Lˆ x , Lˆ y Lˆ z are the components of the angular momentum operator in three dimensions

the commutator  Lˆ x , Lˆ x Lˆ y Lˆ z  may be simplified to

 
(a) iLx Lˆ2z  Lˆ2y (b) iLˆ z Lˆ y Lˆ x

(c) iL  2 Lˆ  Lˆ 
x
2
z
2
y (d) 0

                                                                                
Head office  Branch office 
 
fiziks, H.No. 40 D, G.F, Jia Sarai,  Anand Institute of Mathematics, 
 
Near IIT, Hauz Khas, New Delhi‐16  28‐B/6, Jia Sarai, Near IIT 
 
Phone: 011‐26865455/+91‐9871145498 Hauz Khas, New Delhi‐16 
                                                   
                                             Website: www.physicsbyfiziks.com                                                                                          
                                                           Email: fiziks.physics@gmail.com                                                                     9 
fiziks
Institute for NET/JRF, GATE, IIT‐JAM, JEST, TIFR and GRE in PHYSICAL SCIENCES 
 
Q36. Suppose that the Coulomb potential of the hydrogen atom is changed by adding an
 Ze2 g
inverse-square term such that the total potential is V  r     2 , where g is a
r r
constant. The energy eigenvalues Enlm in the modified potential

(a) depend on n and l , but not on m


(b) depend on n but not on l and m
(c) depend on n and m , but not on l
(d) depend explicitly on all three quantum numbers n , l and m
Q37. The eigenstates corresponding to eigenvalues E1 and E2 of a time independent

Hamiltonian are 1 and 2 respectively. If at t  0 , the system is in a state

  t  0   sin  1  cos  2 the value of   t    t  at time t will be

(a) 1 (b)
 E sin1
2
  E2 cos 2  
E12  E22

(c) eiE1t /  sin   eiE2t /  cos  (d) e iE1t /  sin 2   eiE2t /  cos 2 
Q38. The specific heat per molecule of a gas of diatomic molecules at high temperatures is
(a) 8k B (b) 3.5k B (c) 4.5 k B (d) 3k B

Q39. When an ideal monatomic gas is expanded adiabatically from an initial volume V0 to

T
3V0 , its temperature changes from T0 to T . Then the ratio is
T0
2 1
1  1 3  1 3
(a) (b)   (c)   (d) 3
3 3 3
Q40. A box of volume V containing N molecules of an ideal gas, is divided by a wall with a
V
hole into two compartments. If the volume of the smaller compartment is , the
3
variance of the number of particles in it, is

N 2N N
(a) (b) (c) N (d)
3 9 3
                                                                                
Head office  Branch office 
 
fiziks, H.No. 40 D, G.F, Jia Sarai,  Anand Institute of Mathematics, 
 
Near IIT, Hauz Khas, New Delhi‐16  28‐B/6, Jia Sarai, Near IIT 
 
Phone: 011‐26865455/+91‐9871145498 Hauz Khas, New Delhi‐16 
                                                   
                                             Website: www.physicsbyfiziks.com                                                                                          
                                                           Email: fiziks.physics@gmail.com                                                                     10 
fiziks
Institute for NET/JRF, GATE, IIT‐JAM, JEST, TIFR and GRE in PHYSICAL SCIENCES 
 
Q41. A gas of non-relativistic classical particles in one dimension is subjected to a potential
 1 
V  x    x (where  is a constant). The partition function is    
 k BT 

4m 2m 8m 3m


(a) (b) (c) (d)
 3 2  2  3 2  2  3 2  2  3 2  2
Q42. The dependence of current I on the voltage V of a certain device is given by
2
 V 
I  I 0 1  
 V0 
where I 0 and V0 are constants. In an experiment the current I is measured as the voltage

V applied across the device is increased. The parameters V0 and I 0 can be graphically

determined as
(a) the slope and the y -intercept of the I  V 2 graph
(b) the negative of the ratio of the y -intercept and the slope, and the y -intercept of the

I  V 2 graph
(c) the slope and the y -intercept of the I  V graph
(d) the negative of the ratio of the y -intercept and the slope, and the y -intercept of the

I  V graph
Q43. In the schematic figure given below, assume that the propagation delay of each logic gate
is tgate . 5V

The propagation delay of the circuit will be maximum when the logic inputs A and B
make the transition
(a)  0,1  1,1 (b) 1,1   0,1

(c)  0, 0   1,1 (d)  0, 0    0,1

                                                                                
Head office  Branch office 
 
fiziks, H.No. 40 D, G.F, Jia Sarai,  Anand Institute of Mathematics, 
 
Near IIT, Hauz Khas, New Delhi‐16  28‐B/6, Jia Sarai, Near IIT 
 
Phone: 011‐26865455/+91‐9871145498 Hauz Khas, New Delhi‐16 
                                                   
                                             Website: www.physicsbyfiziks.com                                                                                          
                                                           Email: fiziks.physics@gmail.com                                                                     11 
fiziks
Institute for NET/JRF, GATE, IIT‐JAM, JEST, TIFR and GRE in PHYSICAL SCIENCES 
 
Q44. Given the input voltage Vi , which of the following waveforms correctly represents the

output voltage V0 in the circuit shown below?

0.5 10 K

5K
Vi 0 t
Vi 
V0
0.5V 
5K 10 K
0.5
0
3.0
(a) 2.5
2.0 t
V0 1.5
1.0
0.5
0.0
0

(b) 3.0
2.5
2.0
V0 1.5
1.0 t
0.5
0.0
0

3.0

(c) 2.5
2.0
V0 1.5
1.0 t
0.5
0.0
0

                                                                                
Head office  Branch office 
 
fiziks, H.No. 40 D, G.F, Jia Sarai,  Anand Institute of Mathematics, 
 
Near IIT, Hauz Khas, New Delhi‐16  28‐B/6, Jia Sarai, Near IIT 
 
Phone: 011‐26865455/+91‐9871145498 Hauz Khas, New Delhi‐16 
                                                   
                                             Website: www.physicsbyfiziks.com                                                                                          
                                                           Email: fiziks.physics@gmail.com                                                                     12 
fiziks
Institute for NET/JRF, GATE, IIT‐JAM, JEST, TIFR and GRE in PHYSICAL SCIENCES 
 
3.0
(d)
2.5
2.0 t
V0 1.5
1.0
0.5
0.0
0
Q45. The intensity distribution of a red LED on an absorbing layer of material is a Gaussian
centred at the wavelength 0  660 nm and width 20 nm . If the absorption coefficient

varies with wavelength as  0  K    0  , where  0 and K are positive constants, the

light emerging from the absorber will be


(a) blue shifted retaining the Gaussian intensity distribution
(b) blue shifted with an asymmetric intensity distribution
(c) red shifted retaining the Gaussian intensity distribution
(d) red shifted with an asymmetric intensity distribution

                                                                                
Head office  Branch office 
 
fiziks, H.No. 40 D, G.F, Jia Sarai,  Anand Institute of Mathematics, 
 
Near IIT, Hauz Khas, New Delhi‐16  28‐B/6, Jia Sarai, Near IIT 
 
Phone: 011‐26865455/+91‐9871145498 Hauz Khas, New Delhi‐16 
                                                   
                                             Website: www.physicsbyfiziks.com                                                                                          
                                                           Email: fiziks.physics@gmail.com                                                                     13 
fiziks
Institute for NET/JRF, GATE, IIT‐JAM, JEST, TIFR and GRE in PHYSICAL SCIENCES 
 
PART-C

Q46. What is the Fourier transform  dxeikx f  x  of



dn
f  x    x   n
  x
n 1 dx

where   x  is the Dirac delta-function?

1 1 1 1
(a) (b) (c) (d)
1 i k 1 i k k i k i
Q47. The integral equation
  x, t     dxdt 

eik  x  x i t t 
 
d dk
  2   3  x, t  
2
 2  k 2  m2  i 

is equivalent to the differential equation


 2 2  1
(a)  2  2  m 2  i    x, t     3  x, t 
 t x  6

 2 2 
(b)  2  2  m 2  i    x, t    2  x, t 
 t x 

 2 2 
(c)  2  2  m 2  i    x, t   3 2  x, t 
 t x 

 2 2 
(d)  2  2  m 2  i    x, t    3  x, t 
 t x 

                                                                                
Head office  Branch office 
 
fiziks, H.No. 40 D, G.F, Jia Sarai,  Anand Institute of Mathematics, 
 
Near IIT, Hauz Khas, New Delhi‐16  28‐B/6, Jia Sarai, Near IIT 
 
Phone: 011‐26865455/+91‐9871145498 Hauz Khas, New Delhi‐16 
                                                   
                                             Website: www.physicsbyfiziks.com                                                                                          
                                                           Email: fiziks.physics@gmail.com                                                                     14 
fiziks
Institute for NET/JRF, GATE, IIT‐JAM, JEST, TIFR and GRE in PHYSICAL SCIENCES 
 
Q48. A part of the group multiplication table for a six element group G  e, a, b, c, d , f  is

shown below. (In the following e is the identity element of G )


e a B c d f
e e a B c d f
a a b E d
b b e X f y z
c c
d d
f f
The entries x, y and z should be
(a) x  a, y  d and z  c (b) x  c, y  a and z  d
(c) x  c, y  d and z  a (d) x  a, y  c and z  d

Q49. In finding the roots of the polynomial f  x   3 x 3  4 x  5 using the iterative Newton-

Raphson method, the initial guess is taken to be x  2 . In the next iteration its value is
nearest to
(a) 1.671 (b) 1.656 (c) 1.559 (d) 1.551
Q50. For a particle of energy E and momentum p (in a frame F ), the rapidity y is defined

1  E  p3c 
as y  ln   . In a frame F  moving with velocity v   0, 0,  c  with respect to
2  E  p3c 

F , the rapidity y will be

1  1  
1
(a) y  y  ln 1   2
2
  (b) y  y  ln  
2  1  

 1    1  
(c) y  y  ln   (d) y  y  2 ln  
 1    1  

                                                                                
Head office  Branch office 
 
fiziks, H.No. 40 D, G.F, Jia Sarai,  Anand Institute of Mathematics, 
 
Near IIT, Hauz Khas, New Delhi‐16  28‐B/6, Jia Sarai, Near IIT 
 
Phone: 011‐26865455/+91‐9871145498 Hauz Khas, New Delhi‐16 
                                                   
                                             Website: www.physicsbyfiziks.com                                                                                          
                                                           Email: fiziks.physics@gmail.com                                                                     15 
fiziks
Institute for NET/JRF, GATE, IIT‐JAM, JEST, TIFR and GRE in PHYSICAL SCIENCES 
 
Q51. A canonical transformation  q, p    Q, P  is made through the generating function

F  q, p   q 2 p on the Hamiltonian

p2 
H  q, p    q4
2 q 2
4

where  and  are constants. The equations of motion for  Q, p  are

P 4P  Q
(a) Q  and P    Q (b) Q  and P 
  2
P 2P 2 2P
(c) Q  and P    Q (d) Q  and P    Q
 Q 
Q52. The Lagrangian of a system moving in three dimensions is

L
1 2
2
 1 1

mx1  m x22  x32  kx12  k  x2  x3 
2 2
2

The independent constants of motion is/are


(a) energy alone
(b) only energy, one component of the linear momentum and one component of the
angular momentum
(c) only energy, one component of the linear momentum
(d) only energy, one component of the angular momentum
Q53. Consider a sphere S1 of radius R which carries a uniform charge
S2
R
of density  . A smaller sphere S2 of radius a  is cut out and S1 P
2 
b 
removed from it. The centres of the two spheres are separated by r
 nRˆ
the vector b  , as shown in the figure. The electric field at a
2
point P inside S2 is

R R  R a 
(a) nˆ (b)  r  na
ˆ  (c) nˆ (d) r
3 0 3 0 a 6 0 3 0 R

                                                                                
Head office  Branch office 
 
fiziks, H.No. 40 D, G.F, Jia Sarai,  Anand Institute of Mathematics, 
 
Near IIT, Hauz Khas, New Delhi‐16  28‐B/6, Jia Sarai, Near IIT 
 
Phone: 011‐26865455/+91‐9871145498 Hauz Khas, New Delhi‐16 
                                                   
                                             Website: www.physicsbyfiziks.com                                                                                          
                                                           Email: fiziks.physics@gmail.com                                                                     16 
fiziks
Institute for NET/JRF, GATE, IIT‐JAM, JEST, TIFR and GRE in PHYSICAL SCIENCES 
 
Q54. The value of the electric and magnetic fields in a particular reference frame (in Gaussian
units) are E  3xˆ  4 yˆ and B  3 zˆ respectively. An inertial observer moving with respect

to this frame measures the magnitude of the electric field to be E   4 . The magnitude of

the magnetic field B measured by him is

(a) 5 (b) 9 (c) 0 (d) 1


Q55. A loop of radius a , carrying a current I , is placed in a uniform magnetic field B . If the
 
normal to the loop is denoted by n̂ , the force F and the torque T on the loop are
     
(a) F  0 and T   a 2 I n̂ B (b) F  0 I  B
4
        1 
(c) F  0 I  B and T  I nˆ  B (d) F  0 and T  IB
4  0 0
Q56. A waveguide has a square cross-section of side 2a . For the TM modes of wave vector k ,
the transverse electromagnetic modes are obtained in terms of a function   x, y  which

obeys the equation


 2 2   2 2

 2  2   2  k    x, y   0
 x y  c 

with the boundary condition    a, y     x,  a   0 . The frequency  of the lowest

mode is given by
 4 2   2 
(a)  2  c 2  k 2  2  (b)  2  c 2  k 2  2 
 a   a 

 2   2 
(c)  2  c 2  k 2  2  (d)  2  c 2  k 2  2 
 2a   4a 

                                                                                
Head office  Branch office 
 
fiziks, H.No. 40 D, G.F, Jia Sarai,  Anand Institute of Mathematics, 
 
Near IIT, Hauz Khas, New Delhi‐16  28‐B/6, Jia Sarai, Near IIT 
 
Phone: 011‐26865455/+91‐9871145498 Hauz Khas, New Delhi‐16 
                                                   
                                             Website: www.physicsbyfiziks.com                                                                                          
                                                           Email: fiziks.physics@gmail.com                                                                     17 
fiziks
Institute for NET/JRF, GATE, IIT‐JAM, JEST, TIFR and GRE in PHYSICAL SCIENCES 
 
1
Q57. Consider a particle of mass m in a potential V  x   m 2 x 2  g cos kx . The change in
2
1
the ground state energy, compared to the simple harmonic potential m 2 x 2 , to first
2
order in g is

 k 2h   k 2h 
(a) g exp    (b) g exp  
 2m   2m 

 2k 2 h   k 2h 
(c) g exp    (d) g exp   
 m   4m 
Q58. The energy levels for a particle of mass m in the potential V  x    x , determined in

the WKB approximation

 1
b
2m  E  V  x dx   n   
a  2

(where a, b are the turning points and n  0,1, 2... ), are


2 2
 h  1  3  3h  1  3
(a) En    n   (b) En    n  
4 m  2   4 2m  2 
2 2
 3h  1  3  h  1  3
(c) En    n   (d) En    n  
4 m  2   4 2m  2 
Q59. A particle of mass m moves in one dimension under the influence of the potential
V  x     x  , where  is a positive constant. The uncertainty in the product

 x  p  in its ground state is

 
(a) 2 (b) (c) (d) 2
2 2

                                                                                
Head office  Branch office 
 
fiziks, H.No. 40 D, G.F, Jia Sarai,  Anand Institute of Mathematics, 
 
Near IIT, Hauz Khas, New Delhi‐16  28‐B/6, Jia Sarai, Near IIT 
 
Phone: 011‐26865455/+91‐9871145498 Hauz Khas, New Delhi‐16 
                                                   
                                             Website: www.physicsbyfiziks.com                                                                                          
                                                           Email: fiziks.physics@gmail.com                                                                     18 
fiziks
Institute for NET/JRF, GATE, IIT‐JAM, JEST, TIFR and GRE in PHYSICAL SCIENCES 
 
2  4
Q60. The ground state energy of a particle of mass m in the potential V  x   x ,
6m
1
 x 2
  4
estimated using the normalized trial wavefunction   x     e 2
, is
 

  1   3
 dxx 2 e  x  dx x 4 e  x 
2 2
[use and ]
  2   4 2
3 2 13 8 2 13 2 2 13 3 2 13
(a)   (b)  (c)  (d) 
2m 3m 3m 8m
Q61. Consider a gas of Cs atoms at a number density of 1012 atoms/cc. when the typical inter-
particle distance is equal to the thermal de Broglie wavelength of the particles, the
temperature of the gas is nearest to (Take the mass of a Cs atom to be 22.7 1026 kg )

(a) 1 109 K (b) 7  105 K (c) 1 103 K (d) 2 108 K

Q62. The internal energy E T  of a system at a fixed volume is found to depend on the

temperature T as E T   aT 2  bT 4 . Then the entropy S T  , as a function of

temperature, is
1 2 1 4
(a) aT  bT (b) 2aT 2  4bT 4
2 4
4
(c) 2aT  bT 3 (d) 2aT  2bT 3
3
Q63. A radioactive element X decays toY , which in turn decays to a stable element Z . The
decay constant from X to Y is 1 , and that from Y to Z is 2 . If, to begin with, there are

1 1
only N 0 atoms of X , at short times ( t  as well as ) the number of atoms of Z
1 2
will be
1 12
(a) 12 N 0t 2 (b) N 0t
2 2  1  2 

(c)  1  2  N 0t 2 (d)  1  2  N 0t
2

                                                                                
Head office  Branch office 
 
fiziks, H.No. 40 D, G.F, Jia Sarai,  Anand Institute of Mathematics, 
 
Near IIT, Hauz Khas, New Delhi‐16  28‐B/6, Jia Sarai, Near IIT 
 
Phone: 011‐26865455/+91‐9871145498 Hauz Khas, New Delhi‐16 
                                                   
                                             Website: www.physicsbyfiziks.com                                                                                          
                                                           Email: fiziks.physics@gmail.com                                                                     19 
fiziks
Institute for NET/JRF, GATE, IIT‐JAM, JEST, TIFR and GRE in PHYSICAL SCIENCES 
 
Q64. Two completely overlapping semi-circular parallel plates comprise a capacitive
transducer. One of the plates is rotated by an angle of 10o relative to their common
centre. Ignoring edge effects, the ratio, I n : I 0 of sensitivity of the transducer in the new
configuration with respect to the original one, is
(a) 8 : 9 (b) 11:12 (c) 17 :18 (d) 35 : 36
Q65. The state diagram that detects three or more consecutive 1’s in a serial bit stream is
1
0 Reset
Reset 1 S0 /0 S1 /0
(a) S0 /0 S1 /0 (b) 0

1 0
1 0 1

S3 /1 S2 /0
S3 /1 S2 /0 1
0 0
0 0
0 0 0
Reset 1 Reset
S0 /0 S1 /0 S0 /0 S1 /0
(c) (d) 1

1 0 1
1 1 0

S3 /1 S2 /0 S3 /1 S2 /0
0 1
0 1

Q66. The decay constants f p of the heavy pseudo-scalar mesons, in the heavy quark limit, are

a
related to their masses m p by the relation f p  , where a is an empirical parameter
mp

to be determined. The values m p  6400  160 MeV and f p  180  15 MeV correspond

to uncorrelated measurements of a meson. The error on the estimate of a is


3 3 3 3
(a) 175  MeV  2 (b) 900  MeV  2 (c) 1200  MeV  2 (d) 2400  MeV  2

                                                                                
Head office  Branch office 
 
fiziks, H.No. 40 D, G.F, Jia Sarai,  Anand Institute of Mathematics, 
 
Near IIT, Hauz Khas, New Delhi‐16  28‐B/6, Jia Sarai, Near IIT 
 
Phone: 011‐26865455/+91‐9871145498 Hauz Khas, New Delhi‐16 
                                                   
                                             Website: www.physicsbyfiziks.com                                                                                          
                                                           Email: fiziks.physics@gmail.com                                                                     20 
fiziks
Institute for NET/JRF, GATE, IIT‐JAM, JEST, TIFR and GRE in PHYSICAL SCIENCES 
 
Q67. Consider electrons in graphene, which is a planar monatomic layer of carbon atoms. If
the dispersion relation of the electrons is taken to be   k   ck (where c is constant)

over the entire k -space, then the Fermi energy  F depends on the number density of

electrons  as
1 2 1
(a)  F   2 (b)  F   (c)  F   3 (d)  F   3
Q68. Suppose the frequency of phonons in a one-dimensional chain of atoms is proportional to
the wave vector. If n is the number density of atoms and c is the speed of the phonons,
then the Debye frequency is
 cn
(a) 2 cn (b) 2 cn (c) 3 cn (d)
2
Q69. The band energy of an electron in a crystal for a particular k -direction has the form
  k   A  B cos 2ka , where A and B are positive constants and 0  ka   . The
electron has a hole-like behaviour over the following range of k :
 3 
(a)  ka  (b)  ka  
4 4 2
  3
(c) 0  ka  (d)  ka 
4 2 4
Q70. The ground state electronic configuration of 22 Ti is  Ar  3d 2 4s 2 . Which state, in the

standard spectroscopic notations, is not possible in this configuration?


(a) 1 F3 (b) 1 S0 (c) 1 D2 (d) 3 P0

Q71. In a normal Zeeman Effect experiment using a magnetic field of strength 0.3 T , the
splitting between the components of a 660 nm spectral line is
(a) 12 pm (b) 10 pm (c) 8 pm (d) 6 pm
Q72. The separation between the energy levels of a two-level atom is 2 eV . Suppose that

4 1020 atoms are in the ground state and 7 1020 atoms are pumped into the excited state
just before lasing starts. How much energy will be released in a single laser pulse?
(a) 24.6 J (b) 22.4 J (c) 98 J (d) 48 J
                                                                                
Head office  Branch office 
 
fiziks, H.No. 40 D, G.F, Jia Sarai,  Anand Institute of Mathematics, 
 
Near IIT, Hauz Khas, New Delhi‐16  28‐B/6, Jia Sarai, Near IIT 
 
Phone: 011‐26865455/+91‐9871145498 Hauz Khas, New Delhi‐16 
                                                   
                                             Website: www.physicsbyfiziks.com                                                                                          
                                                           Email: fiziks.physics@gmail.com                                                                     21 
fiziks
Institute for NET/JRF, GATE, IIT‐JAM, JEST, TIFR and GRE in PHYSICAL SCIENCES 
 
Q73. In the large hadron collider  LHC  , two equal energy proton beams traverse in opposite

directions along a circular path of length 27 km . If the total centre of mass energy of a
proton-proton pair is 14 TeV , which of the following is the best approximation for the
proper time taken by a proton to traverse the entire path?
(a) 12 ns (b) 1.2  s (c) 1.2 ns (d) 0.12  s
Q74. Let ES denotes the contribution of the surface energy per nucleon in the liquid drop

model. The ratio ES  27


13 
Al : ES  64
30 
Zn is

(a) 2 : 3 (b) 4 : 3 (c) 5 : 3 (d) 3 : 2


13
Q75. According to the shell model, the nuclear magnetic moment of the 27 Al nucleus is

(Given that for a proton gl  1, g s  5.586 , and for a neutron gl  0, g s  3.826 )

(a) 1.913  N (b) 14.414  N (c) 4.793  N (d) 0

                                                                                
Head office  Branch office 
 
fiziks, H.No. 40 D, G.F, Jia Sarai,  Anand Institute of Mathematics, 
 
Near IIT, Hauz Khas, New Delhi‐16  28‐B/6, Jia Sarai, Near IIT 
 
Phone: 011‐26865455/+91‐9871145498 Hauz Khas, New Delhi‐16 
                                                   
                                             Website: www.physicsbyfiziks.com                                                                                          
                                                           Email: fiziks.physics@gmail.com                                                                     22 
fiziks
Institute for NET/JRF, GATE, IIT‐JAM, JEST, TIFR and GRE in PHYSICAL SCIENCES 
 
NET December 2016 SET-B
Part A
Q1. Find out the missing pattern.
 18 2
5 7 ? 3 6  9
2  7

2 7 14 14
(a) 7  (b)  2 (c) 7  (d) 2 7
14 14 2 

Q2. Seeds when soaked in water gain about 20% by weight and 10% by volume. By what
factor does the density increase?
(a) 1.20 (b) 1.10 (c) 1.11 (d) 1.09
Q3. Retarding frictional force, f , on a moving ball, is proportional to its velocity, V . Two

identical balls roll down identical slopes  A & B  from different heights. Compare the

retarding forces and the velocities of the balls at the bases of the slopes.

A B

(a) f A  f B ;VA  VB (b) f A  f B ; VB  VA

(c) f B  f A ; VB  VA (d) f B  f A ; VA  VB

Q4. Two cockroaches of the same species have the same thickness but different lengths and
widths. Their ability to survive in oxygen deficert environments will be compromises if
(a) their thickness increases, and the rest of the size remains the same.
(b) their thickness remains unchanged, but their length increases.
(c) their thickness remains unchanged, but their width decreases.
(d) their thickness decreases, but the rest of the size remains unchanged.

  
Head office  Branch office 
 
fiziks, H.No. 40 D, G.F, Jia Sarai,  Anand Institute of Mathematics, 
 
Near IIT, Hauz Khas, New Delhi‐16  28‐B/6, Jia Sarai, Near IIT 
 
Phone: 011‐26865455/+91‐9871145498 Hauz Khas, New Delhi‐16 
  
         Website: www.physicsbyfiziks.com  
              Email: fiziks.physics@gmail.com          1 
fiziks
Institute for NET/JRF, GATE, IIT‐JAM, JEST, TIFR and GRE in PHYSICAL SCIENCES 
 
Q5. The bar chart shows number of seats won by four political parties in a state legislative
assembly. 50
35 35

seats
20

party

Which of the following pie-charts correctly depicts this information?


(a) (b)

(c) (d)

Q6. The random errors associated with the measurement of P and Q are 10% and 2%
P
respectively, What is the percentage random error in ?
Q
(a) 12.0 (b) 9.8 (c) 8.0 (d) 10.2
Q7. In how many distinguishable ways can the letters of the word CHANCE be arranged?
(a) 120 (b) 720 (c) 360 (d) 240
Q8. Which of the following graphs correctly shows the speed and the corresponding distance
covered by an object moving along a straight line?

speed distance
(a) (b)
speed, distance

speed, distance

0, 0 time 0, 0 time

  
Head office  Branch office 
 
fiziks, H.No. 40 D, G.F, Jia Sarai,  Anand Institute of Mathematics, 
 
Near IIT, Hauz Khas, New Delhi‐16  28‐B/6, Jia Sarai, Near IIT 
 
Phone: 011‐26865455/+91‐9871145498 Hauz Khas, New Delhi‐16 
  
         Website: www.physicsbyfiziks.com  
              Email: fiziks.physics@gmail.com          2 
fiziks
Institute for NET/JRF, GATE, IIT‐JAM, JEST, TIFR and GRE in PHYSICAL SCIENCES 
 
(c) (d)
speed, distance

speed, distance
0, 0 time 0, 0 time
Q9. A normal TV screen has a width to height ratio of 4 : 3 , while a high definition TV screen
has a ratio of 16 : 9 . What is the approximate ratio of their diagonals, if the heights of the
two types of screens are the same?
(a) 5 : 9 (b) 5 :18 (c) 5 :15 (d) 5 : 6
Q10. Comparing numerical values, which of the following is different from the rest?
(a) The ratio of the circumference of a circle to its diameter.
(b) The sum of the three angles of a plane triangle expressed in radians.
22
(c) .
7
(d) The net volume of a hemisphere of unit radius, and a cone of unit radius and unit
height
1 1
Q11. A river is 4.1km wide. A bridge built across it has of its length on one-bank and of
7 8
its length on the other bank. What is the total length of the bridge?
(a) 5.1 km (b) 4.9 km (c) 5.6 km (d) 5.4 km
Q12. OA, OB , and OC are radii of the quarter circle shown in the figure. AB is also equal to
A
the radius.
B

?
O C
what is angle OCB ?
(a) 600 (b) 750 (c) 550 (d) 650

  
Head office  Branch office 
 
fiziks, H.No. 40 D, G.F, Jia Sarai,  Anand Institute of Mathematics, 
 
Near IIT, Hauz Khas, New Delhi‐16  28‐B/6, Jia Sarai, Near IIT 
 
Phone: 011‐26865455/+91‐9871145498 Hauz Khas, New Delhi‐16 
  
         Website: www.physicsbyfiziks.com  
              Email: fiziks.physics@gmail.com          3 
fiziks
Institute for NET/JRF, GATE, IIT‐JAM, JEST, TIFR and GRE in PHYSICAL SCIENCES 
 
Q13. Intravenous (IV) fluid has to be administered to a child of 12 kg with dehydration, at a
dose of 20 mg of fluid per kg of body weight, in 1 hour. What should be the drip rate (in
drops/min of IV fluid? (1mg  20 drops)
(a) 7 (b) 80 (c) 120 (d) 4
Q14. A hall with a high roof is supported by an array of identical columns such that, to a
person lying on the floor and looking at the ceiling, the columns appear parallel to each
other. Which of the following designs conforms to this?
ROOF ROOF
(a) (b)

FLOOR FLOOR

ROOF ROOF
(c) (d)

FLOOR FLOOR

Q15. The sum of digits of a two-digit number is 9 . If the fraction formed by taking 9 less than
3
the number as numerator and 9 more than the number as denominator is , what is the
4
number?
(a) 36 (b) 63 (c) 45 (d) 54

  
Head office  Branch office 
 
fiziks, H.No. 40 D, G.F, Jia Sarai,  Anand Institute of Mathematics, 
 
Near IIT, Hauz Khas, New Delhi‐16  28‐B/6, Jia Sarai, Near IIT 
 
Phone: 011‐26865455/+91‐9871145498 Hauz Khas, New Delhi‐16 
  
         Website: www.physicsbyfiziks.com  
              Email: fiziks.physics@gmail.com          4 
fiziks
Institute for NET/JRF, GATE, IIT‐JAM, JEST, TIFR and GRE in PHYSICAL SCIENCES 
 
Q16. The distance between X and Y is 1000 km . A person flies from X at 8 A.M. local time
and reaches Y at 10 A.M. local time. He flies back after a halt of 4 hours at Y and
reaches X at 4 P.M. local time on the same day. What is his average speed for the
duration he is in the air?
(a) 500 km/hour
(b) 250 km/hour
(c) 750 km/hour
(d) cannot be calculated with the given information
Q17. If a person travels x% faster than normal, he reaches y minutes earlier than normal.
What is his normal time of travel?
 100   x 
(a)   1 y minutes (b)   1 y minutes
 x   100 

 y   100 
(c)   1 x minutes (d)   1 x minutes
 100   y 
Q18. A and B walk up an escalator one step at a time, while the escalator itself moves up at a
constant speed. A walks twice as fast as B . A reaches the top in 40 steps and B in 30
steps. How many steps of the escalator can be seen when it is not moving?
(a) 30 (b) 40 (c) 50 (d) 60
Q19. Two iron spheres of radii 12 cm and 1cm are melted and fused. Two new spheres are
made without any loss of iron. Their possible radii could be
(a) 9 and 4 cm (b) 9 and10 cm
(c) 8 and 5cm (d) 2 and11cm
Q20. A man buys alcohol at Rs 75 / cL , adds water, and sells it at Rs. 75 / cL making a profit of
50 % . What is the ratio of alcohol to water?
(a) 2 :1 (b) 1: 2 (c) 3 : 2 (d) 2 : 3

  
Head office  Branch office 
 
fiziks, H.No. 40 D, G.F, Jia Sarai,  Anand Institute of Mathematics, 
 
Near IIT, Hauz Khas, New Delhi‐16  28‐B/6, Jia Sarai, Near IIT 
 
Phone: 011‐26865455/+91‐9871145498 Hauz Khas, New Delhi‐16 
  
         Website: www.physicsbyfiziks.com  
              Email: fiziks.physics@gmail.com          5 
fiziks
Institute for NET/JRF, GATE, IIT‐JAM, JEST, TIFR and GRE in PHYSICAL SCIENCES 
 
Part B
Q21. A ball of mass m is dropped from a tall building with zero initial velocity. In addition to
gravity, the ball experiences a damping force of the form  , where  is its
instantaneous velocity and  is a constant. Given the values m  10 kg ,   10 kg / s , ,

and g  10 m / s 2 the distance travelled (in metres) in time t in seconds, is

(a) 10  t  1  e  t  (b) 10  t  1  e t 

(c) 5t 2  1  et  (d) 5t 2

1 3 2
 
Q22. The matrix M   3 1 0  satisfies the equation
0 0 1
 
(a) M 3  M 2  10M  12 I  0 (b) M 3  M 2  12 M  10 I  0
(c) M 3  M 2  10M  10 I  0 (d) M 3  M 2  10 M  10 I  0
Q23. The Laplace transform of
t
 , 0t T
f t   T
 1 t T

is
 1  e  sT  1  e 
 sT
1  e   sT
1  e 
sT

(a) (b) (c) (d)


s 2T s 2T s 2T s 2T
Q24. A relativistic particle moves with a constant velocity v with respect to the laboratory
frame. In time  , measured in the rest frame of the particle, the distance that it travels in
the laboratory frame is

c v2 v
(a) v (b) (c) v 1  (d)
v2 c2 v2
1 1
c2 c2

  
Head office  Branch office 
 
fiziks, H.No. 40 D, G.F, Jia Sarai,  Anand Institute of Mathematics, 
 
Near IIT, Hauz Khas, New Delhi‐16  28‐B/6, Jia Sarai, Near IIT 
 
Phone: 011‐26865455/+91‐9871145498 Hauz Khas, New Delhi‐16 
  
         Website: www.physicsbyfiziks.com  
              Email: fiziks.physics@gmail.com          6 
fiziks
Institute for NET/JRF, GATE, IIT‐JAM, JEST, TIFR and GRE in PHYSICAL SCIENCES 
 
Q25. A particle in two dimensions is in a potential V  x, y   x  2 y . Which of the following

(apart from the total energy of the particle) is also a constant of motion?
(a) p y  2 px (b) px  2 p y

(c) px  2 p y (d) p y  2 px

Q26. The dynamics of a particle governed by the Lagrangian


1 2 1 2
L mx  kx  kxxt
 describes
2 2
(a) an undamped simple harmonic oscillator
(b) a damped harmonic oscillator with a time varying damping factor
(c) an undamped harmonic oscillator with a time dependent frequency
(d) a free particle
Q27. The parabolic coordinates  ,  are related to the Cartesian coordinates  x, y  by

x   and y  
1 2
2
   2  . The Lagrangian of a two-dimensional simple harmonic

oscillator of mass m and angular frequency  is

m    2   2  2   2  
1  2
(a)
2
 
(b)
1
2 
 
m  2   2    2   2   2  2   2  
1
4 
 
m  2   2   2   2   2 
1 1
(c)
2  2 
 1 
m  2   2   2   2   2 
1
(d)
2  4 
Q28. Consider two radioactive atoms, each of which has a decay rate of 1 per year. The
probability that at least one of them decays in the first two years is

(d) 1  e 2 
1 3 2
(a) (b) (c) 1  e 4
4 4

  
Head office  Branch office 
 
fiziks, H.No. 40 D, G.F, Jia Sarai,  Anand Institute of Mathematics, 
 
Near IIT, Hauz Khas, New Delhi‐16  28‐B/6, Jia Sarai, Near IIT 
 
Phone: 011‐26865455/+91‐9871145498 Hauz Khas, New Delhi‐16 
  
         Website: www.physicsbyfiziks.com  
              Email: fiziks.physics@gmail.com          7 
fiziks
Institute for NET/JRF, GATE, IIT‐JAM, JEST, TIFR and GRE in PHYSICAL SCIENCES 
 
 1
Q29. The Fourier transform  dxf  x eikx of the function f  x   2 is
 x 2
 2k    2k
(a) 2 e (b) 2 e  2k
(c) e 2k
(d) e
2 2
Q30. A screen has two slits, each of width w with their centres at a
distance 2w apart. It is illuminated by a monochromatic plane w
wave travelling along the x -axis. w 
x
The intensity of the interference pattern, measured on a distant
w
screen, at an angle
n
 to the x -axis is
w
(a) zero for n  1, 2,3...
(b) maximum for n  1, 2,3...
1 3 5
(c) maximum for n  , , ...
2 2 2
(d) zero for n  0 only
Q31. The electric field of an electromagnetic wave is

E  z , t   E0 cos  kz  t  iˆ  2 E0 sin  kz  t  ˆj

where  and k are positive constants. This represents


(a) a linearly polarised wave travelling in the positive z -direction
(b) a circularly polarised wave travelling in the negative z -direction
(c) an elliptically polarised wave travelling in the negative z -direction
(d) an unpolarised wave travelling in the positive z -direction
Q32. Consider the two lowest normalized energy eigenfunctions  0  x  and  1  x  of a one

d 0
dimensional system. They satisfy  0  x    0*  x  and  1  x    , where  is a real
dx
constant. The expectation value of the momentum operator in the state  1 is

  2
(a)  (b) 0 (c) (d)
 2
 2
2
  
Head office  Branch office 
 
fiziks, H.No. 40 D, G.F, Jia Sarai,  Anand Institute of Mathematics, 
 
Near IIT, Hauz Khas, New Delhi‐16  28‐B/6, Jia Sarai, Near IIT 
 
Phone: 011‐26865455/+91‐9871145498 Hauz Khas, New Delhi‐16 
  
         Website: www.physicsbyfiziks.com  
              Email: fiziks.physics@gmail.com          8 
fiziks
Institute for NET/JRF, GATE, IIT‐JAM, JEST, TIFR and GRE in PHYSICAL SCIENCES 
 
d
Q33. Consider the operator a  x  acting on smooth functions of x . The commutator
dx
 , cos x  is
(a)  sin x (b) cos x (c)  cos x (d) 0
1 1
Q34. Let a   x  ip  and a†   x  ip  be the lowering and raising operators of a
2 2
simple harmonic oscillator in units where the mass, angular frequency and  have been
set to unity. If 0 is the ground state of the oscillator and  is a complex constant, the

expectation value of  x  in the state   exp   a †   *a  0 , is

(a)  (b)  
2 1
(c)
1
   *  (d)
1
   * 

2
2i 2
   
Q35. Consider the operator   p  qA , where p is the momentum operator,

A   Ax , Ay , Az  is the vector potential and q denotes the electric charge. If

B   Bx , By , Bz  denotes the magnetic field, the z -component of the vector operator
 
   is
(a) iqBz  q  Ax p y  Ay px  (b) iqBz  q  Ax p y  Ay px 

(c) iqBz (d) iqBz

Q36. A conducting circular disc of radius r and resistivity  rotates with an angular velocity
 in a magnetic field B perpendicular to it. A voltmeter is connected as shown in the
figure below. Assuming its internal resistance to be infinite, the reading on the voltmeter
(a) depends on  , B, r and 
B
(b) depends on  , B and r but not on 
(c) is zero because the flux through the loop is not
changing
r V
(d) is zero because a current the flows in the direction
of B
  
Head office  Branch office 
 
fiziks, H.No. 40 D, G.F, Jia Sarai,  Anand Institute of Mathematics, 
 
Near IIT, Hauz Khas, New Delhi‐16  28‐B/6, Jia Sarai, Near IIT 
 
Phone: 011‐26865455/+91‐9871145498 Hauz Khas, New Delhi‐16 
  
         Website: www.physicsbyfiziks.com  
              Email: fiziks.physics@gmail.com          9 
fiziks
Institute for NET/JRF, GATE, IIT‐JAM, JEST, TIFR and GRE in PHYSICAL SCIENCES 
 
Q37. The charge per unit length of a circular wire of radius a in the xy -plane, with its centre at
the origin, is   0 cos  , where 0 is a constant and the angle  is measured from the

positive x -axis. The electric field at the centre of the circle is


 0  0
(a) E   iˆ (b) E  iˆ
4 0  4 0 
 0  0
(c) E   ˆj (d) E  kˆ
4 0  4 0 
Q38. The partition function of a two-level system governed by the Hamiltonian
  
H 
   
is


(a) 2sinh   2   2
(b) 2 cosh       2 2

(c) cosh        sinh   


 2   2 
1 2 2

2 

(d)
1

2
 
cosh   2   2  sinh   2   2 
 
Q39. A silica particle of radius 0.1  m is put in a container of water at T  300 K . The

densities of silica and water are 2000 kg / m3 and 1000 kg / m3 , respectively. Due to
thermal fluctuations, the particle is not always at the bottom of the container. The average
height of the particle above the base of the container is approximately
(a) 103 m (b) 3 104 m (c) 104 m (d) 5 105 m

  
Head office  Branch office 
 
fiziks, H.No. 40 D, G.F, Jia Sarai,  Anand Institute of Mathematics, 
 
Near IIT, Hauz Khas, New Delhi‐16  28‐B/6, Jia Sarai, Near IIT 
 
Phone: 011‐26865455/+91‐9871145498 Hauz Khas, New Delhi‐16 
  
         Website: www.physicsbyfiziks.com  
              Email: fiziks.physics@gmail.com          10 
fiziks
Institute for NET/JRF, GATE, IIT‐JAM, JEST, TIFR and GRE in PHYSICAL SCIENCES 
 
Q40. Which of the following circuits implements the Boolean function
F  A, B, C    1, 2, 4, 6  ?

C I0 C I0
(a) I1 4  1 (b) I1 4  1

I MUX
2
F I MUX
2
F

I 3 S1 S0 I 3 S1 S0

A B A B

C I0 0 I0
(c) I1 4  1 (d) I1 4  1
1
I MUX
2
F I MUX
2
F

I 3 S1 S0 C I 3 S1 S0

A B A B
Q41. A pair of parallel glass plates separated by a distance d is illuminated by white light as
shown in the figure below. Also shown is the graph of the intensity of the reflected light
I as a function of the wavelength  recorded by a spectrometer.

spectrometer

partially incident
reflecting mirror white light

air gap
glass plates
d

  
Head office  Branch office 
 
fiziks, H.No. 40 D, G.F, Jia Sarai,  Anand Institute of Mathematics, 
 
Near IIT, Hauz Khas, New Delhi‐16  28‐B/6, Jia Sarai, Near IIT 
 
Phone: 011‐26865455/+91‐9871145498 Hauz Khas, New Delhi‐16 
  
         Website: www.physicsbyfiziks.com  
              Email: fiziks.physics@gmail.com          11 
fiziks
Institute for NET/JRF, GATE, IIT‐JAM, JEST, TIFR and GRE in PHYSICAL SCIENCES 
 
1
0.8

0.6

Intensity 0.4

0.2

0
490 500 510 520 530

Assuming that the interference takes place only between light reflected by the bottom
surface of the top plate and the top surface of bottom plate, the distance d is closest to
(a) 12  m (b) 24  m (c) 60  m (d) 120  m

  aV  
Q42. The I  V characteristics of a device can be expressed as I  I s exp    1 , where T
  T  
is the temperature and a and I s are constants independent of T and V . Which one of

the following plots is correct for a fixed applied voltage V ?

3 2
1
2
(a) (b)
log 1

log 1

1 1
2
0 3 3
1 2 3 0 1 2
aV / T aV / T

2 4
3
1
2
log 1

log 1

(c) 0 (d) 1
1 0
1
2 2
3 3 3 3
0 1 2 0 1 2
aV / T aV / T
  
Head office  Branch office 
 
fiziks, H.No. 40 D, G.F, Jia Sarai,  Anand Institute of Mathematics, 
 
Near IIT, Hauz Khas, New Delhi‐16  28‐B/6, Jia Sarai, Near IIT 
 
Phone: 011‐26865455/+91‐9871145498 Hauz Khas, New Delhi‐16 
  
         Website: www.physicsbyfiziks.com  
              Email: fiziks.physics@gmail.com          12 
fiziks
Institute for NET/JRF, GATE, IIT‐JAM, JEST, TIFR and GRE in PHYSICAL SCIENCES 
 
Q43. The active medium in a blue LED (light emitting diode) is a Gax In1 x N alloy. The band

gaps of GaN and InN are 3.5 eV and 1.5 eV respectively. If the band gap of Gax In1 x N

varies approximately linearly with x , the value of x required for the emission of blue
light of wavelength 400 nm is (take hc  1200 eV -nm )
(a) 0.95 (b) 0.75 (c) 0.50 (d) 0.33
Q44. Consider a gas of N classical particles in a two-dimensional square box of side L . If the
total energy of the gas is E , the entropy (apart from an additive constant) is
 L2 E   LE 
(a) Nk B ln   (b) Nk B ln  
 N   N 

L E  E
(c) 2 Nk B ln   (d) L2 k B ln  
 N  N

Q45. Consider a continuous time random walk. If a step has taken place at time t  0 , the
probability that the next step takes place between t and t  dt is given by bt dt , where b
is a constant. What is the average time between successive steps?

2  1  
(a) (b) (c) (d)
b b 2 b 2b

  
Head office  Branch office 
 
fiziks, H.No. 40 D, G.F, Jia Sarai,  Anand Institute of Mathematics, 
 
Near IIT, Hauz Khas, New Delhi‐16  28‐B/6, Jia Sarai, Near IIT 
 
Phone: 011‐26865455/+91‐9871145498 Hauz Khas, New Delhi‐16 
  
         Website: www.physicsbyfiziks.com  
              Email: fiziks.physics@gmail.com          13 
fiziks
Institute for NET/JRF, GATE, IIT‐JAM, JEST, TIFR and GRE in PHYSICAL SCIENCES 
 
PART C
Q46. Given the values sin 450  0.7071, sin 500  0.7660, sin 550  0.8192 and

sin 600  0.8660 , the approximate value of sin 520 , computed by Newton’s forward
difference method, is
(a) 0.804 (b) 0.776 (c) 0.788 (d) 0.798
f 2 f
Q47. Let f  x, t  be a solution of the heat equation  D 2 in one dimension. The initial
t x

condition at t  0 is f  x, 0   e x for   x   . Then for all t  0, f  x, t  is given by


2

 
 dx e  x 
2
[Useful integral: ]
 
2 2
x x
1  1 
(a) e 1 Dt (b) e 1 2 Dt
1  Dt 1  2 Dt
2
x x2
1  
(c) e 1 4 Dt (d) e 1 Dt

1  4 Dt
Q48. After a perfectly elastic collision of two identical balls, one of which was initially at rest,
the velocities of both the balls are non zero. The angle  between the final, velocities (in
the lab frame) is

(a)   (b)   
2
 
(c) 0    (d)  
2 2
k
Q49. Consider circular orbits in a central force potential V  r    , where k  0 and
rn
0  n  2 . If the time period of a circular orbit of radius R is T1 and that of radius 2 R is

T2
T2 , then
T1
n 2 n
n 1
(a) 2 2 (b) 2 3 (c) 2 2 (d) 2n

  
Head office  Branch office 
 
fiziks, H.No. 40 D, G.F, Jia Sarai,  Anand Institute of Mathematics, 
 
Near IIT, Hauz Khas, New Delhi‐16  28‐B/6, Jia Sarai, Near IIT 
 
Phone: 011‐26865455/+91‐9871145498 Hauz Khas, New Delhi‐16 
  
         Website: www.physicsbyfiziks.com  
              Email: fiziks.physics@gmail.com          14 
fiziks
Institute for NET/JRF, GATE, IIT‐JAM, JEST, TIFR and GRE in PHYSICAL SCIENCES 
 
c
Q50. Consider a radioactive nucleus that is travelling at a speed with respect to the lab
2
frame. It emits  -rays of frequency v0 in its rest frame. There is a stationary detector,

(which is not on the path of the nucleus) in the lab. If a  -ray photon is emitted when the
nucleus is closest to the detector, its observed frequency at the detector is

3 1 1 2
(a) v0 (b) v0 (c) v0 (d) v0
2 3 2 3
Q51. Suppose that free charges are present in a material of dielectric constant  10 and
resistively   1011   m . Using Ohm’s law and the equation of continuity for charge, the
1
time required for the charge density inside the material to decay by is closest to
e
(a) 106 S (b) 106 S (c) 1012 S (d) 10 S
Q52. A particle with charge q moves with a uniform angular velocity  in a circular orbit of
radius a in the xy - plane, around a fixed charge  q , which is at the centre of the orbit at

 0, 0, 0  . Let the intensity of radiation at the point  0, 0, R  be I1 and at  2 R, 0, 0  be ‘ I 2

I2
The ratio for R  a , is
I1
1 1
(a) 4 (b) (c) (d) 8
4 8
Q53. A parallel plate capacitor is formed by two circular conducting plates of radius a
separated by a distance d , where d  a . It is being slowly charged by a current that is
nearly constant. At an instant when the current is I , the magnetic induction between the
a
plates at a distance from the centre of the plate, is
2
0 I 0 I 0 I 0 I
(a) (b) (c) (d)
a 2 a a 4 a

  
Head office  Branch office 
 
fiziks, H.No. 40 D, G.F, Jia Sarai,  Anand Institute of Mathematics, 
 
Near IIT, Hauz Khas, New Delhi‐16  28‐B/6, Jia Sarai, Near IIT 
 
Phone: 011‐26865455/+91‐9871145498 Hauz Khas, New Delhi‐16 
  
         Website: www.physicsbyfiziks.com  
              Email: fiziks.physics@gmail.com          15 
fiziks
Institute for NET/JRF, GATE, IIT‐JAM, JEST, TIFR and GRE in PHYSICAL SCIENCES 
 
Q54. Two uniformly charged insulating solid spheres A and B , both of radius a , carry total
charges Q and Q , respectively. The spheres are placed touching each other as shown
in the figure.    
 A   B 
If the potential at the centre of the sphere A is VA and that at the
   
centre of B is VB then the difference VA  VB is

Q Q Q Q
(a) (b) (c) (d)
4 0 a 2 0 a 2 0 a 4 0 a
3
Q55. A stable asymptotic solution of the equation xn 1  1  is x  2 . If we take
1  xn
n 1
xn  2 n and xn 1  2 n 1 , where n and n 1 are both small, the ratio is
n
approximately
1 1 1 2
(a)  (b)  (c)  (d) 
2 4 3 3
Q56. The 2  2 identity matrix I and the Pauli matrices  x ,  y ,  z do not form a group under
matrix multiplication. The minimum number of 2  2 matrices, which includes these four
matrices, and form a group (under matrix multiplication) is
(a) 20 (b) 8 (c) 12 (d) 16
Q57. The dynamics of a free relativistic particle of mass m is governed by the Dirac
   
Hamiltonian H  c . p   mc 2 , where p is the momentum operator and    x ,  y ,  z 

and  are four 4  4 Dirac matrices. The acceleration operator can be expressed as
2ic   
(a)  cp   H  (b) 2ic 2

ic  2ic  
(c) H (d)   cp   H 
 

  
Head office  Branch office 
 
fiziks, H.No. 40 D, G.F, Jia Sarai,  Anand Institute of Mathematics, 
 
Near IIT, Hauz Khas, New Delhi‐16  28‐B/6, Jia Sarai, Near IIT 
 
Phone: 011‐26865455/+91‐9871145498 Hauz Khas, New Delhi‐16 
  
         Website: www.physicsbyfiziks.com  
              Email: fiziks.physics@gmail.com          16 
fiziks
Institute for NET/JRF, GATE, IIT‐JAM, JEST, TIFR and GRE in PHYSICAL SCIENCES 
 
Q58. A particle of charge q in one dimension is in a simple harmonic potential with angular
2
t
 
frequency  . It is subjected to a time- dependent electric field E  t   Ae  
, where A

and  are positive constants and   1 . If in the distant past t   the particle was
in its ground state, the probability that it will be in the first excited state as t   is
proportional to
1 1
  2  2 1
2
(a) e (b) e 2 (c) 0 (d)
 
2

Q59. Consider a random walk on an infinite two-dimensional


triangular lattice, a part of which is shown in the figure below.
If the probabilities of moving to any of the nearest neighbour
sites are equal, what is the probability that the walker returns to
the starting position at the end of exactly three steps?
1 1 1 1
(a) (b) (c) (d)
36 216 18 12
Q60. An atom has a non-degenerate ground-state and a doubly-degenerate excited state. The
energy difference between the two states is  . The specific heat at very low temperatures
   1 is given by
(a) k B    (b) k B e  

(c) 2k B    e 
2
(d) k B

Q61. The electrons in graphene can be thought of as a two-dimensional gas with a linear
 
energy-momentum relation E  p v , where p   px , p y  and v is a constant. If  is the

number of electrons per unit area, the energy per unit area is proportional to
(a)  3/ 2 (b)  (c)  1/ 3 (d)  2

  
Head office  Branch office 
 
fiziks, H.No. 40 D, G.F, Jia Sarai,  Anand Institute of Mathematics, 
 
Near IIT, Hauz Khas, New Delhi‐16  28‐B/6, Jia Sarai, Near IIT 
 
Phone: 011‐26865455/+91‐9871145498 Hauz Khas, New Delhi‐16 
  
         Website: www.physicsbyfiziks.com  
              Email: fiziks.physics@gmail.com          17 
fiziks
Institute for NET/JRF, GATE, IIT‐JAM, JEST, TIFR and GRE in PHYSICAL SCIENCES 
 
Q62. In the circuit below, the input voltage Vi is 2V ,Vcc  16 V , R2  2k  and RL  10 k 
VCC


Vi  R1 RL
R2

The value of R1 required to deliver 10 mW of power across RL is

(a) 12k  (b) 4k  (c) 8k  (d) 14 k 

Q63. Two sinusoidal signals are sent to an analog multiplier of scale factor 1V 1 followed by a
low pass filter (LPF).

V1  5cos 100t 

LPF
Multiplier fC  5Hz
Vout

V2  20 cos 100t   / 3

If the roll-off frequency of the LPF is f c  5 Hz , the output voltage Vout is

(a) 5V (b) 25V (c) 100 V (d) 50V


Q64. The resistance of a sample is measured as a function of temperature, and the data are
shown below.
T  0C 2 4 6 8
R  90 105 110 115

The slope of R vs T graph, using a linear least-squares fit to the data, will be
6 4 2 8
(a) 0
(b) 0
(c) 0
(d) 0
C C C C
  
Head office  Branch office 
 
fiziks, H.No. 40 D, G.F, Jia Sarai,  Anand Institute of Mathematics, 
 
Near IIT, Hauz Khas, New Delhi‐16  28‐B/6, Jia Sarai, Near IIT 
 
Phone: 011‐26865455/+91‐9871145498 Hauz Khas, New Delhi‐16 
  
         Website: www.physicsbyfiziks.com  
              Email: fiziks.physics@gmail.com          18 
fiziks
Institute for NET/JRF, GATE, IIT‐JAM, JEST, TIFR and GRE in PHYSICAL SCIENCES 
 
Q65. A particle is scattered by a central potential V  r   V0 re   r , where V0 and  are positive
 
constants. If the momentum transfer q is such that q  q   , the scattering cross-

section in the Born approximation, as q   , depends on q as

dn
 x e dx  n 
n ax
[You may use e ax dx ]
da
(a) q 8 (b) q 2 (c) q 2 (d) q 6

Q66. A particle in one dimension is in a potential V  x   A  x  a  . Its wavefunction   x  is

d
continuous everywhere. The discontinuity in at x  a is
dx

(b) A   a     a  
2m
(a) A  a 
2
2
(c) A (d) 0
2m
Q67. Consider a hexagonal lattice with basis vectors as shown in the figure below.
y

x

a2 a
1

a
If the lattice spacing is a  1 , the reciprocal lattice vectors are

 4   2 2   4   2 2 
(a)  ,0,   ,  (b)  ,0,  , 
 3   3 3  3   3 3

 4   2   2 2   2 
(c)  0,  ,  ,  (d)  ,  ,  2 , 
 3   3  3 3  3

  
Head office  Branch office 
 
fiziks, H.No. 40 D, G.F, Jia Sarai,  Anand Institute of Mathematics, 
 
Near IIT, Hauz Khas, New Delhi‐16  28‐B/6, Jia Sarai, Near IIT 
 
Phone: 011‐26865455/+91‐9871145498 Hauz Khas, New Delhi‐16 
  
         Website: www.physicsbyfiziks.com  
              Email: fiziks.physics@gmail.com          19 
fiziks
Institute for NET/JRF, GATE, IIT‐JAM, JEST, TIFR and GRE in PHYSICAL SCIENCES 
 
Q68. In the L  S coupling scheme, the terms arising from two non-equivalent p -electrons are

(a) 3S , 1P, 3P, 1D, 3D (b) 1S , 3S , 1P, 1D

(c) 1S , 3S , 3P, 3D (d) 1S , 3S , 1P, 3P, 1D, 3D


Q69. The total spin of a hydrogen atom is due to the contribution of the spins of the electron
and the proton. In the high temperature limit, the ratio of the number of atoms in the spin-
1 state to the number in the spin- 0 state is
1 1
(a) 2 (b) 3 (c) (d)
2 3
Q70. A two level system in a thermal (black body) environment can decay from the excited
state by both spontaneous and thermally stimulated emission. At room temperature
 300 K  , the frequency below which thermal emission dominates over spontaneous

emission is nearest to
(a) 1013 Hz (b) 108 Hz (c) 105 Hz (d) 1011 Hz
Q71. What should be the minimum energy of a photon for it to split an  -particle at rest into a
tritium and a proton?
4
(The masses of 2 He, 13 H and 1
1 H are 4.0026 amu,3.0161 amu and 1.0073 amu

respectively, and 1 amu  938 MeV )


(a) 32.2 MeV (b) 3MeV (c) 19.3 MeV (d) 931.5 MeV
Q72. Which of the following reaction(s) is/are allowed by the conservation laws?
(i)    n   0  K 
(ii)    p   0  K 0
(a) both (i) and (ii) (b) only (i)
(c) only (ii) (d) neither (i) nor (ii)

  
Head office  Branch office 
 
fiziks, H.No. 40 D, G.F, Jia Sarai,  Anand Institute of Mathematics, 
 
Near IIT, Hauz Khas, New Delhi‐16  28‐B/6, Jia Sarai, Near IIT 
 
Phone: 011‐26865455/+91‐9871145498 Hauz Khas, New Delhi‐16 
  
         Website: www.physicsbyfiziks.com  
              Email: fiziks.physics@gmail.com          20 
fiziks
Institute for NET/JRF, GATE, IIT‐JAM, JEST, TIFR and GRE in PHYSICAL SCIENCES 
 
Q73. A particle, which is a composite state of three quarks u , d and s , has electric charge,
spin and strangeness respectively, equal to
1 1 1
(a) 1, , 1 (b) 0, 0, 1 (c) 0, , 1 (d) 1,   1
2 2 2
Q74. Consider a one-dimensional chain of atoms with lattice constant a . The energy of an
electron with wave-vector k is   k      cos  ka  , where  and  are constants. If

an electric field E is applied in the positive x -direction, the time dependent velocity of
an electron is
(In the following B is the constant)
 eE 
(a) Proportional to cos  B  at  (b) proportional to E
  

 eE 
(c) independent of E (d) proportional to sin  B  at 
  
Q75. A thin rectangular conducting plate of length a and width b is placed in the xy -plane in
two different orientations as shown in the figures below. In both cases a magnetic field B
is applied in the z -direction and a current flows in the x direction due to the applied
voltage V . b
y B

B
x a
V1 a
V1 b

   
V V
If the Hall voltage across the y -direction in the two cases satisfy V2  2V1 the ratio a : b
must be
(a) 1: 2 (b) 1: 2 (c) 2 :1 (d) 2 :1

  
Head office  Branch office 
 
fiziks, H.No. 40 D, G.F, Jia Sarai,  Anand Institute of Mathematics, 
 
Near IIT, Hauz Khas, New Delhi‐16  28‐B/6, Jia Sarai, Near IIT 
 
Phone: 011‐26865455/+91‐9871145498 Hauz Khas, New Delhi‐16 
  
         Website: www.physicsbyfiziks.com  
              Email: fiziks.physics@gmail.com          21 
fiziks
Institute for NET/JRF, GATE, IIT‐JAM, JEST, TIFR and GRE in PHYSICAL SCIENCES 
 
NET December 2015
PART ‘A’
Q1. In each of the following groups of words is a hidden number, based on which you should arrange
them in descending order. Pick the correct answer:
E. Papers I Xeroxed
F. Wi-Fi veteran
G. Yourself ourselves
H. Breaks even
(a) H, F, G, H (b) E, G, F, H (c) H, F, G, E (d) H, E, F, G
Q2. The number of squares in the figure is
(a) 30
(b) 29
(c) 25
(d) 20
Q3. A shopkeeper purchases a product for Rs. 100 and sells it making a profit of 10% . The customer
resells it to the same shopkeeper incurring a loss of 10% . In these dealings the shopkeeper
makes
(a) no profit, no loss (b) Rs. 11
(c) Re. 1 (d) Rs. 20
Q4. Five congruent rectangles are drawn inside a big rectangle of perimeter 165 as shown. What is
the perimeter of one of the five rectangles?
(a) 37
(b) 75
(c) 15
(d) 165

                                                                                
Head office  Branch office 
 
fiziks, H.No. 23, G.F, Jia Sarai,  Anand Institute of Mathematics, 
 
Near IIT, Hauz Khas, New Delhi‐16  28‐B/6, Jia Sarai, Near IIT 
 
Phone: 011‐26865455/+91‐9871145498 Hauz Khas, New Delhi‐16 
                                                   
                                             Website: www.physicsbyfiziks.com                                                                                          
                                                           Email: fiziks.physics@gmail.com                                                                   1 
fiziks
Institute for NET/JRF, GATE, IIT‐JAM, JEST, TIFR and GRE in PHYSICAL SCIENCES 
 
Q5. A person walks downhill at 10 km / h , uphill at 6 km / h and on the plane at 7.5 km / h . If the
person takes 3 hours to go from a place A to another place B , and 1 hour on the way back, the
distance between A and B is
(a) 15 km
(b) 23.5 km
(c) 16 km
(d) Given data is insufficient to calculate distance.
Q6. A vessel is partially filled with water. More water is added to it at a rate directly proportional to
 dV 
time i.e.,  t  . Which of the following graphs depicts correctly the variation of total volume
 dt 
V of water with time t ?
(a) V (b) V

0 t 0 t

(c) V (d) V

0 t 0 t

Q7. At one instant, the hour hand and the minute hand of a clock are one over the other in between
the markings for 5 and 6 on the dial. At this instant, the tip of the minute hand
(a) is closer to the marking for 6
(b) is equidistant from the markings for 5 and 6
(c) is closer to marking for 5
(d) is equidistant from the markings for 11 and 12

                                                                                
Head office  Branch office 
 
fiziks, H.No. 23, G.F, Jia Sarai,  Anand Institute of Mathematics, 
 
Near IIT, Hauz Khas, New Delhi‐16  28‐B/6, Jia Sarai, Near IIT 
 
Phone: 011‐26865455/+91‐9871145498 Hauz Khas, New Delhi‐16 
                                                   
                                             Website: www.physicsbyfiziks.com                                                                                          
                                                           Email: fiziks.physics@gmail.com                                                                   2 
fiziks
Institute for NET/JRF, GATE, IIT‐JAM, JEST, TIFR and GRE in PHYSICAL SCIENCES 
 
Q8. A bird leaves its nest and flies away. Its distance x from the nest is plotted as a function of time
t . Which of the following plots cannot be right?
(a) (b)

x x

nest t nest t

(c) (d)

x x

nest t nest t
Q9. A cubical cardboard box made of 1cm thick card board has outer side of 29 cm . A tight-fitting
cubical box of the same thickness is placed inside it, then another one inside it and so on. How
many cubical boxes will be there in the entire set?
(a) 29 (b) 28 (c) 15 (d) 14
Q10. Secondary colours are made by a mixture of three primary colours, Red, Green and Blue, in
different proportions; each of the primary colours comes in 8 possible levels. Grey corresponds
to equal proportions of Red, Green and Blue. How many shades of grey exist ii this scheme?
(a) 83 (b) 8 (c) 38 (d) 8  3
Q11. The triangle formed by the lines y  x, y  1  x and x  0 in a two dimensional plane is ( x and
y axes have the same scale)
(a) isosceles and right-angled (b) isosceles but not right-angled
(c) right-angled but not isosceles (d) neither isosceles nor right angled
Q12. There are two buckets A and B . Initially A has 2 liters of water and B is empty. At every
1
hour 1 liter of water is transferred from A to B followed by returning liter back to A from
2
B half an hour later. The earliest A will get empty is in:
(a) 5 h (b) 4 h (c) 3 h (d) 2 h

                                                                                
Head office  Branch office 
 
fiziks, H.No. 23, G.F, Jia Sarai,  Anand Institute of Mathematics, 
 
Near IIT, Hauz Khas, New Delhi‐16  28‐B/6, Jia Sarai, Near IIT 
 
Phone: 011‐26865455/+91‐9871145498 Hauz Khas, New Delhi‐16 
                                                   
                                             Website: www.physicsbyfiziks.com                                                                                          
                                                           Email: fiziks.physics@gmail.com                                                                   3 
fiziks
Institute for NET/JRF, GATE, IIT‐JAM, JEST, TIFR and GRE in PHYSICAL SCIENCES 
 
Q13. Statement A: The following statement is true
Statement B: The preceding statement is false
Choose the correct inference from the following:
(a) Statements A and B are always true
(b) Statements A and B can be true if there is at least one statement between A and B
(c) Statements A and B can be true if there are at least two statements between A and B
(d) Statements A and B can never be true, independently
Q14. A car is moving at 60 km / h . The instantaneous velocity of the upper most points of its wheels is
(a) 60 km / h forward (b) 120 km / h forward
(c) 60 km / h backward (d) 120 km / h backward
Q15. If D  I  M  1501
C  I  V  I  L  157
L  I  V  I  D  557
C  I  V  I  C  207
What is V  I  M  ?
(a) Cannot be found (b) 1009
(c) 1006 (d) 509
Q16. A living cell has a protoplasm which is water based and demarcated by a lipid bilayer membrane.
1
If a cell is pierced up to th of its diameter with a very sharp needle, after taking the needle out
5
(a) no effect will be observed.
(b) protoplasm will leak out from the hole made by the needle for a few minutes until the cell
heals the wound.
(c) protoplasm will keep on leaking out till the cell is dead.
(d) the cell will burst like a balloon.

                                                                                
Head office  Branch office 
 
fiziks, H.No. 23, G.F, Jia Sarai,  Anand Institute of Mathematics, 
 
Near IIT, Hauz Khas, New Delhi‐16  28‐B/6, Jia Sarai, Near IIT 
 
Phone: 011‐26865455/+91‐9871145498 Hauz Khas, New Delhi‐16 
                                                   
                                             Website: www.physicsbyfiziks.com                                                                                          
                                                           Email: fiziks.physics@gmail.com                                                                   4 
fiziks
Institute for NET/JRF, GATE, IIT‐JAM, JEST, TIFR and GRE in PHYSICAL SCIENCES 
 
Q17. Density of a rice grain is 1.5 g / cc and bulk density of rice heap is 0.80 g / cc . If a 1 litre
container is completely filled with rice, what will be the approximate volume of pore space in the
container?
(a) 350 cc (b) 465 cc (c) 550 cc (d) 665 cc
Q18. A turtle starts swimming from a point A located on the circumference of a circular pond. After
swimming for 4 meters in a straight line it hits point B on the circumference or the pond. From
there it changes direction and swims for 3 meters in a straight line and arrives at point D
diametrically opposite to point A . How far is point D from A ?
(a) 3 m (b) 4 m (c) 7 m (d) 5 m
Q19. Four circles of unit radius each are drawn such that each one touches two others and their centres
lie on the vertices of a square. The area of the region enclosed between the circles is
(a)   1 (b)   2 (c) 3   (d) 4  
Q20. A film projector and microscope give equal magnification. But a film projector is not used to see
living cells because
(a) a living cell cannot be placed in a film projector.
(b) the viewer’s eye is close to a microscope whereas it is far away from the projector’s screen.
(c) a microscope produces a virtual image whereas a projector produces a real image.
(d) a microscope has greater resolving power than a projector.
PART ‘B’
Q21. In the scattering of some elementary particles, the scattering cross-section  is found to depend
on the total energy E and the fundamental constants h (Planck’s constant) and c (the speed of
light in vacuum). Using dimensional analysis, the dependence of  on these quantities is given
by
2
hc hc  hc  hc
(a) (b) 3 / 2 (c)   (d)
E E E E

                                                                                
Head office  Branch office 
 
fiziks, H.No. 23, G.F, Jia Sarai,  Anand Institute of Mathematics, 
 
Near IIT, Hauz Khas, New Delhi‐16  28‐B/6, Jia Sarai, Near IIT 
 
Phone: 011‐26865455/+91‐9871145498 Hauz Khas, New Delhi‐16 
                                                   
                                             Website: www.physicsbyfiziks.com                                                                                          
                                                           Email: fiziks.physics@gmail.com                                                                   5 
fiziks
Institute for NET/JRF, GATE, IIT‐JAM, JEST, TIFR and GRE in PHYSICAL SCIENCES 
 
1
Q22. If y  , then x is
tanh  x 

 y 1   y 1 
(a) ln   (b) ln  
 y 1   y 1 

y 1 y 1
(c) ln (d) ln
y 1 y 1

z
Q23. The function of a complex variable z has
sin  z 2
(a) a simple pole at 0 and poles of order 2 at  n for n  1, 2,3...

(b) a simple pole at 0 and poles of order 2 at  n and i n for n  1, 2,3...

(c) poles of order 2 at  n , n  0,1, 2, 3...


(d) poles of order 2 at  n , n  0,1, 2, 3...

Q24. The Fourier transform of f  x  is f  k    dxeikx f  x  .


If f  x     x      x      x  , where   x  is the Dirac delta-function (and prime denotes

derivative), what is f  k  ?

(a)   i  k  i k 2 (b)    k   k 2

(c)   i  k   k 2 (d) i   k  i k 2
dx
Q25. The solution of the differential equation  2 1  x 2 , with initial condition x  0 at t  0 is
dt
   
 sin 2t , 0  t  4 sin 2t , 0  t  2
(a) x   (b) x  
sinh 2t ,   1, 
t t
 4  2

 
sin 2t , 0  t  4
(c) x   (d) x  1  cos 2t , t  0
 1, 
t
 4

                                                                                
Head office  Branch office 
 
fiziks, H.No. 23, G.F, Jia Sarai,  Anand Institute of Mathematics, 
 
Near IIT, Hauz Khas, New Delhi‐16  28‐B/6, Jia Sarai, Near IIT 
 
Phone: 011‐26865455/+91‐9871145498 Hauz Khas, New Delhi‐16 
                                                   
                                             Website: www.physicsbyfiziks.com                                                                                          
                                                           Email: fiziks.physics@gmail.com                                                                   6 
fiziks
Institute for NET/JRF, GATE, IIT‐JAM, JEST, TIFR and GRE in PHYSICAL SCIENCES 
 
Q26. A particle moves in three dimensional space in a central potential V  r   kr 4 where k is a

constant. The angular frequency  for a circular orbit depends on it radius R as


(a)   R (b)   R 1 (c)   R1 / 4 (d)   R 2 / 3
Q27. Two masses m each, are placed at the points  x, y    a, a  and  a, a  and two masses, 2m

each, are placed at the points  a, a  and  a, a  . The principal moments of inertia of the

system are
(a) 2m 2 , 4ma 2 (b) 4ma 2 ,8ma 2

(c) 4ma 2 , 4ma 2 (d) 8ma 2 ,8ma 2


Q28. The Lagrangian of a system is given by
1 5 
L mq12  2mq22  k  q12  2q22  2q1q2 
2 4 
where m and k are positive constants. The frequencies of its normal modes are

(a)
k
,
2m m
3k
(b)
k
2m

13  73 
5k k k 6k
(c) , (d) ,
2m m 2m m
Q29. Consider a particle of mass m moving with a speed v . If TR denotes the relativistic kinetic

energy and TN its non-relativistic approximation, then the value of


TR  TN  for v  0.01 c , is
TR

(a) 1.25  105 (b) 5.0  105


(c) 7.5  105 (d) 1.0  104
Q30. A hollow metallic sphere of radius a , which is kept at a potential V0 has a charge Q at its centre.
The potential at a point outside the sphere, at a distance r from the centre, is
Q Va
(a) V0 (b)  0
4 0 r r

Q V0 a 2 V0 a
(c)  2 (d)
4 0 r r r
                                                                                
Head office  Branch office 
 
fiziks, H.No. 23, G.F, Jia Sarai,  Anand Institute of Mathematics, 
 
Near IIT, Hauz Khas, New Delhi‐16  28‐B/6, Jia Sarai, Near IIT 
 
Phone: 011‐26865455/+91‐9871145498 Hauz Khas, New Delhi‐16 
                                                   
                                             Website: www.physicsbyfiziks.com                                                                                          
                                                           Email: fiziks.physics@gmail.com                                                                   7 
fiziks
Institute for NET/JRF, GATE, IIT‐JAM, JEST, TIFR and GRE in PHYSICAL SCIENCES 
 
Q31. Consider a charge Q at the origin of 3 - dimensional coordinate system. The flux
of the electric field through the curved surface of a cone that has a height h and a
circular base of radius R (as shown in the figure) is h
Q Q hQ QR Q
(a) (b) (c) (d) R
0 2 0 R 0 2h 0

Q32. Given a uniform magnetic field B  B0 kˆ (where B0 is a constant), a possible choice for the
magnetic vector potential A is

(a) B0 yiˆ (b)  B0 yiˆ 


(c) B0 xjˆ  yiˆ  
(d) B0 xiˆ  yjˆ 

Q33. A beam of unpolarized light in a medium with dielectric constant 1 is reflected from a plane

interface formed with another medium of dielectric constant 2  3 1 . The two media have

identical magnetic permeability. If the angle of incidence is 600 , then the reflected light
(a) is plane polarized perpendicular to the plane of incidence
(b) is plane polarized parallel to the plane of incidence
(c) is circularly polarized
(d) has the same polarization as the incident light

Q34. A Hermitian operator O has two normalized eigenstates 1 and 2 with eigenvalues 1 and 2 ,

respectively. The two states u  cos  1  sin  2 and v  cos  1  sin  2 are such that

v O   7 / 4 and u v  0 . Which of the following are possible values of  and  ?

   
(a)    and   (b)   and  
6 3 6 3
   
(c)    and   (d)   and   
4 4 3 6

                                                                                
Head office  Branch office 
 
fiziks, H.No. 23, G.F, Jia Sarai,  Anand Institute of Mathematics, 
 
Near IIT, Hauz Khas, New Delhi‐16  28‐B/6, Jia Sarai, Near IIT 
 
Phone: 011‐26865455/+91‐9871145498 Hauz Khas, New Delhi‐16 
                                                   
                                             Website: www.physicsbyfiziks.com                                                                                          
                                                           Email: fiziks.physics@gmail.com                                                                   8 
fiziks
Institute for NET/JRF, GATE, IIT‐JAM, JEST, TIFR and GRE in PHYSICAL SCIENCES 
 
x
Q35. The ground state energy of a particle of mass m in the potential V  x   V0 cosh   , where L
L
2
and V0 are constants (with V0  ) is approximately
2ml 2

 2V0  V0
(a) V0  (b) V0 
L m L m

 V0  V0
(c) V0  (d) V0 
4L m 2L m
Q36. Let  nlm denote the eigenstates of a hydrogen atom in the usual notation. The state

1
2 200  3 211  7 210  5 211 
5
is an eigenstate of
(a) L2 , but not of the Hamiltonian or Lz

(b) the Hamiltonian, but not of L2 or Lz

(c) the Hamiltonian, L2 and Lz

(d) L2 and Lz , but not of the Hamiltonian

1
Q37. The Hamiltonian for a spin- particle at rest is given by H  E0  z   x  , where  x and  z
2
are Pauli spin matrices and E0 and  are constants. The eigenvalues of this Hamiltonian are

(a)  E0 1   2 (b)  E0 1   2

 1 
(c) E0 (doubly degenerate) (d) E0  1   2 
 2 
Q38. The heat capacity of (the interior of a refrigerator is 4.2 kJ / K . The minimum’ work that must

be done to lower the internal temperature from 18o C to 17 o C when the outside temperature is

27o C will be
(a) 2.20 kJ (b) 0.80 kJ (c) 0.30 kJ (d) 0.14 kJ

                                                                                
Head office  Branch office 
 
fiziks, H.No. 23, G.F, Jia Sarai,  Anand Institute of Mathematics, 
 
Near IIT, Hauz Khas, New Delhi‐16  28‐B/6, Jia Sarai, Near IIT 
 
Phone: 011‐26865455/+91‐9871145498 Hauz Khas, New Delhi‐16 
                                                   
                                             Website: www.physicsbyfiziks.com                                                                                          
                                                           Email: fiziks.physics@gmail.com                                                                   9 
fiziks
Institute for NET/JRF, GATE, IIT‐JAM, JEST, TIFR and GRE in PHYSICAL SCIENCES 
 
Q39. For a system of independent non interacting one-dimensional oscillators, the value of the free
energy per oscillator, in the limit T  0 , is
1 3
(a)  (b)  (c)  (d) 0
2 2
Q40. The partition function of a system of N Ising spins is Z  1N  2N where 1 and 2 are

functions of temperature, but are independent of N . If 1  2 , the free energy per spin in the

limit N   is
 
(a) k BT ln  1  (b) k BT ln 2
 2 
(c) k BT ln  12  (d)  k BT ln 1

1
Q41. The Hamiltonian of a system of N non interacting spin - particles is H   0 B  i Siz , where
2
Siz  1 are components of i th spin along an external magnetic field B . At a temperature T
0 B

such that e kBT  2 . the specific heat per particle is


16 8 16
(c) k B  ln 2  k B  ln 2 
2 2
(a) kB (b) k B ln 2 (d)
25 25 25
Q42. If the reverse bias voltage of a silicon varactor is increased by a factor of 2 , the corresponding
transition capacitance
(a) increases by a factor of 2 (b) increases by a factor of 2

(c) decreases h a factor of 2 (d) decreases by a factor of 2

                                                                                
Head office  Branch office 
 
fiziks, H.No. 23, G.F, Jia Sarai,  Anand Institute of Mathematics, 
 
Near IIT, Hauz Khas, New Delhi‐16  28‐B/6, Jia Sarai, Near IIT 
 
Phone: 011‐26865455/+91‐9871145498 Hauz Khas, New Delhi‐16 
                                                   
                                             Website: www.physicsbyfiziks.com                                                                                          
                                                           Email: fiziks.physics@gmail.com                                                                   10 
fiziks
Institute for NET/JRF, GATE, IIT‐JAM, JEST, TIFR and GRE in PHYSICAL SCIENCES 
 
Q43. In the schematic figure given below the initial values of 4 bit shift registers A and B are 1011
and 0010 respectively The values at SOA . and SOB after the pulse T2 are respectively.

Shift Register A Shift Register B


Sl A SOA SlB SOB
CLK CLK
CLOCK
SHIFT

T0 T1 T2

CLOCK

SHIFT
(a) 1110 and1001 (b) 1101 and1001
(c) 1101 and 1100 (d) 1110 and 1100

Q44. If the parameters y and x are related by y  log  x  , then the circuit that can be used to produce

an output voltage V0 varying linearly with x is

(a) y (b) y
 
Vo Vo
 

(c) (d)
y  y 
Vo Vo
 

                                                                                
Head office  Branch office 
 
fiziks, H.No. 23, G.F, Jia Sarai,  Anand Institute of Mathematics, 
 
Near IIT, Hauz Khas, New Delhi‐16  28‐B/6, Jia Sarai, Near IIT 
 
Phone: 011‐26865455/+91‐9871145498 Hauz Khas, New Delhi‐16 
                                                   
                                             Website: www.physicsbyfiziks.com                                                                                          
                                                           Email: fiziks.physics@gmail.com                                                                   11 
fiziks
Institute for NET/JRF, GATE, IIT‐JAM, JEST, TIFR and GRE in PHYSICAL SCIENCES 
 
Q45. Two data sets A and B consist of 60 and 10 readings of a voltage measured using voltmeters
of resolution of 1 mV and 0.5 mV respectively. The uncertainty in the mean voltage obtained
from the data sets A and B are U A and U B , respectively. If the uncertainty of the mean of the

combined data sets is U AB then which of the following statements is correct?

(a) U AB  U A and U AB  U B (b) U AB  U A and U AB  U B

(c) U AB  U A and U AB  U B (d) U AB  U A and U AB  U B

PART ‘C’
Q46. The Hermite polynomial H n  x  satisfies the differential equation

d 2Hn dH n
2
 2x  2nH n  x   0
dx dx
The corresponding generating function
1
G  t , x    n 0 H n  x  t n satisfies the equation

n!
 2G G G  2G G G
(a)  2x  2t 0 (b)  2x  2t 2 0
x 2
x t x 2
x t
 2G G G  2G G  2G
(c)  2x 2 0 (d)  2x 2 0
x 2
x t x 2 x xt
Q47. A function f  x  satisfies the differential equation

d2 f
2
  2 f    x  a 
dx

where  is positive. The Fourier transform f  k   f  dx eikx f  x  of f , and the solution of


the equation are, respectively,

(a)
eika
k 
2 2
and
1  x  a
2
e e 
 xa
 (b)
eika
k 
2 2
and
1  x  a
2
e

(c)
eika
k 
2 2
and
1  i x  a
2
e e 
i x  a
 (d)
eika
k 
2 2
and
1
2i

e  x  a  ei x  a 

                                                                                
Head office  Branch office 
 
fiziks, H.No. 23, G.F, Jia Sarai,  Anand Institute of Mathematics, 
 
Near IIT, Hauz Khas, New Delhi‐16  28‐B/6, Jia Sarai, Near IIT 
 
Phone: 011‐26865455/+91‐9871145498 Hauz Khas, New Delhi‐16 
                                                   
                                             Website: www.physicsbyfiziks.com                                                                                          
                                                           Email: fiziks.physics@gmail.com                                                                   12 
fiziks
Institute for NET/JRF, GATE, IIT‐JAM, JEST, TIFR and GRE in PHYSICAL SCIENCES 
 
dx
Q48. For a dynamical system governed by the equation  2 1  x 2 , with x  1
dt
(a) x  1 and x  1 are both unstable fixed points
(b) x  1 and x  1 are both stable fixed points
(c) x  1 is an unstable fixed point and x  1 is a stable fixed points
(d) x  1 is a stable fixed point and x  1 is a unstable fixed points
8 1 1
Q49. The value of the integral  0 x 5
2
dx , valuated using Simpson’s rule with h  2 is
3
(a) 0.565 (b) 0.620 (c) 0.698 (d) 0.736
Q50. A canonical transformation  p, q    P, Q  is performed on the Hamiltonian

1 1
H 2
 m 2 q 2 via the generating function
2mp 2
1
F m q 2 cot Q . If Q  0   0 , which of the following graphs shows schematically the
2
dependence of Q  t  on t ?

(a) (b)
Q t 
Q t 

(c) (d)
Q t  Q t 

                                                                                
Head office  Branch office 
 
fiziks, H.No. 23, G.F, Jia Sarai,  Anand Institute of Mathematics, 
 
Near IIT, Hauz Khas, New Delhi‐16  28‐B/6, Jia Sarai, Near IIT 
 
Phone: 011‐26865455/+91‐9871145498 Hauz Khas, New Delhi‐16 
                                                   
                                             Website: www.physicsbyfiziks.com                                                                                          
                                                           Email: fiziks.physics@gmail.com                                                                   13 
fiziks
Institute for NET/JRF, GATE, IIT‐JAM, JEST, TIFR and GRE in PHYSICAL SCIENCES 
 
Q51. A distant source, emitting radiation of frequency  moves with a velocity source
4c / 5
4c
in a certain direction with respect to a receiver (as shown in the figure). 
5
3
The upper cut-off frequency of the receiver is . Let  the angle as
2
Receiver
shown. For the receiver to detect the radiation,  should at least be

1 3  2   2
(a) cos 1   (b) cos 1   (c) cos 1   (d) cos 1  
2 4  5  3
Q52. The Lagrangian of a particle moving in a plane s given in Cartesian coordinates as
L  xy
  x 2  y 2
In polar coordinates the expression for the canonical momentum pr (conjugate to the radial
coordinate r ) is
(a) r sin   r cos  (b) r cos   r sin 
(c) 2r cos   r sin 2 (d) r sin 2  r cos 2
Q53. A small magnetic needle is kept at  0, 0  with its moment along the x -axis. Another small

magnetic needle is at the point 1,1 and is free to rotate in the xy - plane. In equilibrium the

angle  between their magnetic moments is such that


1
(a) tan   (b) tan   0 (c) tan   3 (d) tan   1
3

Q54. A dipole of moment p , oscillating at frequency  , radiates spherical waves. The vector
potential at large distance is
   eikr 
A  r   0 i p
4 r
1  
To order   the magnetic field B at a point r  rnˆ is
r
0  2  eikr 0  2  eikr
(a)    nˆ
ˆ
n  p (b)   ˆ
n  p 
4 C r 4 C r
0 2   eikr  0  2  eikr
(c)   k  nˆ  p  p (d)  p
4 r 4 C r
                                                                                
Head office  Branch office 
 
fiziks, H.No. 23, G.F, Jia Sarai,  Anand Institute of Mathematics, 
 
Near IIT, Hauz Khas, New Delhi‐16  28‐B/6, Jia Sarai, Near IIT 
 
Phone: 011‐26865455/+91‐9871145498 Hauz Khas, New Delhi‐16 
                                                   
                                             Website: www.physicsbyfiziks.com                                                                                          
                                                           Email: fiziks.physics@gmail.com                                                                   14 
fiziks
Institute for NET/JRF, GATE, IIT‐JAM, JEST, TIFR and GRE in PHYSICAL SCIENCES 
 
Q55. The frequency dependent dielectric constant of a material is given by
A
    1 
   2  i
2
0

where A is a positive constant, 0 the resonant frequency and  the damping coefficient. For an

electromagnetic wave of angular frequency   0 which of the following is true? (Assume


that  1 ).
0
(a) There is negligible absorption of the wave
(b) The wave propagation is highly dispersive
(c) There is strong absorption of the electromagnetic wave
(d) The group velocity and the phase velocity will have opposite sign
Q56. A hydrogen atom is subjected to the perturbation
2r
V pert  r   cos
a0

where a0 is the Bohr radius. The change in the ground state energy to first order in 

   
(a) (b) (c) (d)
4 2 2 4
Q57. A positron is suddenly absorbed by the nucleus of a tritium  H  atom to turn the latter into a
3
1

He  ion. If the electron in the tritium atom was initially in the ground state, the probability that
the resulting He  ion will be in its ground state is
8 128 512
(a) 1 (b) (c) (d)
9 243 729
Q58. The product of the uncertainties  Lx   Ly  for a particle in the state a 1,1  b 1, 1 where

l , m denotes an eigenstate of L2 and Lz will be a minimum for

(a) a  ib (b) a  0 and b  1

3 1
(c) a  and b  (d) a  b
2 2

                                                                                
Head office  Branch office 
 
fiziks, H.No. 23, G.F, Jia Sarai,  Anand Institute of Mathematics, 
 
Near IIT, Hauz Khas, New Delhi‐16  28‐B/6, Jia Sarai, Near IIT 
 
Phone: 011‐26865455/+91‐9871145498 Hauz Khas, New Delhi‐16 
                                                   
                                             Website: www.physicsbyfiziks.com                                                                                          
                                                           Email: fiziks.physics@gmail.com                                                                   15 
fiziks
Institute for NET/JRF, GATE, IIT‐JAM, JEST, TIFR and GRE in PHYSICAL SCIENCES 
 
Q59. The ground state energy of a particle in potential V  x   g x , estimated using the trail
wavefunction
 c 2
 5 a x ,
  x   a
2
  x a
0, x a

(where g and c are constants) is
1/ 3 1/ 3 1/ 3 1/ 3
15   2 g 2  5  2 g 2  3  2 g 2  7  2 g 2 
(a)   (b)   (c)   (d)  
16  m  6 m  4 m  8 m 

1
Q60. An ensemble of non-interacting spin - particles is in contact with a heat bath at temperature T
2
and is subjected to an external magnetic field. Each particle can be in one of the two quantum
states of energies  0 . If the mean energy per particle is  0 / 2 , then the free energy per
particle is

(a) 2 0

ln 4 / 3  (b)  0 ln  3/ 2  (c) 2 0 ln 2 (d)  0
ln 2
ln 3 ln 3
Q61. Which of the following graphs shows the qualitative dependence of the free energy f  h, T  of a

ferromagnet in an external magnetic field h , and at a fixed temperature T  TC , where TC is the

critical temperature?
f
(a) f (b)
h

h
f f
(c) (d)

h h

                                                                                
Head office  Branch office 
 
fiziks, H.No. 23, G.F, Jia Sarai,  Anand Institute of Mathematics, 
 
Near IIT, Hauz Khas, New Delhi‐16  28‐B/6, Jia Sarai, Near IIT 
 
Phone: 011‐26865455/+91‐9871145498 Hauz Khas, New Delhi‐16 
                                                   
                                             Website: www.physicsbyfiziks.com                                                                                          
                                                           Email: fiziks.physics@gmail.com                                                                   16 
fiziks
Institute for NET/JRF, GATE, IIT‐JAM, JEST, TIFR and GRE in PHYSICAL SCIENCES 
 
Q62. Consider a random walker on a square lattice. At each step the walker moves to a nearest
neighbour site with equal probability for each of the four sites. The walker starts at the origin and
takes 3 steps. The probability that during this walk no site is visited more than one is
(a) 12 / 27 (b) 27 / 64 (c) 3/ 8 (d) 9 /16
Q63. Consider an n - MOSFET with the following parameters: current drive 15V
strength K  60 A / V 2 , breakdown voltage BVDS  10 V , ratio of effective 80 k 

W VDS
gate width to the channel length  5 and threshold voltage Vth  0.5V . 1.2 V
L
In the circuit given below, this n - MOSFET is operating in the
(a) ohmic region (b) cut-off region
(c) saturation region (d) breakdown
Q64. The state diagram corresponding to the following circuit is

x D A
y
CLOCK

Flip Flop

00, 01,10 01,11


(a) 11 00 (b) 00,10 00,10

0 1 0 1

01,10,11 01,11

00,11 00, 01,10


(c) 01,10 00,10 (d) 11 11

0 1

00,11 00, 01,10

                                                                                
Head office  Branch office 
 
fiziks, H.No. 23, G.F, Jia Sarai,  Anand Institute of Mathematics, 
 
Near IIT, Hauz Khas, New Delhi‐16  28‐B/6, Jia Sarai, Near IIT 
 
Phone: 011‐26865455/+91‐9871145498 Hauz Khas, New Delhi‐16 
                                                   
                                             Website: www.physicsbyfiziks.com                                                                                          
                                                           Email: fiziks.physics@gmail.com                                                                   17 
fiziks
Institute for NET/JRF, GATE, IIT‐JAM, JEST, TIFR and GRE in PHYSICAL SCIENCES 
 
Q65. A sinusoidal signal of peak to peak amplitude 1V and unknown time period is input to the

following circuit for 5 seconds duration. If the counter measures a value  3E8  H in hexadecimal

then the time period of the input signal is


0.1  F
 10 bit
Vi  counter
1K 10 K

(a) 2.5 ms (b) 4 ms (c) 10 ms (d) 5 ms


Q66. The first order diffraction peak of a crystalline solid occurs at a scattering angle of 300 when the
diffraction pattern is recorded using an x-ray beam of wavelength 0.15 nm . If the error in

measurements of the wavelength and the angle are 0.01 nm and 10 respectively, then the error in
calculating the inter-planar spacing will approximately be
(a) 1.1 102 nm (b) 1.3  104 nm (c) 2.5  102 nm (d) 2.0  10 3 nm
Q67. The dispersion relation of electrons in a 3-dimensional lattice in the tight binding approximation
is given by,
 k   cos k x a   cos k y a   cos k z a
where a is the lattice constant and  ,  ,  are constants with dimension of energy. The effective
   
mass tensor at the corner of the first Brillouin zone  , ,  is
a a a
 1   1 
 0 0  0 0 
     
 
2
1   
2
1 
(a) 2  0  0 (b) 2  0  0 
a    a   
 1  1
 0 0  0 0  
    

1  1 
 0 0  0 0 
   
2  1  2  1 
(c) 2  0 0 (d) 2  0 0 
a    a   
 1  1
0 0 0 0  
    

                                                                                
Head office  Branch office 
 
fiziks, H.No. 23, G.F, Jia Sarai,  Anand Institute of Mathematics, 
 
Near IIT, Hauz Khas, New Delhi‐16  28‐B/6, Jia Sarai, Near IIT 
 
Phone: 011‐26865455/+91‐9871145498 Hauz Khas, New Delhi‐16 
                                                   
                                             Website: www.physicsbyfiziks.com                                                                                          
                                                           Email: fiziks.physics@gmail.com                                                                   18 
fiziks
Institute for NET/JRF, GATE, IIT‐JAM, JEST, TIFR and GRE in PHYSICAL SCIENCES 
 
Q68. A thin metal film of dimension 2 mm  2 mm contains 4  1012 electrons. The magnitude of the
Fermi wavevector of the system, in the free electron approximation, is
(a) 2   107 cm 1 (b) 2  107 cm 1 (c)   107 cm 1 (d) 2  107 cm 1
Q69. For an electron moving through a one-dimensional periodic lattice of periodicity a , which of the
following corresponds to an energy eigenfunction consistent with Bloch’s theorem?
  x   x     x  2 x   
(a)   x   A exp  i   cos    (b)   x   A exp  i   cos   
  a  2a      a  a  

  2 x  2 x      x  x 
(c)   x   A exp  i   i cosh     (d)   x   A exp  i  i 
  a  a     a 2a  

Q70. The LS configurations of the ground state of 12Mg , 13


Al , 17 Cl and 18
Ar are, respectively,

(a) 3 S1 , 2 P1 / 2 , 2 P1 / 2 and 1 S0 (b) 3 S1 , 2 P3 / 2 , 2 P3 / 2 and 3 S1

(c) 1 S0 , 2 P1/ 2 , 2 P3 / 2 and 1 S0 (d) 1 S0 , 2 P3 / 2 , 2 P1/ 2 and 3 S1

Q71. For a two level system, the population of atoms in the upper and lower levels are 3  1018 and
0.7  1018 , respectively. If the coefficient of stimulated emission is 3.0  105 m 3 / W -s 3 and the
energy density is 9.0 J / m3 -Hz , the rate of stimulated emission will be

(a) 6.3 1016 s 1 (b) 4.1  1016 s 1 (c) 2.7  1016 s 1 (d) 1.8  1016 s 1
Q72. The first ionization potential of K is 4.34 eV , the electron affinity of CI is 3.82 eV and the
equilibrium separation of KCI is 0.3 nm . The required to dissociate
a KCI molecule into a K and a CI atom is
(a) 8.62 eV (b) 8.16 eV (c) 4.28 eV (d) 4.14 eV

                                                                                
Head office  Branch office 
 
fiziks, H.No. 23, G.F, Jia Sarai,  Anand Institute of Mathematics, 
 
Near IIT, Hauz Khas, New Delhi‐16  28‐B/6, Jia Sarai, Near IIT 
 
Phone: 011‐26865455/+91‐9871145498 Hauz Khas, New Delhi‐16 
                                                   
                                             Website: www.physicsbyfiziks.com                                                                                          
                                                           Email: fiziks.physics@gmail.com                                                                   19 
fiziks
Institute for NET/JRF, GATE, IIT‐JAM, JEST, TIFR and GRE in PHYSICAL SCIENCES 
 
Q73. Consider the following processes involving free particles
(i) n  p  e   ve (ii) p  n   

(iii) p  n      0   0 (iv) p  ve  n  e 

Which of the following statements is true?


(a) Process (i) obeys all conservation laws
(b) Process (ii) conserves baryon number, but violates energy-momentum conservation
(c) process (iii) is not allowed by strong interaction but is allowed by weak interactions
(d) Process (iv) conserves baryon number, but violates lepton number conservation

Q74. The electric quadrupole moment of an odd proton nucleus is


 2 j  1 r 2 , where j is the total
2  j  1

angular momentum. Given that R0  1.2 fm , what is the value in barn, of the quadrupole
27
moment of the Al nucleus in the shell model?
(a) 0.043 (b) 0.023 (c) 0.915 (d) 0
Q75. Of the nuclei of mass number A  125 , the binding energy calculated from the liquid drop model
(given that the coefficients for the Coulomb and the asymmetry energy are ac  0.7 MeV and

asym  22.5 MeV respectively) is a maximum for


125 124 125 125
(a) 54 Xe (b) 53 I (c) 52 Te (d) 51 Sb

                                                                                
Head office  Branch office 
 
fiziks, H.No. 23, G.F, Jia Sarai,  Anand Institute of Mathematics, 
 
Near IIT, Hauz Khas, New Delhi‐16  28‐B/6, Jia Sarai, Near IIT 
 
Phone: 011‐26865455/+91‐9871145498 Hauz Khas, New Delhi‐16 
                                                   
                                             Website: www.physicsbyfiziks.com                                                                                          
                                                           Email: fiziks.physics@gmail.com                                                                   20 
fiziks
Institute for NET/JRF, GATE, IIT‐JAM, JEST, TIFR and GRE in PHYSICAL SCIENCES 
 
NET-JRF JUNE 2015
PART A
Q1. By reading the accompanying graph, determine the liquid
INCORRECT statement out of the following.

Pressure
solid gas
(a) Melting point increases with pressure
(b) Melting point decreases with pressure
(c) Boiling point increases with pressure
Temperature
(d) Solid, liquid and gas can co-exist at the same pressure and
temperature
Q2. A float is drifting in a river 10 m downstream of a boat that can be rowed at a speed of
10 m / minute in still water. If the boat is rowed downstream, the time taken to catch up
with the float
(a) will be 1 minute (b) will be more than 1 minute
(c) will be less than 1 minute
(d) can be determined only if the speed of the river is known
Q3. Consider a series of letters placed in the following way:
U _G_C _C _ S _ I _ R
Each letter moves one step to its right and the extreme right letter takes the first position,
completing one operation. After which of the following numbers of operations do the Cs
not sit side by side?
(a) 3 (b) 10 (c) 19 (d) 25
Q4. If you change only one observation from a set of 10 observations, which of the following
will definitely charge?
(a) Mean (b) Median (c) Mode (d) Standard deviation
Q5. An inclined plane rests against a horizontal cylinder of radius R . If the plane makes an
angle of 300 with the ground, the point of contact of the plane with the cylinder is at a
height of
(a) 1.500 R (b) 1.866 R (c) 1.414 R (d) 1.000 R

                                                                                
Head office  Branch office 
 
fiziks, H.No. 23, G.F, Jia Sarai,  Anand Institute of Mathematics, 
 
Near IIT, Hauz Khas, New Delhi‐16  28‐B/6, Jia Sarai, Near IIT 
 
Phone: 011‐26865455/+91‐9871145498 Hauz Khas, New Delhi‐16 
                                                   
                                             Website: www.physicsbyfiziks.com                                                                                          
                                                           Email: fiziks.physics@gmail.com                                                                   1 
fiziks
Institute for NET/JRF, GATE, IIT‐JAM, JEST, TIFR and GRE in PHYSICAL SCIENCES 
 
Q6. In a fast moving car with open windows, the driver feels a continuous incoming breeze.
The pressure inside the car, however, does not keep increasing because,
(a) air coming in from the front window goes out from the rear
(b) air comes is as well as goes out through every window but the driver only feels the
incoming one.
(c) no air actually comes in and the feeling of breeze is an illusion.
(d) Cool air reduces the temperature therefore the pressure does not increase
Q7. What is the maximum number of parallel, non-overlapping cricket pitches (length 24 m ,
width 3m ) that can be laid in a field of diameter 140 m , if the boundary is required to be
at least 60 m from the centre of any pitch?
(a) 6 (b) 7 (c) 12 (d) 4
Q8. The product of the perimeter of a triangle, the radius of its in-circle, and a number gives
the area of the triangle. The number is
1 1 1
(a) (b) (c) (d) 1
4 3 2
Q9. The maximum number of points formed by intersection of all pairs of diagonals of
convex octagon is
(a) 70 (b) 400 (c) 120 (d) 190
Q10. Each of the following pairs of words hides a number, based on which you can arrange
them in ascending order. Pick the correct answer:
I. Cloth reel K. Silent wonder
J. Good tone L. Bronze rod
(a) L, K, J, I (b) I, J, K, L
(c) K, L, J, I (d) K, J, I, L
22
Q11. Which of the following values is same as 22 ?
(a) 26 (b) 28 (c) 216 (d) 2222

                                                                                
Head office  Branch office 
 
fiziks, H.No. 23, G.F, Jia Sarai,  Anand Institute of Mathematics, 
 
Near IIT, Hauz Khas, New Delhi‐16  28‐B/6, Jia Sarai, Near IIT 
 
Phone: 011‐26865455/+91‐9871145498 Hauz Khas, New Delhi‐16 
                                                   
                                             Website: www.physicsbyfiziks.com                                                                                          
                                                           Email: fiziks.physics@gmail.com                                                                   2 
fiziks
Institute for NET/JRF, GATE, IIT‐JAM, JEST, TIFR and GRE in PHYSICAL SCIENCES 
 
Q12. If
2a
 b2
c6
84
8d 6

Here a, b, c and d are digits. Then a  b 


(a) 4 (b) 9 (c) 11 (d) 16
Q13. A 12 m  4 m rectangular roof is resting on four 4 m tall thin poles. Sunlight falls on the

roof at an angle of 450 from the east, creating a shadow on the ground. What will be the
area of the shadow?
(a) 24 m 2 (b) 36 m 2 (c) 48 m 2 (d) 60 m 2
Q14. Find the height of a box of base area 24 cm  48 cm , in which the longest stick that can be
kept is 56 cm long.
(a) 8 cm (b) 32 cm (c) 37.5 cm (d) 16 cm
Q15. An infinite row of boxes is arranged. Each box has half the volume of the previous box.
If the largest box has volume of 20 cc , what is the total volume of all the boxes?
(a) Infinite (b) 400 cc (c) 40 cc (d) 80 cc
Q16. Find the missing element based on the given pattern

1  2  3

1  2  3

(a) (b) (c) (d)

                                                                                
Head office  Branch office 
 
fiziks, H.No. 23, G.F, Jia Sarai,  Anand Institute of Mathematics, 
 
Near IIT, Hauz Khas, New Delhi‐16  28‐B/6, Jia Sarai, Near IIT 
 
Phone: 011‐26865455/+91‐9871145498 Hauz Khas, New Delhi‐16 
                                                   
                                             Website: www.physicsbyfiziks.com                                                                                          
                                                           Email: fiziks.physics@gmail.com                                                                   3 
fiziks
Institute for NET/JRF, GATE, IIT‐JAM, JEST, TIFR and GRE in PHYSICAL SCIENCES 
 
Q17. A man starts his journey at 0100 Hrs local time to reach another country at 0900 Hrs
local time on the same date. He starts a return journey on the same night at 2100 hrs
local time to his original place, taking the same time to travel back. If the time zone of his
country of visit lags by 10 hours, the duration for which the man was away from his
place is
(a) 48 hours (b) 20 hours (c) 25 hours (d) 36 hours
 1   1 
   
Q18. Let r be a positive number satisfying r  1234 
r  1234 
 2 . Then
r 4321  r 4321  ?
 4321 
 
 1234 
(a) 2 (b) 2 (c) 23087 (d) 21234
Q19. ABC is right angled triangle inscribed in a semicircle. Smaller semicircles are drawn on
sides BC and AC . If the area of the triangle is a , what is the total area of the shaded
lumes? B

A
C

 a a
(a) a (b) (c) (d)
a  2
Q20. An ant can lift another ant of its size whereas an elephant cannot lift another elephant of
its size, because
(a) ant muscle fibres are stronger than elephant muscle fibres
(b) ant has proportionately thicker legs than elephant
(c) strength scales as the square of the size while weight scales as cube of the size
(d) ants work cooperatively, whereas elephants work as individuals

                                                                                
Head office  Branch office 
 
fiziks, H.No. 23, G.F, Jia Sarai,  Anand Institute of Mathematics, 
 
Near IIT, Hauz Khas, New Delhi‐16  28‐B/6, Jia Sarai, Near IIT 
 
Phone: 011‐26865455/+91‐9871145498 Hauz Khas, New Delhi‐16 
                                                   
                                             Website: www.physicsbyfiziks.com                                                                                          
                                                           Email: fiziks.physics@gmail.com                                                                   4 
fiziks
Institute for NET/JRF, GATE, IIT‐JAM, JEST, TIFR and GRE in PHYSICAL SCIENCES 
 
PART – ‘B’
Q21. A particle moves in two dimensions on the ellipse x 2  4 y 2  8 . At a particular instant it is

at the point  x, y    2,1 and the x -component of its velocity is 6 (in suitable units).

Then the y-component of its velocity is


(a) 3 (b) 2 (c) 1 (d) 4
 dx
Q22. The value of integral  1  x 4

 
(a) (b) (c) 2 (d) 2
2 2
Q23. Consider the periodic function f  t  with time period T as show in the figure below.

f t 

t
3 2 1 1 2 3

1
The spikes, located at t   2n  1 , where n  0, 1, 2,..., are Dirac-delta function of
2

strength 1 . The amplitudes an in the Fourier expansion


 2 int
f t   ae
n 
n
T
are given by

1 n n
(a)  1 (d) n
n
(b) sin (c) i sin
n 2 2
Q24. Consider three inertial frames of reference A, B and C . The frame B moves with a
c c
velocity with respect to A , and C moves with a velocity with respect to B in the
2 10
same direction. The velocity of C as measured in A is

3c 4c c 3c
(a) (b) (c) (d)
7 7 7 7

                                                                                
Head office  Branch office 
 
fiziks, H.No. 23, G.F, Jia Sarai,  Anand Institute of Mathematics, 
 
Near IIT, Hauz Khas, New Delhi‐16  28‐B/6, Jia Sarai, Near IIT 
 
Phone: 011‐26865455/+91‐9871145498 Hauz Khas, New Delhi‐16 
                                                   
                                             Website: www.physicsbyfiziks.com                                                                                          
                                                           Email: fiziks.physics@gmail.com                                                                   5 
fiziks
Institute for NET/JRF, GATE, IIT‐JAM, JEST, TIFR and GRE in PHYSICAL SCIENCES 
 
1
Q25. If the Lagrangian of a dynamical system in two dimensions is L  mx 2  mxy , then its
2
Hamiltonian is
1 1 2 1 1 2
(a) H  px p y  py (b) H  px p y  px
m 2m m 2m
1 1 2 1 1 2
(c) H  px p y  py (d) H  px p y  px
m 2m m 2m
d 2x dx
Q26. Consider the differential equation 2
 3  2 x  0 . If x  0 at t  0 and x  1 at t  1 ,
dt dt
the value of x at t  2 is
(a) e 2  1 (b) e 2  e (c) e  2 (d) 2e
Q27. A Plan electromagnetic wave is travelling along the positive z -direction. The maximum
electric field along the x - direction is 10 V / m . The approximate maximum values of the
power per unit area and the magnetic induction B , respectively, are
(a) 3.3 107 watts / m 2 and 10 tesla

(b) 3.3 107 watts / m 2 and 3.3  10 tesla

(c) 0.265 watts / m 2 and 10 tesla

(d) 0.265 watts / m 2 and 3.3 108 tesla


 
Q28.   
Which of the following transformations V , A  V ', A ' of the electrostatic potential 

V and the vector potential A is a gauge transformation?
   

(a) V   V  ax, A  A  atkˆ 
(b) V   V  ax, A  A  atkˆ  
   

(c) V   V  ax, A  A  atiˆ  
(d) V   V  ax, A  A  atiˆ 
Q29. The ratio of the energy of the first excited state E1 of a particle in a three-dimensional

L
rectangular box of side L, L and , is
2
(a) 3 : 2 (b) 2 :1 (c) 4 :1 (d) 4 : 3

                                                                                
Head office  Branch office 
 
fiziks, H.No. 23, G.F, Jia Sarai,  Anand Institute of Mathematics, 
 
Near IIT, Hauz Khas, New Delhi‐16  28‐B/6, Jia Sarai, Near IIT 
 
Phone: 011‐26865455/+91‐9871145498 Hauz Khas, New Delhi‐16 
                                                   
                                             Website: www.physicsbyfiziks.com                                                                                          
                                                           Email: fiziks.physics@gmail.com                                                                   6 
fiziks
Institute for NET/JRF, GATE, IIT‐JAM, JEST, TIFR and GRE in PHYSICAL SCIENCES 
 
Q30. Suppose the yz -plane forms a chargeless boundary between two media of permittivities
 left and  right where  left :  right  1: 2 . If the uniform electric field on the left is
 
 
Eleft  c iˆ  ˆj  kˆ (where c is a constant), than the electric field on the right Eright is


(a) c 2iˆ  ˆj  kˆ  
(b) c iˆ  2 ˆj  2kˆ 
1   1 1 
(c) c  iˆ  ˆj  kˆ  (d) c  iˆ  ˆj  kˆ 
2   2 2 

Q31. If Li are the components of the angular momentum operator  i1,2,3   , Li  , Li  equals
  L
   
(a) L (b) 2L (c) 3L (d)  L
Q32. The Laplace transform of 6t 3  3sin 4t is
36 12 36 12
(a)  2 (b)  2
s 4
s  16 s 4
s  16
18 12 36 12
(c)  2 (d)  2
s 4
s  16 s 3
s  16
 
Q33. A particle of mass m moves in the one dimensional potential V  x   x3  x 4 where
3 4
 ,   0 . One of the equilibrium points is x  0 . The angular frequency of small
oscillations about the other equilibrium point is
2   
(a) (b) (c) (d)
3m m 12m 24m

Q34. A particle of unit mass moves in the xy -plane in such a way that x  t   y  t  and

y  t    x  t  . We can conclude that it is in a conservative force-field which can be

derived from the potential

(a)
1 2
2

x  y2  (b)
1 2
2

x  y2 
(c) x  y (d) x  y

                                                                                
Head office  Branch office 
 
fiziks, H.No. 23, G.F, Jia Sarai,  Anand Institute of Mathematics, 
 
Near IIT, Hauz Khas, New Delhi‐16  28‐B/6, Jia Sarai, Near IIT 
 
Phone: 011‐26865455/+91‐9871145498 Hauz Khas, New Delhi‐16 
                                                   
                                             Website: www.physicsbyfiziks.com                                                                                          
                                                           Email: fiziks.physics@gmail.com                                                                   7 
fiziks
Institute for NET/JRF, GATE, IIT‐JAM, JEST, TIFR and GRE in PHYSICAL SCIENCES 
 
Q35. A proton moves with a speed of 300 m / s in a circular orbit in the xy -plan in a magnetic
field 1 tesla along the positive z -direction. When an electric field of 1 V / m is applied
along the positive y -direction, the center of the circular orbit
(a) remains stationary
(b) moves at 1 m / s along the negative x -direction
(c) moves at 1 m / s along the positive z -direction
(d) moves at 1 m / s along the positive x -direction

Q36. The wavefunction of a particle in one-dimension is denoted by   x  in the coordinate


 ipx
represention and by   p     x  e 
dx in the momentum representation. If the action
of an operator T̂ on   x  is given by Tˆ  x     x  a  , where a is a constant then
Tˆ  p  is given by
 iap
i
(a)  ap  p  (b) e 
  p

 iap
 i 
(c) e 
  p (d) 1  ap    p 
  
1
Q37. A particle moves in one dimension in the potential V  k  t  x 2 , where k  t  is a time
2
d
dependent parameter. Then V , the rate of change of the expectation value V of the
dt
potential energy is
1 dk 2 k 1 dk 2 1
(a) x   xp  px  (b) x  p2
2 dt 2m 2 dt 2m
k 1 dk 2
(c) xp  px (d) x
2m 2 dt
Q38. In a two-state system, the transition rate of a particle from state 1 to state 2 is t12 , and the

transition rate from state 2 to state 1 is t21 . In the steady state, the probability of finding
the particle in state 1 is
t21 t12 t12t21 t12  t21
(a) (b) (c) (d)
t12  t21 t12  t21 t12  t21 t12  t21

                                                                                
Head office  Branch office 
 
fiziks, H.No. 23, G.F, Jia Sarai,  Anand Institute of Mathematics, 
 
Near IIT, Hauz Khas, New Delhi‐16  28‐B/6, Jia Sarai, Near IIT 
 
Phone: 011‐26865455/+91‐9871145498 Hauz Khas, New Delhi‐16 
                                                   
                                             Website: www.physicsbyfiziks.com                                                                                          
                                                           Email: fiziks.physics@gmail.com                                                                   8 
fiziks
Institute for NET/JRF, GATE, IIT‐JAM, JEST, TIFR and GRE in PHYSICAL SCIENCES 
 
Q39. A system of N non-interacting classical particles, each of mass m is in a two
dimensional harmonic potential of the form V  r     x 2  y 2  where  is a positive

 1 
constant. The canonical partition function of the system at temperature T is    :
 k BT 
N
   2   2N N N
 2m      2m 2 
(a)    (b)   (c)   (d)  2 
 2m        2 m    
Q40. The concentration of electrons, n and holes p , for an intrinsic semiconductor at a

 E  3
temperature T can be expressed as n  p  AT exp   g  , where Eg is the band 2

 2 k BT 
3
2
gap and A is a constant. If the mobility of both types of carries is proportional to T ,
then the log of the conductivity is a linear function of T 1 , with slope
Eg Eg  Eg  Eg
(a) (b) (c) (d)
 2k B  kB  2k B  kB

 Pa 4
Q41. The viscosity  of a liquid is given by Poiseuille’s formula  . Assume that l and
8lV
V can be measured very accurately, but the pressure P has an rms error of 1% and the
radius a has an independent rms error of 3% . The rms error of the viscosity is closest to
(a) 2% (b) 4% (c) 12% (d) 13%
Q42. A system of N distinguishable particles, each of which can be in one of the two energy
levels 0 and  , has a total energy n , where n is an integer. The entropy of the system is
proportional to

 N !  N! 
(d) ln 
 n ! N  n  ! 
(a) N ln n (b) n ln N (c) ln  
 n!   

                                                                                
Head office  Branch office 
 
fiziks, H.No. 23, G.F, Jia Sarai,  Anand Institute of Mathematics, 
 
Near IIT, Hauz Khas, New Delhi‐16  28‐B/6, Jia Sarai, Near IIT 
 
Phone: 011‐26865455/+91‐9871145498 Hauz Khas, New Delhi‐16 
                                                   
                                             Website: www.physicsbyfiziks.com                                                                                          
                                                           Email: fiziks.physics@gmail.com                                                                   9 
fiziks
Institute for NET/JRF, GATE, IIT‐JAM, JEST, TIFR and GRE in PHYSICAL SCIENCES 
 
Q43. The condition for the liquid and vapour phases of a fluid to be in equilibrium is given by
dP Q
the approximate equation  l (Clausius-Clayperon equation), where vvap is the
dT Tvvap

volume per particle in the vapour phase, and Ql is the latent heat, which may be taken to
be a constant. If the vapour obeys ideal gas law, which of the following plots is correct?

(a) ln P (b) ln P

O T O T

(c) ln P (d) ln P
O T O T

Q44. Consider the circuits shown in figures (a) and (b) below
2K 1K

10 K 10 K
10V 10V

10.7 V 5V

(a) (b)
If the transistors in Figures (a) and (b) have current gain   dc  of 100 and 10

respectively, then they operate in the


(a) active region and saturation region respectively
(b) saturation region and active region respectively
(c) saturation region in both cases
(d) active region in both cases

                                                                                
Head office  Branch office 
 
fiziks, H.No. 23, G.F, Jia Sarai,  Anand Institute of Mathematics, 
 
Near IIT, Hauz Khas, New Delhi‐16  28‐B/6, Jia Sarai, Near IIT 
 
Phone: 011‐26865455/+91‐9871145498 Hauz Khas, New Delhi‐16 
                                                   
                                             Website: www.physicsbyfiziks.com                                                                                          
                                                           Email: fiziks.physics@gmail.com                                                                   10 
fiziks
Institute for NET/JRF, GATE, IIT‐JAM, JEST, TIFR and GRE in PHYSICAL SCIENCES 
 
Q45. Which of the following circuits behaves as a controlled inverter?
(a)
i/ p
o/ p
control

(b)
i/ p

o/ p

control

(c)
i/ p

o/ p

control

(d)
i/ p

o/ p

control

                                                                                
Head office  Branch office 
 
fiziks, H.No. 23, G.F, Jia Sarai,  Anand Institute of Mathematics, 
 
Near IIT, Hauz Khas, New Delhi‐16  28‐B/6, Jia Sarai, Near IIT 
 
Phone: 011‐26865455/+91‐9871145498 Hauz Khas, New Delhi‐16 
                                                   
                                             Website: www.physicsbyfiziks.com                                                                                          
                                                           Email: fiziks.physics@gmail.com                                                                   11 
fiziks
Institute for NET/JRF, GATE, IIT‐JAM, JEST, TIFR and GRE in PHYSICAL SCIENCES 
 
PART – ‘C’
Q46. The rank-2 tensor xi x j , where xi are the Cartesian coordinates of the position vector in

three dimensions, has 6 independent elements. Under rotation, these 6 elements


decompose into irreducible sets (that is, the elements of each set transform only into
linear combinations of elements in that set) containing
(a) 4 and 2 elements
(b) 5 and 1 elements
(c) 3, 2 and 1 elements
(d) 4, 1 and 1 elements

2 f 2  f
2
Q47. Let f  x, t  be a solution of the wave equation  v in 1 -dimension. If at
t 2 x 2
f
t  0, f  x, 0   e  x and  x, 0   0 for all x , then f  x, t  for all future times t  0 is
2

t
described by

 x 2 v 2t 2 
(b) e x vt 
2
(a) e
1  x vt 2 3  x  vt 2 1   x vt 2  x  vt  
2
(c) e  e (d) e  e
4 4 2  
Q48. The differential cross-section for scattering by a target is given by
d
 ,    a 2  b2 cos2 
d
If N is the flux of the incoming particles, the number of particles scattered per unit time
is
4  1 
(a)
3
N a2  b2  (b) 4 N  a 2  b 2 
 6 

1 1   1 
(c) 4 N  a 2  b 2  (d) 4 N  a 2  b 2 
2 3   3 

                                                                                
Head office  Branch office 
 
fiziks, H.No. 23, G.F, Jia Sarai,  Anand Institute of Mathematics, 
 
Near IIT, Hauz Khas, New Delhi‐16  28‐B/6, Jia Sarai, Near IIT 
 
Phone: 011‐26865455/+91‐9871145498 Hauz Khas, New Delhi‐16 
                                                   
                                             Website: www.physicsbyfiziks.com                                                                                          
                                                           Email: fiziks.physics@gmail.com                                                                   12 
fiziks
Institute for NET/JRF, GATE, IIT‐JAM, JEST, TIFR and GRE in PHYSICAL SCIENCES 
 
Q49. Consider a rectangular wave guide with transverse dimensions 2 m 1 m driven with an

angular frequency   109 rad / s . Which transverse electric TE  modes will propagate

in this wave guide?


(a) TE10 , TE01 and TE20 (b) TE10 , TE11 and TE20

(c) TE01 , TE10 and TE11 (d) TE01 , TE10 and TE22

Q50. The electric and magnetic fields in the charge free region z  0 are given by
 
E  r , t   E0 e k1z cos  k2 x  t  ˆj
  E
E  r , t   0 e k1z  k1 sin  k2 x  t  iˆ  k2 cos  k2 x  t  kˆ 
  

where  , k1 and k2 are positive constants. The average energy flow in the x -direction is

E02 k2 2 k1z E02 k2


(a) e (b) e2 k1z
20 0

E02 k1 2 k1z 1
(c) e (d) c 0 E02 e2 k1 z
20 2
Q51. A uniform magnetic field in the positive z -direction passes through a circular wire loop
of radius 1 cm and resistance 1  lying in the xy -plane. The field strength is reduced
from 10 tesla to 9 tesla in1 s . The charge transferred across any point in the wire is
approximately
(a) 3.1 104 coulomb (b) 3.4  104 coulomb
(c) 4.2 104 coulomb (d) 5.2 104 coulomb
Q52. Three real variables a, b and c are each randomly chosen from a uniform probability

distribution in the interval  0,1 . The probability that a  b  2c is

3 2 1 1
(a) (b) (c) (d)
4 3 2 4

                                                                                
Head office  Branch office 
 
fiziks, H.No. 23, G.F, Jia Sarai,  Anand Institute of Mathematics, 
 
Near IIT, Hauz Khas, New Delhi‐16  28‐B/6, Jia Sarai, Near IIT 
 
Phone: 011‐26865455/+91‐9871145498 Hauz Khas, New Delhi‐16 
                                                   
                                             Website: www.physicsbyfiziks.com                                                                                          
                                                           Email: fiziks.physics@gmail.com                                                                   13 
fiziks
Institute for NET/JRF, GATE, IIT‐JAM, JEST, TIFR and GRE in PHYSICAL SCIENCES 
 
dy
Q53. Consider the differential equation  x 2  y with the initial condition y  2 at x  0 .
dx
Let y1 and y 1  be the solutions at x  1 obtained using Euler’s forward algorithm with
 
2

 y1  y 1  
1    
2
step size 1 and respectively. The value of   is
2  y 1  
 2 
1 1
(a) (b) 1 (c) (d) 1
2 2
Q54. Let q and p be the canonical coordinate and momentum of a dynamical system. Which
of the following transformations is canonical?
1 2 1 2 1 1
1. Q1  q and P1  p 2. Q2   p  q  and P2   p  q 
2 2 2 2
(a) neither 1 nor 2 (b) both 1 and 2
(c) only 1 (d) only 2
Q55. Which of the following figures is a schematic representation of the phase space
trajectories (i.e., cotours of constant energy) of a particle moving in a one-dimensional
1 2 1 4
potential V  x   x  x ? p
2 4
p

(a) (b)
x
x

p p

x x
(c) (d)

                                                                                
Head office  Branch office 
 
fiziks, H.No. 23, G.F, Jia Sarai,  Anand Institute of Mathematics, 
 
Near IIT, Hauz Khas, New Delhi‐16  28‐B/6, Jia Sarai, Near IIT 
 
Phone: 011‐26865455/+91‐9871145498 Hauz Khas, New Delhi‐16 
                                                   
                                             Website: www.physicsbyfiziks.com                                                                                          
                                                           Email: fiziks.physics@gmail.com                                                                   14 
fiziks
Institute for NET/JRF, GATE, IIT‐JAM, JEST, TIFR and GRE in PHYSICAL SCIENCES 
 
Q56. A rod of length L carries a total charge Q distributed uniformly. If this is observed in a
frame moving with a speed v along the rod, the charge per unit length (as measured by
the moving observer) is

Q  v2  Q v2
(a) 1   (b) 1 2
L  c2  L c

Q Q
(c) (d)
v 2  v2 
L 1 L 1  2 
c2  c 
1
Q57. A particle of mass m is in a potential V  m 2 x 2 , where  is a constant.
2
m  ipˆ  daˆ
Let aˆ   xˆ   . In the Heisenberg picture is given by
2  m  dt

(a)  â (b) i aˆ (c)  â † (d) i aˆ †


Q58. Two different sets of orthogonal basis vectors
 1   0    1 1 1  1  
  ,      ,  
 0   1    2 1 2  1 
are given for a two dimensional real vector space. The matrix representation of a linear
operator  in these bases are related by a unitary transformation. The unitary matrix may
be chosen to be
 0 1  0 1 1 1 1  1 1 0 
(a)   (b)   (c)   (d)  
1 0  1 0 2 1 1 2 1 1 
Q59. Consider three Ising spins at the vertices of a triangle which interact with each other with
a ferromagnetic Ising interaction of strength J . The partition function of the system at
 1 
temperature T is given by    :
 k BT 

(a) 2e3 J  6e   J (b) 2e 3 J  6e  J

(c) 2e3 J  6e 3 J  3e  J  3e   J (d)  2 cosh  J 


3

                                                                                
Head office  Branch office 
 
fiziks, H.No. 23, G.F, Jia Sarai,  Anand Institute of Mathematics, 
 
Near IIT, Hauz Khas, New Delhi‐16  28‐B/6, Jia Sarai, Near IIT 
 
Phone: 011‐26865455/+91‐9871145498 Hauz Khas, New Delhi‐16 
                                                   
                                             Website: www.physicsbyfiziks.com                                                                                          
                                                           Email: fiziks.physics@gmail.com                                                                   15 
fiziks
Institute for NET/JRF, GATE, IIT‐JAM, JEST, TIFR and GRE in PHYSICAL SCIENCES 
 
Q60. For the circuit and the input sinusoidal waveform shown in the correct waveform at the
output? 5K
 0.10

10 K  0.05

Vi 0.00 t

Vi 1K
 V0 0.05

0.10

(The time scales in all the plots are the same.)


(a) (b) 1.00
1.00
0.50
0.50
Vo 0.00 t
Vo 0.00 t
0.50
0.50
1.00
1.00

(c) (d)
1.00 1.00

0.50 0.50

0 0.00 0 0.00

0.50 0.50

1.00 1.00

                                                                                
Head office  Branch office 
 
fiziks, H.No. 23, G.F, Jia Sarai,  Anand Institute of Mathematics, 
 
Near IIT, Hauz Khas, New Delhi‐16  28‐B/6, Jia Sarai, Near IIT 
 
Phone: 011‐26865455/+91‐9871145498 Hauz Khas, New Delhi‐16 
                                                   
                                             Website: www.physicsbyfiziks.com                                                                                          
                                                           Email: fiziks.physics@gmail.com                                                                   16 
fiziks
Institute for NET/JRF, GATE, IIT‐JAM, JEST, TIFR and GRE in PHYSICAL SCIENCES 
 
Q61. In the circuit given below, the thermistor has a resistance 3 k  at 250 C . Its resistance

decreases by 150  per 0 C upon heating. The output voltage of the circuit at 300 C is
T

1V

1K
 Vout

(a) 37.5 V (b) 2.25 V (c) 2.25 V (d) 3.75 V


 
Q62. The Dirac Hamiltonian H  c . p   mc 2 for a free electron corresponds to the classical

relation E 2  p 2 c 2  m2 c 4 . The classical energy momentum relation of a particle of charge


q in a electromagnetic field is

 e    e 
(a) c . p  A. A   mc 2  e (b) c .  p  A    mc 2  e
c  c 
  e   e 
(c) c   . p  e  A    mc 2 (d) c .  p  A    mc 2  e
 c   c 
Q63. A particle of energy E scatters off a repulsive spherical potential
V for r  a
V r    0
0 for r  a

where V0 and a are positive constants. In the low energy limit, the total scattering cross-
2
 1  2m
section is   4 a  tanh ka  1 , where k 2  2 V0  E   0 . In the limit V0  
2

 ka  h
the ratio of  to the classical scattering cross-section off a sphere a radius a is
1
(a) 4 (b) 3 (c) 1 (d)
2

                                                                                
Head office  Branch office 
 
fiziks, H.No. 23, G.F, Jia Sarai,  Anand Institute of Mathematics, 
 
Near IIT, Hauz Khas, New Delhi‐16  28‐B/6, Jia Sarai, Near IIT 
 
Phone: 011‐26865455/+91‐9871145498 Hauz Khas, New Delhi‐16 
                                                   
                                             Website: www.physicsbyfiziks.com                                                                                          
                                                           Email: fiziks.physics@gmail.com                                                                   17 
fiziks
Institute for NET/JRF, GATE, IIT‐JAM, JEST, TIFR and GRE in PHYSICAL SCIENCES 
 
Q64. A large number N of Brownian particles in one dimension start their diffusive motion
from the origin at time t  0 . The diffusion coefficient is D . The number of particles
crossing a point at a distance L from the origin, per unit time, depends on L and time t
as
 L2 4 Dt
N NL
(a) e  4 Dt  (b) e L2
4 Dt 4 Dt
 L2 4 Dt
N  4 Dt  L2
(c) e (d) Ne
16 Dt 3

Q65. X -ray of wavelength   a is reflected from the 111 plane of a simple cubic lattice. If

the lattice constant is a , the corresponding Bragg angle (in radian) is


   
(a) (b) (c) (d)
6 4 3 8
Q66. The critical magnetic fields of a superconductor at temperatures 4 K and 8 K are
11 mA / m and 5.5 mA / m respectively. The transition temperature is approximately
(a) 8.4 K (b) 10.6 K (c) 12.9 K (d) 15.0 K

Q67. Of the following term symbols of the np 2 atomic configurations, 1 S0 , 3 P0 , 3 P1 , 3 P2 and


1
D2 which is the ground state?

(a) 3 P0 (b) 1 S0 (c) 3 P2 (d) 3 P1

Q68. The reaction 2


1 D 12 D 42 He   0 cannot proceed via strong interactions because it
violates the conservation of
(a) angular momentum (b) electric charge
(c) baryon number (d) isospin

                                                                                
Head office  Branch office 
 
fiziks, H.No. 23, G.F, Jia Sarai,  Anand Institute of Mathematics, 
 
Near IIT, Hauz Khas, New Delhi‐16  28‐B/6, Jia Sarai, Near IIT 
 
Phone: 011‐26865455/+91‐9871145498 Hauz Khas, New Delhi‐16 
                                                   
                                             Website: www.physicsbyfiziks.com                                                                                          
                                                           Email: fiziks.physics@gmail.com                                                                   18 
fiziks
Institute for NET/JRF, GATE, IIT‐JAM, JEST, TIFR and GRE in PHYSICAL SCIENCES 
 
Q69. Let us approximate the nuclear potential in the shell model by a three dimensional
isotropic harmonic oscillator. Since the lowest two energy levels have angular momenta
l  0 and l  1 respectively, which of the following two nuclei have magic numbers of
protons and neutrons?
(a) 42 He and 16
8 O (b) 12 D and 84 Be

(c) 42 He and 84 Be (d) 42 He and 12


6 C

Q70. An ideal Bose gas in d -dimensions obeys the dispersion relation  k  Ak s , where A  
and s are constants. For Bose-Einstein condensation to occur, the occupancy of excited
states
d s
 s
e
Ne  c        1
d
0 e

where c is a constant, should remain finite even for   0 . This can happen if
d 1 1 d 1
(a)  (b)  
s 4 4 s 2
d 1 d
(c) 1 (d)  1
s 2 s
Q71. For the logic circuit given below, the decimal count sequence and the modulus of the
circuit corresponding to ABCD are 1

J Flip
clock 3 bit ring D
counter Flop
K
A B C

MSB LSB
(a) 8  4  2  1  9  5 (mod 6 )
(b) 8  4  2  9  5  3 (mod 6 )
(c) 2  5  9  1  3 (mod 5 )
(d) 8  5  1  3  7 (mod 5 )

                                                                                
Head office  Branch office 
 
fiziks, H.No. 23, G.F, Jia Sarai,  Anand Institute of Mathematics, 
 
Near IIT, Hauz Khas, New Delhi‐16  28‐B/6, Jia Sarai, Near IIT 
 
Phone: 011‐26865455/+91‐9871145498 Hauz Khas, New Delhi‐16 
                                                   
                                             Website: www.physicsbyfiziks.com                                                                                          
                                                           Email: fiziks.physics@gmail.com                                                                   19 
fiziks
Institute for NET/JRF, GATE, IIT‐JAM, JEST, TIFR and GRE in PHYSICAL SCIENCES 
 
Q72. The low-energy electronic excitations in a two-dimensional sheet of grapheme is given

 
by E k  vk , where v is the velocity of the excitations. The density of states is

proportional to
3 1
(a) E (b) E 2 (c) E 2 (d) E 2
Q73. The charm quark assigned a charm quantum number C  1 . How should the Gellmann-
Nishijima formula for electric charge be modified for four flavors of quarks?
1 1
(a) I 3  B  S C (b) I 3  B  S  C
2 2
1 1
(c) I 3  B  S C (d) I 3  B  S  C
2 2
 1
Q74. A diatomic molecule has vibrational states with energies Ev    v   and rotational
 2

states with energies E j  Bj  j  1 , where v and j are non-negative integers. Consider

the transitions in which both the initial and final states are restricted to v  1 and j  2
and subject to the selection rules v  1 and j  1 . Then the largest allowed energy
of transition is
(a)   3B (b)   B
(c)   4B (d) 2  B
Q75. A He  Ne laser operates by using two energy levels of Ne separated by 2.26 eV .
Under steady state conditions of optical pumping, the equivalent temperature of the
system at which the ratio of the number of atoms in the upper state to that in the lower
1
state will be , is approximately (the Boltzmann constant k B  8.6  105 eV / K )
20
(a) 1010 K (b) 108 K (c) 106 K (d) 104 K

                                                                                
Head office  Branch office 
 
fiziks, H.No. 23, G.F, Jia Sarai,  Anand Institute of Mathematics, 
 
Near IIT, Hauz Khas, New Delhi‐16  28‐B/6, Jia Sarai, Near IIT 
 
Phone: 011‐26865455/+91‐9871145498 Hauz Khas, New Delhi‐16 
                                                   
                                             Website: www.physicsbyfiziks.com                                                                                          
                                                           Email: fiziks.physics@gmail.com                                                                   20 
fiziks
Institute for NET/JRF, GATE, IIT-JAM, JEST, TIFR and GRE in PHYSICAL SCIENCES

NET/JRF JUNE-14
PART ‘A’
Q1. The following diagram shows 2 perpendicularly inter-grown prismatic crystals (twins) of
identical shape and size. What is the volume of the object shown (units are arbitrary)?

10

2 2

(a) 60 (b) 65 (c) 72 (d) 80


Q2. Suppose in a box there are 20 red , 30 black, 40 blue and 50 white balls. What is the
minimum number of balls to be drawn, without replacement, so that you are certain about
getting 4 red, 5 black, 6 blue and 7 white balls?
(a) 140 (b) 97 (c) 104 (d) 124
Q3. In the growing years of a child, the height increases as the square root of the age while
the weight increases in direct proportion to the age. The ratio of the weight to the square
of the height in this phase of growth
(a) is constant
(b) reduces with age
(c) increases with age
(d) is constant only if the weight and height at birth are both zero.
Q4. Students in group A obtained the following marks: 40, 80, 70, 50, 60, 90, 30. Students in
group B obtained 40, 80, 35, 70, 85, 45, 50, 75, 60 marks. Define dispersion (D) =
dispersion
(maximum marks – minimum marks) and Relative dispresion RD   . Then,
mean
(a) RD of group A = RD of group B (b) RD of group A > RD of group B
(c) RD of group A < RD of group B (d) D of group A < D of group B

Head office Branch office


fiziks, H.No. 23, G.F, Jia Sarai, Anand Institute of Mathematics,
Near IIT, Hauz Khas, New Delhi-16 28-B/6, Jia Sarai, Near IIT
Phone: 011-26865455/+91-9871145498 Hauz Khas, New Delhi-16
Website: www.physicsbyfiziks.com
Email: fiziks.physics@gmail.com 1
fiziks
Institute for NET/JRF, GATE, IIT-JAM, JEST, TIFR and GRE in PHYSICAL SCIENCES

Q5. In 450 g of pure coffee powder 50 g of chicory is added. A person buys 100 g of this
mixture and adds 5 g of chicory to that. What would be the rounded-off percentage of
chicory in this final mixture?
(a) 10 (b) 5 (c) 14 (d) 15
Q6. The time gap between the two instants, one before and one after 12.00 noon, when the
angle between the hour hand and the minute hand is 66, is
(a) 12 min (b) 16 min (c) 18 min (d) 24 min
Q7. Suppose
2
xy   x  y 
2
x o y  x  y 
1
x * y  x  y 
x y  x y
+, – and  have their usual meanings. What is the value of
197o315  197315 197 * 315 ?
(a) 118 (b) 512 (c) 2 (d) 4

Q8. If A  B  24, B  C  32, C  D  48 then A  D


(a) cannot be found (b) is a perfect square
(c) is a perfect cube (d) is odd

Q9. If all horses are donkeys, some donkeys are monkeys, and some monkeys are men, then
which statement must be true?
(a) All donkeys are men (b) Some donkeys may be men
(c) Some horses are men (d) All horses are also monkeys

Q10. A rectangular area of sides 9 and 6 units is to be covered by square tiles of sides 1, 2, and
5 units. The minimum number of tiles needed for this is
(a) 3 (b) 11 (c) 12 (d) 15

Head office Branch office


fiziks, H.No. 23, G.F, Jia Sarai, Anand Institute of Mathematics,
Near IIT, Hauz Khas, New Delhi-16 28-B/6, Jia Sarai, Near IIT
Phone: 011-26865455/+91-9871145498 Hauz Khas, New Delhi-16
Website: www.physicsbyfiziks.com
Email: fiziks.physics@gmail.com 2
fiziks
Institute for NET/JRF, GATE, IIT-JAM, JEST, TIFR and GRE in PHYSICAL SCIENCES

Q11.  
Suppose n is a positive integer. Then n 2  n 2n  1
(a) may not be divisible by 2
(b) is always divisible by 2 but may not be divisible by 3
(c) is always divisible by 3 but may not be divisible by 6
(d) is always divisible by 6
Q12. There is a train of length 500 m, in which a man is standing at the rear end. At the instant
the rear end crosses a stationary observer on a platform, the man starts walking from the
rear to the front and the front to the rear of the train at a constant speed of 3 km/hr. the
speed of the train is 80 km/hr. The distance of the man from the observer at the end of 30
minutes is
(a) 41.5 km (b) 40.5 km (c) 40.0 km (d) 41.0 km
Q13. Three identical flat equilateral-triangular plates of side 5 cm each are placed together
such that they form a trapezium. The length of the longer of the two parallel sides of this
trapezium is

3
(a) 5 cm (b) 5 2 cm (c) 10 cm (d) 10 3 cm
4
Q14. An archer climbs to the top of a 10 m high building and aims at a bird atop a tree 17 m
away. The line of sight from the archer to the bird makes an angle of 45 to the horizontal.
What is the height of the tree?
(a) 17 m (b) 27 m (c) 37 m (d) 47 m
Q15. Consider a right-angled triangle ABC where AB = AC = 3. A rectangle APOQ is drawn
inside it, as shown, such that the height of the rectangle is twice
its width. The rectangle is moved horizontally by a distance 0.2 as C

shown schematically in the diagram (not to scale). What is the


area of  ABC Q O S
value of the ratio
area of  OST T
(a) 625 (b) 400

(c) 225 (d) 125


A P B

Head office Branch office


fiziks, H.No. 23, G.F, Jia Sarai, Anand Institute of Mathematics,
Near IIT, Hauz Khas, New Delhi-16 28-B/6, Jia Sarai, Near IIT
Phone: 011-26865455/+91-9871145498 Hauz Khas, New Delhi-16
Website: www.physicsbyfiziks.com
Email: fiziks.physics@gmail.com 3
fiziks
Institute for NET/JRF, GATE, IIT-JAM, JEST, TIFR and GRE in PHYSICAL SCIENCES

Q16. 80 gsm paper is cut into sheets of 200 mm  300 mm size and assembled in packets of
500 sheets. What will be the weight of a packet? (gsm = g/m2)
(a) 1.2 kg (b) 2.4 kg (c) 3.6 kg (d) 4.8 kg

Q17. Find the missing letter


A B C D
F I L O
K P U Z
P W D ?
(a) P (b) K (c) J (d) L

Q18. A merchant buys equal numbers of shirts and trousers and pays Rs 38000. if the cost of 3
shirts is Rs. 800 and that of a trouser is Rs. 1000, then how many shirts were bought?
(a) 60 (b) 30 (c) 15 (d) 10

Q19.  
Consider the set of numbers 171 , 17 2 , .....17 300 . How many of these numbers end with
the digit 3?
(a) 60 (b) 75 (c) 100 (d) 150

Q20. Find the missing number in the triangle,

7 1 8

90 13 ?

3 5 6 4 2 6

(a) 16 (b) 96 (c) 50 (d) 80

Head office Branch office


fiziks, H.No. 23, G.F, Jia Sarai, Anand Institute of Mathematics,
Near IIT, Hauz Khas, New Delhi-16 28-B/6, Jia Sarai, Near IIT
Phone: 011-26865455/+91-9871145498 Hauz Khas, New Delhi-16
Website: www.physicsbyfiziks.com
Email: fiziks.physics@gmail.com 4
fiziks
Institute for NET/JRF, GATE, IIT-JAM, JEST, TIFR and GRE in PHYSICAL SCIENCES

PART ‘B’

Q21. A time-dependent current I t   Ktzˆ (where K is a constant) is switched on at t  0 in
an infinite current-carrying wire. The magnetic vector potential at a perpendicular
distance a from the wire is given (for time t  a / c ) by
2 2 2
 K c t a ct  a 2  z 2 0 K ct
t
(a) zˆ 0 (b) zˆ  dz a
4 c  c 2t 2  a 2
dz
a 2  z 2 1/ 2 4  ct
2
 z2  1/ 2

ct c 2t 2  a 2
 K ct  a 2  z 2  K t
(c) zˆ 0  dz a (d) zˆ 0  dz
4 c  ct
2
z 
2 1/ 2 4  c 2t 2  a 2 a 2
 z2 1/ 2

Q22. A current i p flows through the primary coil of a transformer. The graph of i p t  as a

function of time t is shown in the figure below.


i p t 

1 2 3 t
Which of the following graphs represents the current i S in the secondary coil?

(a) (b)
i s t  i s t 

1 2 3 t

1 2 3 t

Head office Branch office


fiziks, H.No. 23, G.F, Jia Sarai, Anand Institute of Mathematics,
Near IIT, Hauz Khas, New Delhi-16 28-B/6, Jia Sarai, Near IIT
Phone: 011-26865455/+91-9871145498 Hauz Khas, New Delhi-16
Website: www.physicsbyfiziks.com
Email: fiziks.physics@gmail.com 5
fiziks
Institute for NET/JRF, GATE, IIT-JAM, JEST, TIFR and GRE in PHYSICAL SCIENCES

(c) i s t  (d) i s t 

1 2 3t

1 2 3t
Q23. If the electrostatic potential in spherical polar coordinates is
 r    0 e  r / r0
where  0 and r0 are constants, then the charge density at a distance r  r0 will be

 0 0 e   2e 0 0
(a) 2
(b) 0 2 0 (c)  0 20 (d) 
er0 2r0 er0 r02

Q24. If A  yziˆ  zxˆj  xykˆ and C is the circle of unit radius in the plane defined by z  1 ,

with the centre on the z - axis, then the value of the integral  A  d  is
C

 
(a) (b)  (c) (d) 0
2 4
Q25. The time period of a simple pendulum under the influence of the acceleration due to
gravity g is T . The bob is subjected to an additional acceleration of magnitude 3 g in
the horizontal direction. Assuming small oscillations, the mean position and time period
of oscillation, respectively, of the bob will be
(a) 0 o to the vertical and 3T (b) 30 o to the vertical and T / 2

(c) 60o to the vertical and T / 2 (d) 0o to the vertical and T / 3


Q26. Consider the differential equation
d2x dx
2
2 x 0
dt dt
with the initial conditions x0   0 and x 0   1 . The solution xt  attains its maximum
value when t is
(a) 1/2 (b) 1 (c) 2 (d) 

Head office Branch office


fiziks, H.No. 23, G.F, Jia Sarai, Anand Institute of Mathematics,
Near IIT, Hauz Khas, New Delhi-16 28-B/6, Jia Sarai, Near IIT
Phone: 011-26865455/+91-9871145498 Hauz Khas, New Delhi-16
Website: www.physicsbyfiziks.com
Email: fiziks.physics@gmail.com 6
fiziks
Institute for NET/JRF, GATE, IIT-JAM, JEST, TIFR and GRE in PHYSICAL SCIENCES

Q27. Consider the matrix

 0 2i 3i 
 
M    2i 0 6i 
  3i  6i 0 
 
The eigenvalues of M are
(a)  5,  2, 7 (b)  7, 0, 7 (c)  4i, 2i, 2i (d) 2, 3, 6

1
Q28. If C is the contour defined by z  , the value of the integral
2
dz
 C sin 2 z

is
(a)  (b) 2 i (c) 0 (d)  i

Q29. Given



n0

Pn  x t n  1  2 xt  t 2 
1 / 2
, for t  1 , the value of P5  1 is

(a) 0.26 (b) 1 (c) 0.5 (d) –1

Q30. Consider an electromagnetic wave at the interface between two homogenous dielectric
media of dielectric constants  1 and  2 . Assuming  2   1 and no charges on the
 
surface, the electric field vector E and the displacement vector D in the two media
satisfy the following inequalities
       
(a) E 2  E1 and D2  D1 (b) E 2  E1 and D2  D1
       
(c) E 2  E1 and D2  D1 (d) E 2  E1 and D2  D1

Head office Branch office


fiziks, H.No. 23, G.F, Jia Sarai, Anand Institute of Mathematics,
Near IIT, Hauz Khas, New Delhi-16 28-B/6, Jia Sarai, Near IIT
Phone: 011-26865455/+91-9871145498 Hauz Khas, New Delhi-16
Website: www.physicsbyfiziks.com
Email: fiziks.physics@gmail.com 7
fiziks
Institute for NET/JRF, GATE, IIT-JAM, JEST, TIFR and GRE in PHYSICAL SCIENCES

Q31. Consider a system of two non-interacting identical fermions, each of mass m in an


infinite square well potential of width a . (Take the potential inside the well to be zero
and ignore spin). The composite wavefunction for the system with total energy
5 2  2
E is
2ma 2
2   x1   2x 2   2x1   x 2 
(a) sin   sin    sin   sin  
a  a   a   a   a 

2   x1   2x 2   2x1   x 2 


(b) sin   sin    sin   sin  
a  a   a   a   a 

2   x1   3x 2   3x1   x 2 


(c) sin   sin    sin   sin  
a   a   2a   2 a   a 

2   x1   x 2   x   x 
(d) sin   cos   sin 2  cos 2 
a  a   a   a   a 
1
Q32. A particle of mass m in the potential V  x, y  
2
 
m 2 4 x 2  y 2 , is in an eigenstate of

5
energy E   . The corresponding un-normalized eigen function is
2
 m   m 
(a) y exp  
2x 2  y 2   (b) x exp  
2x 2  y 2  
 2   2 
 m 2   m 2 
(c) y exp  
x  y2   (d) xy exp  x  y2  
 2   2 
1 
Q33. A particle of mass m and coordinate q has the Lagrangian L  mq 2  qq 2 where 
2 2
is a constant. The Hamiltonian for the system is given by
p 2 qp 2 p2
(a)  (b)
2m 2m 2 2m  q 

p2 qp 2 pq
(c)  (d)
2m 2m  q 2 2

Head office Branch office


fiziks, H.No. 23, G.F, Jia Sarai, Anand Institute of Mathematics,
Near IIT, Hauz Khas, New Delhi-16 28-B/6, Jia Sarai, Near IIT
Phone: 011-26865455/+91-9871145498 Hauz Khas, New Delhi-16
Website: www.physicsbyfiziks.com
Email: fiziks.physics@gmail.com 8
fiziks
Institute for NET/JRF, GATE, IIT-JAM, JEST, TIFR and GRE in PHYSICAL SCIENCES

Q34. A light source is switched on and off at a constant frequency f . An observer moving with
a velocity u with respect to the light source will observe the frequency of the switching
to be
1 1 / 2 1/ 2
 u2   u2   u2   u2 
(a) f 1  2  (b) f 1  2  (c) f 1  2  (d) f 1  2 
 c   c   c   c 

Q35. A charged particle is at a distance d from an infinite conducting plane maintained at zero
potential. When released from rest, the particle reaches a speed u at a distance d / 2 from
the plane. At what distance from the plane will the particle reach the speed 2u ?
(a) d / 6 (b) d / 3 (c) d / 4 (d) d / 5

Q36. A particle of mass m in three dimensions is in the potential


0 ra
V r   
 r a
Its ground state energy is
 22  2 2 3 2  2 9 2  2
(a) (b) (c) (d)
2ma 2 ma 2 2ma 2 2ma 2

  1
Q37. Given that pˆ r  i   , the uncertainty p r in the ground state.
 r r 
1
 0 r   e  r / a0
3
a 0

of the hydrogen atom is

 2  2
(a) (b) (c) (d)
a0 a0 2a 0 a0

Head office Branch office


fiziks, H.No. 23, G.F, Jia Sarai, Anand Institute of Mathematics,
Near IIT, Hauz Khas, New Delhi-16 28-B/6, Jia Sarai, Near IIT
Phone: 011-26865455/+91-9871145498 Hauz Khas, New Delhi-16
Website: www.physicsbyfiziks.com
Email: fiziks.physics@gmail.com 9
fiziks
Institute for NET/JRF, GATE, IIT-JAM, JEST, TIFR and GRE in PHYSICAL SCIENCES

Q38. Which of the graphs below gives the correct qualitative behaviour of the energy density
E r   of blackbody radiation of wavelength  at two temperatures T1 and T2 T1  T2  ?
(a) (b)
T2
T2
Er  
Er  
T1

 

(c) (d)
T2
T2
Er   Er  
T1
T1

 
Q39. One gram of salt is dissolved in water that is filled to a height of 5 cm in a beaker of
diameter 10 cm. The accuracy of length measurement is 0.01 cm while that of mass
measurement is 0.01 mg. When measuring the concentration C , the fractional error
C / C is
(a) 0.8% (b) 0.14% (c) 0.5% (d) 0.28%

Q40. The inner shield of a triaxial conductor is driven by an (ideal) op-amp follower circuit as
shown. The effective capacitance between the signal-carrying conductor and ground is

Signal


(a) unaffected (b) doubled (c) halved (d) made zero

Head office Branch office


fiziks, H.No. 23, G.F, Jia Sarai, Anand Institute of Mathematics,
Near IIT, Hauz Khas, New Delhi-16 28-B/6, Jia Sarai, Near IIT
Phone: 011-26865455/+91-9871145498 Hauz Khas, New Delhi-16
Website: www.physicsbyfiziks.com
Email: fiziks.physics@gmail.com 10
fiziks
Institute for NET/JRF, GATE, IIT-JAM, JEST, TIFR and GRE in PHYSICAL SCIENCES

Q41. An op-amp based voltage follower


(a) is useful for converting a low impedance source into a high impedance source.
(b) is useful for converting a high impedance source into a low impedance source.
(c) has infinitely high closed loop output impedance
(d) has infinitely high closed loop gain
Q42. A system can have three energy levels: E  0,   . The level E  0 is doubly
degenerate, while the others are non-degenerate. The average energy at inverse
temperature  is

 e   e   
(a)   tanh   (b)
1  e   e   
  
(c) zero (d)   tanh 
 2 
Q43. The free energy F of a system depends on a thermodynamic variable  as

F  a 2  b 6
with a, b  0 . The value of  , when the system is in thermodynamic equilibrium, is
1/ 4 1/ 4 1/ 4
(a) zero (b)  a / 6b  (c)  a / 3b  (d)  a / b 
Q44. For a particular thermodynamic system the entropy S is related to the internal energy U
and volume V by
S  cU 3 / 4V 1 / 4
where c is a constant. The Gibbs potential G  U  TS  pV for this system is
3 pU cU US
(a) (b) (c) zero (d)
4T 3 4V
Q45. An RC network produces a phase-shift of 30 o . How many such RC networks should be
cascaded together and connected to a Common Emitter amplifier so that the final circuit
behaves as an oscillator?
(a) 6 (b) 12 (c) 9 (d) 3

Head office Branch office


fiziks, H.No. 23, G.F, Jia Sarai, Anand Institute of Mathematics,
Near IIT, Hauz Khas, New Delhi-16 28-B/6, Jia Sarai, Near IIT
Phone: 011-26865455/+91-9871145498 Hauz Khas, New Delhi-16
Website: www.physicsbyfiziks.com
Email: fiziks.physics@gmail.com 11
fiziks
Institute for NET/JRF, GATE, IIT-JAM, JEST, TIFR and GRE in PHYSICAL SCIENCES

PART ‘C’
Q46. For the logic circuit shown in the below

X
B

A simplified equivalent circuit is A


(a) A (b) B
B X X
C
C

A A
(c) (d)
B B
X X
C C
Q47. The coordinates and momenta xi , pi i  1, 2, 3 of a particle satisfy the canonical Poisson

bracket relations xi , p j    ij . If C1  x2 p3  x3 p2 and C 2  x1 p 2  x 2 p1 are constants of

motion, and if C 3  C1 , C 2   x1 p3  x3 p1 , then

(a) C 2 , C 3   C1 and C 3 , C1   C 2

(b) C 2 , C 3   C1 and C 3 , C1   C 2

(c) C 2 , C 3   C1 and C 3 , C1   C 2

(d) C 2 , C 3   C1 and C 3 , C1   C 2

Head office Branch office


fiziks, H.No. 23, G.F, Jia Sarai, Anand Institute of Mathematics,
Near IIT, Hauz Khas, New Delhi-16 28-B/6, Jia Sarai, Near IIT
Phone: 011-26865455/+91-9871145498 Hauz Khas, New Delhi-16
Website: www.physicsbyfiziks.com
Email: fiziks.physics@gmail.com 12
fiziks
Institute for NET/JRF, GATE, IIT-JAM, JEST, TIFR and GRE in PHYSICAL SCIENCES

Q48. The recently-discovered Higgs boson at the LHC experiment has a decay mode into a
photon and a Z boson. If the rest masses of the Higgs and Z boson are 125 GeV/c 2 and

90 GeV/c 2 respectively, and the decaying Higgs particle is at rest, the energy of the
photon will approximately be
(a) 35 3 GeV (b) 35 GeV (c) 30 GeV (d) 15 GeV
1
Q49. Let y   x1  x2    , where x1 and x2 are independent and identically distributed
2
Gaussian random variables of mean  and standard deviation  . Then y 4 /  4 is

(a) 1 (b) 3/4 (c) 1/2 (d) 1/4


Q50. The graph of a real periodic function f  x  for the range  ,  is shown below
f x 

Which of the following graphs represents the real part of its Fourier transform?
(a) (b)
Re f k  Re f k 

k k

(c) (d)
Re f k  Re f k 

k k

Head office Branch office


fiziks, H.No. 23, G.F, Jia Sarai, Anand Institute of Mathematics,
Near IIT, Hauz Khas, New Delhi-16 28-B/6, Jia Sarai, Near IIT
Phone: 011-26865455/+91-9871145498 Hauz Khas, New Delhi-16
Website: www.physicsbyfiziks.com
Email: fiziks.physics@gmail.com 13
fiziks
Institute for NET/JRF, GATE, IIT-JAM, JEST, TIFR and GRE in PHYSICAL SCIENCES

Q51. The matrices

0  1 0  0 0 1 
A  1 0 0, B  0 0 0 
0 0 0 0 0 0 

0 0 0 
and C  0 0 1
0 0 0

satisfy the commutation relations


(a)  A, B  B  C , B, C   0, C , A  B  C
(b)  A, B  C , B, C   A, C , A  B
(c)  A, B  B, B, C   0, C , A  A
(d)  A, B  C , B, C   0, C , A  B
Q52. The time evolution of a one-dimensional dynamical system is described by
dx
dt

 x  1 x 2  b 2 . 
If this has one stable and two unstable fixed points, then the parameter b satisfies
(a) 0  b  1 (b) b  1 (c) b  1 (d) b  2
Q53. A canonical transformation relates the old coordinates q, p  to the new ones Q, P  by

the relations Q  q 2 and P  p / 2q . The corresponding time independent generating


function is
(a) P / q 2 (b) q 2 P (c) q 2 / P (d) qP 2
1
Q54. The integral  x dx is to be evaluated up to 3 decimal places using Simpson’s 3-point
0

rule. If the interval 0, 1 is divided into 4 equal parts, the correct result is
(a) 0.683 (b) 0.667 (c) 0.657 (d) 0.638
Q55. The pressure of a nonrelativistic free Fermi gas in three-dimensions depends, at T  0 ,
on the density of fermions n as
(a) n 5 / 3 (b) n 1 / 3 (c) n 2 / 3 (d) n 4 / 3

Head office Branch office


fiziks, H.No. 23, G.F, Jia Sarai, Anand Institute of Mathematics,
Near IIT, Hauz Khas, New Delhi-16 28-B/6, Jia Sarai, Near IIT
Phone: 011-26865455/+91-9871145498 Hauz Khas, New Delhi-16
Website: www.physicsbyfiziks.com
Email: fiziks.physics@gmail.com 14
fiziks
Institute for NET/JRF, GATE, IIT-JAM, JEST, TIFR and GRE in PHYSICAL SCIENCES

Q56. Consider an electron in b.c.c. lattice with lattice constant a . A single particle
 
wavefunction that satisfies the Bloch theorem will have the form f r  exp ik .r , with  

f r  being

 2   2   2 
(a) 1  cos  x  y  z   cos  x  y  z   cos  x  y  z 
 a   a   a 
 2   2   2 
(b) 1  cos x  y   cos  y  z   cos z  x 
 a   a   a 
     
(c) 1  cos  x  y   cos  y  z   cos  z  x 
a  a  a 
     
(d) 1  cos  x  y  z   cos  x  y  z   cos  x  y  z 
a  a  a 
Q57. A charge  e  is placed in vacuum at the point d ,0,0 , where d  0 . The region x  0

d 
is filled uniformly with a metal. The electric field at the point  ,0,0  is
2 
10e 10e
(a)  1, 0, 0 (b) 1, 0, 0
9 0 d 2 9 0 d 2
e e
(c) 1, 0, 0 (d)  1, 0, 0
 0 d 2  0 d 2
Q58. A beam of light of frequency  is reflected from a dielectric-metal interface at normal
incidence. The refractive index of the dielectric medium is n and that of the metal is
n 2  n1  i  . If the beam is polarised parallel to the interface, then the phase change
experienced by the light upon reflection is
(a) tan 2 /   (b) tan 1 1 /   (c) tan 1 2 /   (d) tan 1 2  

Q59. A permanently deformed even-even nucleus with J p  2  has rotational energy 93 keV.
The energy of the next excited state is
(a) 372 keV (b) 310 keV (c) 273 keV (d) 186 keV

Head office Branch office


fiziks, H.No. 23, G.F, Jia Sarai, Anand Institute of Mathematics,
Near IIT, Hauz Khas, New Delhi-16 28-B/6, Jia Sarai, Near IIT
Phone: 011-26865455/+91-9871145498 Hauz Khas, New Delhi-16
Website: www.physicsbyfiziks.com
Email: fiziks.physics@gmail.com 15
fiziks
Institute for NET/JRF, GATE, IIT-JAM, JEST, TIFR and GRE in PHYSICAL SCIENCES

Q60. A thin, infinitely long solenoid placed along the z - axis contains a magnetic flux  .
Which of the following vector potentials corresponds to the magnetic field at an arbitrary
point x, y, z  ?

  y  x 
(a) Ax , Ay , Az     2 2
, 2 2
,0 
 2 x  y 2 x  y 

  y  x 
(b) Ax , Ay , Az     2 2 2
, 2 2 2
,0 
 2 x  y  z 2 x  y  z 

  x y  x y 
(c) Ax , Ay , Az     2 2
, 2 2
,0 
 2 x  y 2 x  y 

  x  y 
(d) Ax , Ay , Az     2 2
, 2 2
,0 
 2 x  y 2 x  y 
Q61. The ground state eigenfunction for the potential V  x     x  where  x  is the delta

function, is given by  x   Ae  x
, where A and   0 are constants. If a perturbation

H   bx 2 is applied, the first order correction to the energy of the ground state will be
b b 2b b
(a) (b) (c) (d)
2 2
2 2 2 2
Q62. An electron is in the ground state of a hydrogen atom. The probability that it is within the
Bohr radius is approximately equal to
(a) 0.60 (b) 0.90 (c) 0.16 (d) 0.32
0 0xa
Q63. A particle in the infinite square well V x   
 otherwise

 3  x 
 A sin   0 xa
is prepared in a state with the wavefunction  x    a 
0 otherwise

The expectation value of the energy of the particle is
5 2 2 9 2 2 9 2 2  2 2
(a) (b) (c) (d)
2ma 2 2ma 2 10ma 2 2ma 2

Head office Branch office


fiziks, H.No. 23, G.F, Jia Sarai, Anand Institute of Mathematics,
Near IIT, Hauz Khas, New Delhi-16 28-B/6, Jia Sarai, Near IIT
Phone: 011-26865455/+91-9871145498 Hauz Khas, New Delhi-16
Website: www.physicsbyfiziks.com
Email: fiziks.physics@gmail.com 16
fiziks
Institute for NET/JRF, GATE, IIT-JAM, JEST, TIFR and GRE in PHYSICAL SCIENCES

Q64. The scattering amplitude f   for the potential V r   e   r , where  and  are
positive constants, is given, in the Born approximation, by
 2k 2
(in the following b  2k sin and E  )
2 2m
4 m 4m
(a)  (b) 
 b 
2
 2

2 2  b b2   2
2 2
 
4m 4m
(c)  (d) 
2 b2   2 
 2 b2   2 3

Q65. In one dimension, a random walker takes a step with equal probability to the left or right.
What is the probability that the walker returns to the starting point after 4 steps?
(a) 3/8 (b) 5/16 (c) 1/4 (d) 1/16

Q66. The average local internal magnetic field acting on an Ising spin is H int  M , where M
is the magnetization and  is a positive constant. At a temperature T sufficiently close
to (and above) the critical temperature Tc , the magnetic susceptibility at zero external

field is proportional to ( k B is the Boltzmann constant)


1 1
(a) k B T   (b) k B T    (c) k B T    (d) tanh k B T   

Q67. An electromagnetically-shielded room is designed so that at a frequency   10 7 rad/s


the intensity of the external radiation that penetrates the room is 1% of the incident
1 1
radiation. If    10 6 m  is the conductivity of the shielding material, its
2
minimum thickness should be (given that ln 10  2.3 )
(a) 4.60 mm (b) 2.30 mm (c) 0.23 mm (d) 0.46 mm

Head office Branch office


fiziks, H.No. 23, G.F, Jia Sarai, Anand Institute of Mathematics,
Near IIT, Hauz Khas, New Delhi-16 28-B/6, Jia Sarai, Near IIT
Phone: 011-26865455/+91-9871145498 Hauz Khas, New Delhi-16
Website: www.physicsbyfiziks.com
Email: fiziks.physics@gmail.com 17
fiziks
Institute for NET/JRF, GATE, IIT-JAM, JEST, TIFR and GRE in PHYSICAL SCIENCES

Q68. The following data is obtained in an experiment that measures the viscosity  as a
function of molecular weight M for a set of polymers,
M Da   kPa  s 
990 0.28  0.03
5032 30  2
10191 250  10
19825 2000  200
The relation that best describes the dependence of  on M is

(a)  ~ M 4 / 9 (b)  ~ M 3 / 2 (c)  ~ M 2 (d)  ~ M 3

Q69. In a classical model, a scalar (spin-0) meson consists of a quark and an antiquark bound
by a potential
b
V r   ar 
r
where a  200 MeV fm -1 and b  100 MeV fm . If the masses of the quark and antiquark
are negligible, the mass of the meson can be estimated as approximately
(a) 141 MeV/c 2 (b) 283 MeV/c 2 (c) 353 MeV/c 2 (d) 425 MeV/c 2

Q70. A spectral line due to a transition from an electronic state p to an s state splits into three
Zeeman lines in the presence of a strong magnetic field. At intermediate field strengths
the number of spectral lines is
(a) 10 (b) 3 (c) 6 (d) 9

Q71. A double slit interference experiment uses a laser emitting light of two adjacent
frequencies v1 and v2 v1  v 2  . The minimum path difference between the interfering
beams for which the interference pattern disappears is
c c c c
(a) (b) (c) (d)
v2  v1 v2  v1 2v 2  v1  2v 2  v1 

Head office Branch office


fiziks, H.No. 23, G.F, Jia Sarai, Anand Institute of Mathematics,
Near IIT, Hauz Khas, New Delhi-16 28-B/6, Jia Sarai, Near IIT
Phone: 011-26865455/+91-9871145498 Hauz Khas, New Delhi-16
Website: www.physicsbyfiziks.com
Email: fiziks.physics@gmail.com 18
fiziks
Institute for NET/JRF, GATE, IIT-JAM, JEST, TIFR and GRE in PHYSICAL SCIENCES

Q72. The van der Waals’ equation of state for a gas is given by
 a 
 P  2 V  b   RT
 V 
where P, V and T represent the pressure, volume and temperature respectively, and a
and b are constant parameters. At the critical point, where all the roots of the above
cubic equation are degenerate, the volume is given by
a a 8a
(a) (b) (c) (d) 3b
9b 27b 2 27bR

Q73. How much does the total angular momentum quantum number J change in the transition
 
of Cr 3d 6 atom as it ionize to Cr 2  3d 4 ?  
(a) Increases by 2 (b) Decreases by 2 (c) Decreases by 4 (d) Does not change

Q74. The dispersion relation for electrons in an f.c.c. crystal is given, in the tight binding
approximation, by
 k a kya kya k a
 k   4 0 cos x cos  cos cos z 
 2 2 2 2 

where a is the lattice constant and  0 is a constant with the dimension of energy. The x -

 
component of the velocity of the electron at  , 0, 0  is
a 
(a)  2 0 a /  (b) 2 0 a /  (c)  4 0 a /  (d) 4 0 a / 

Q75. The function  x, y, z , t   cos z  vt   Resin  x  iy  satisfies the equation

1  2   2 2 2   1 2 2   2 2 
(a)      (b)  2 2  2    2  2 
v 2 t 2  x 2 y 2 z 2   v t z   x y 

 1 2 2   2 2   2 1 2   2 2 
(c)  2 2  2    2  2  (d)  2  2 2    2  2 
 v t z   x y   z v t   x y 

Head office Branch office


fiziks, H.No. 23, G.F, Jia Sarai, Anand Institute of Mathematics,
Near IIT, Hauz Khas, New Delhi-16 28-B/6, Jia Sarai, Near IIT
Phone: 011-26865455/+91-9871145498 Hauz Khas, New Delhi-16
Website: www.physicsbyfiziks.com
Email: fiziks.physics@gmail.com 19
fiziks
Institute for NET/JRF, GATE, IIT-JAM, JEST, TIFR and GRE in PHYSICAL SCIENCES

NET PHYSICS JUNE-2013


PART-A
Q1. There is an equilateral triangle in the XY plane with its centre at the origin. The distance
of its sides from the origin is 3.5 cm. The area of its circumcircle in cm2 is
(a) 38.5 (b) 49 (c) 63.65 (d) 154

Q2. A sphere of iron of radius R/2 fixed to one end of a string was lowered into water in a
cylindrical container of base radius R to keep exactly half the sphere dipped. The rise in
the level of water in the container will be

(a) R/3 (b) R/4 (c) R/8 (d) R/12

Q3. A crystal grows by stacking of unit cells of 10 × 20 × 5 nm size as shown in the diagram
given below. How many unit cells will make a crystal of 1 cm3 volume?
5 nm

20 nm
10 nm
Unit Cell (not to scale)
5 nm

10 nm 20 nm
Crystal (not to scale)

(a) 10 6 (b) 109 (c) 1012 (d) 1018

Head office Branch office


fiziks, H.No. 23, G.F, Jia Sarai, Anand Institute of Mathematics,
Near IIT, Hauz Khas, New Delhi-16 28-B/6, Jia Sarai, Near IIT
Phone: 011-26865455/+91-9871145498 Hauz Khas, New Delhi-16
Website: www.physicsbyfiziks.com
Email: fiziks.physics@gmail.com 1
fiziks
Institute for NET/JRF, GATE, IIT-JAM, JEST, TIFR and GRE in PHYSICAL SCIENCES

1 1 1
Q4. What is the value of    .... to  ?
1 2 2  3 3  4
(a) 2 / 3 (b) 1 (c) 2 (d) 

Q5. A solid cylinder of basal area A was held dipped in water in a cylindrical vessel of basal
area 2 A vertically such that a length h of the cylinder is immersed. The lower tip of the
cylinder is at a height h from the base of the vessel. What will be the height of water in
the vessel when the cylinder is taken out?

h
h
3 4 5
(a) 2 h (b) h (c) h (d) h
2 3 4

Q6. Of all the triangles that can be inscribed in a semicircle of radius R with the diameter as
one side, the biggest one has the area
(a) R 2 (b) R 2 2 (c) R 2 3 (d) 2R 2

Q7. Choose the largest number:


(a) 2 500 (b) 3 400 (c) 4 300 (d) 5 200

Q8. A daily sheet calendar of the year 2013 contains sheets of 10  10 cm size. All the sheets
of the calendar are spread over the floor of a room of 5m × 7.3 m size. What percentage
of the floor will be covered by these sheets?
(a) 0.1 (b) 1 (c) 10 (d) 100

Head office Branch office


fiziks, H.No. 23, G.F, Jia Sarai, Anand Institute of Mathematics,
Near IIT, Hauz Khas, New Delhi-16 28-B/6, Jia Sarai, Near IIT
Phone: 011-26865455/+91-9871145498 Hauz Khas, New Delhi-16
Website: www.physicsbyfiziks.com
Email: fiziks.physics@gmail.com 2
fiziks
Institute for NET/JRF, GATE, IIT-JAM, JEST, TIFR and GRE in PHYSICAL SCIENCES

Q9. How many rectangles (which are not squares) are there in the following figure?
(a) 56
(b) 70
(c) 86
(d) 100
Q10. Define a  b  lcma, b   gcd a, b  and a  b  a b  b a . What is the value of
1  2  3  4? Here lcm = least common multiple and gcd = greatest common
divisor.
(a) 145 (b) 286 (c) 436 (d) 572

Q11. A square pyramid is to be made using a wire such that only one strand of wire is used for
each edge. What is the minimum number of times that the wire has to be cut in order to
make the pyramid?
(a) 3 (b) 7 (c) 2 (d) 1

Q12. A crow is flying along a horizontal circle of radius R at a height R above the horizontal
ground. Each of a number of men on the ground found that the angular height of the crow
 
was a fixed angle   45o when it was closest to him. Then all these men must be on a
circle on the ground with a radius
(a) R  R sin  (b) R  R cos (c) R  R tan  (d) R  R cot 

Q13. How many pairs of positive integers have gcd 20 and lcm 600?
(gcd = greatest common divisor; lcm = least common multiple)
(a) 4 (b) 0 (c) 1 (d) 7

Head office Branch office


fiziks, H.No. 23, G.F, Jia Sarai, Anand Institute of Mathematics,
Near IIT, Hauz Khas, New Delhi-16 28-B/6, Jia Sarai, Near IIT
Phone: 011-26865455/+91-9871145498 Hauz Khas, New Delhi-16
Website: www.physicsbyfiziks.com
Email: fiziks.physics@gmail.com 3
fiziks
Institute for NET/JRF, GATE, IIT-JAM, JEST, TIFR and GRE in PHYSICAL SCIENCES

Q14. During an evening party, when Ms. Black, Ms. Brown and Ms. White met, Ms. Brown
remarked, “it is interesting that our dresses are white, black or brown, but for each of us
the name does not match the colour of the dress!”. Ms. White replied, “But your white
dress does not suit you!”. Pick the correct answer
(a) Ms. White’s dress was brown (b) Ms. Black’s dress was white
(c) Ms. White’s dress was black (d) Ms. Black’s dress was black

Q15. Two integers are picked at random from the first 15 positive integers without
replacement. What is the probability that the sum of the two numbers is 20?
3 1 1 1
(a) (b) (c) (d)
4 21 105 20
Q16. Identify the next figure in the sequence

(a) (b) (c) (d)

Q17. In a customer survey conducted during Monday to Friday, of the customers who asked
for child care facilities in super markets, 23% were men and the rest, women. Among
them 19.9% of the women and 8.8% of the men were willing to pay for the facilities.
A. What is the ratio of the men to women customers who wanted child care facilities?
B. If the survey had been conducted during the weekend instead, how will the result
change?
With the above data,
(a) Only A can be answered (b) Only B can be answered
(c) Both A and B can be answered (d) Neither A nor B can be answered

Head office Branch office


fiziks, H.No. 23, G.F, Jia Sarai, Anand Institute of Mathematics,
Near IIT, Hauz Khas, New Delhi-16 28-B/6, Jia Sarai, Near IIT
Phone: 011-26865455/+91-9871145498 Hauz Khas, New Delhi-16
Website: www.physicsbyfiziks.com
Email: fiziks.physics@gmail.com 4
fiziks
Institute for NET/JRF, GATE, IIT-JAM, JEST, TIFR and GRE in PHYSICAL SCIENCES

Q18. The map given below shows contour lines which connect points of equal ground surface
elevation in a region. Inverted ‘V’ shaped portions of contour lines represent a valley
along which a river flows. What is the downstream direction of the river?

N
200
100
100
SCALE  1 : 5000

(a) North (b) South (c) East (d) West

Q19. During a summer vacation, of 20 friends from a hostel, each wrote a letter to each of all
others. The total number of letters written was
(a) 20 (b) 400 (c) 200 (d) 380

Q20. A person has to cross a square field by going from A to C. The person is only allowed to
move towards the east or towards the north or use a combination of these movements.
The total distance traveled by the person
(a) depends on the length of each step D C
NORTH
(b) depends on the total number of steps
(c) is different for different paths   EAST

(d) is the same for all paths

Head office Branch office


fiziks, H.No. 23, G.F, Jia Sarai, Anand Institute of Mathematics,
Near IIT, Hauz Khas, New Delhi-16 28-B/6, Jia Sarai, Near IIT
Phone: 011-26865455/+91-9871145498 Hauz Khas, New Delhi-16
Website: www.physicsbyfiziks.com
Email: fiziks.physics@gmail.com 5
fiziks
Institute for NET/JRF, GATE, IIT-JAM, JEST, TIFR and GRE in PHYSICAL SCIENCES

PART-B

Q21. Two identical bosons of mass m are placed in a one-dimensional potential


1
V x   m 2 x 2 . The bosons interact via a weak potential,
2

V12  V0 exp  mx1  x 2  / 4
2

where x1 and x2 denote coordinates of the particles. Given that the ground state
1
mx 2
 m  4 
wavefunction of the harmonic oscillator is 0  x     e 2
. The ground state
  
energy of the two-boson system, to the first order in V0 , is

V0 
(a)   2V0 (b)  

1

  2  
(c)   V0 1   (d)   V0 1  
   

Q22. Ten grams of ice at 0°C is added to a beaker containing 30 grams of water at 25°C. What
is the final temperature of the system when it comes to thermal equilibrium? (The
specific heat of water is 1 cal/gm/°C and latent heat of melting of ice is 80 cal/gm)
(a) 0°C (b) 7.5°C (c) 12.5°C (d) -1.25°C

Q23. A vessel has two compartments of volume V1 and V2 , containing an ideal gas at pressures

p1 and p 2 , and temperatures T1 and T2 respectively. If the wall separating the


compartments is removed, the resulting equilibrium temperature will be
p1T1  p 2T2 V1T1  V2T2
(a) (b)
p1  p 2 V1  V2

p1V1  p 2V2 1/ 2
(c) (d) T1T2 
 p1V1 / T1    p2V2 / T2 

Head office Branch office


fiziks, H.No. 23, G.F, Jia Sarai, Anand Institute of Mathematics,
Near IIT, Hauz Khas, New Delhi-16 28-B/6, Jia Sarai, Near IIT
Phone: 011-26865455/+91-9871145498 Hauz Khas, New Delhi-16
Website: www.physicsbyfiziks.com
Email: fiziks.physics@gmail.com 6
fiziks
Institute for NET/JRF, GATE, IIT-JAM, JEST, TIFR and GRE in PHYSICAL SCIENCES

Q24. For temperature T1  T2 , the qualitative temperature dependence of the probability


distribution F v  of the speed v of a molecule in three dimensions is correctly
represented by the following figure:

T2
T1
F(V)

F(V)
T1 T2
(a) (b)

T1
F(V)

F(V)

T2 T1
T2
(c) (d)

V V

Q25. A system of non-interacting spin-1/2 charged particles are placed in an external magnetic
field. At low temperature T , the leading behavior of the excess energy above the ground
state energy, depends on T as: ( c is a constant)
(a) cT (b) cT 3 (c) e  c / T (d) c (is independent of T )

Q26. The acceleration due to gravity g is determined by measuring the time period T and the
length L of a simple pendulum. If the uncertainties in the measurements of T and L are
T and L respectively, the fractional error g / g in measuring g is best approximated
by
L T L 2 T
(a)  (b) 
L T L T
2 2 2 2
L T  L   2 T 
(c)  (d)    
L T  L   T 

Head office Branch office


fiziks, H.No. 23, G.F, Jia Sarai, Anand Institute of Mathematics,
Near IIT, Hauz Khas, New Delhi-16 28-B/6, Jia Sarai, Near IIT
Phone: 011-26865455/+91-9871145498 Hauz Khas, New Delhi-16
Website: www.physicsbyfiziks.com
Email: fiziks.physics@gmail.com 7
fiziks
Institute for NET/JRF, GATE, IIT-JAM, JEST, TIFR and GRE in PHYSICAL SCIENCES

Q27. A silicon transistor with built-in voltage 0.7 V is used in the circuit shown, with
VBB  9.7V , RB  300k, VCC  12V and RC  2k . Which of the following figures
correctly represents the load line and quiescent Q point?

RC

RB 
 
VCC
V BB

iC iC
   mA  
32 35 6 35
(a) 32 (b) 32
Q
30 Q
30
0 9.7 VCE V  0 12 VCE V 

iC iC
mA   A  
6 35 32 Q 35
(c) Q
32 (d) 32
30 30
0 12 VCE V  0 9. 7 VCE V 

Q28. If the analog input to an 8-bit successive approximation ADC is increased from 1.0 V to
2.0 V, then the conversion time will
(a) remain unchanged (b) double
(c) decrease to half its original value (d) increase four times

Head office Branch office


fiziks, H.No. 23, G.F, Jia Sarai, Anand Institute of Mathematics,
Near IIT, Hauz Khas, New Delhi-16 28-B/6, Jia Sarai, Near IIT
Phone: 011-26865455/+91-9871145498 Hauz Khas, New Delhi-16
Website: www.physicsbyfiziks.com
Email: fiziks.physics@gmail.com 8
fiziks
Institute for NET/JRF, GATE, IIT-JAM, JEST, TIFR and GRE in PHYSICAL SCIENCES

Q29. The insulation resistance R of an insulated cable is measured by connecting it in parallel


with a capacitor C , a voltmeter, and battery B as shown. The voltage across the cable
dropped from 150 V to 15 V, 1000 seconds after the switch S is closed. If the capacitance
of the cable is 5F then its insulation resistance is approximately
S


B V R C

(a) 109  (b) 108  (c) 10 7  (d) 10 6 

Q30. The approximation cos  1 is valid up to 3 decimal places as long as  is less than:

(take 180 o /   57.29 o )


(a) 1.28° (b) 1.81° (c) 3.28° (d) 4.01°

Q31. The area of a disc in its rest frame S is equal to 1 (in some units). The disc will appear
distorted to an observer O moving with a speed u with respect to S along the plane of the
disc. The area of the disc measured in the rest frame of the observer O is ( c is the speed
of light in vacuum)
1/ 2 1 / 2 1
 u2   u2   u2   u2 
(a) 1  2  (b) 1  2  (c) 1  2  (d) 1  2 
 c   c   c   c 

Head office Branch office


fiziks, H.No. 23, G.F, Jia Sarai, Anand Institute of Mathematics,
Near IIT, Hauz Khas, New Delhi-16 28-B/6, Jia Sarai, Near IIT
Phone: 011-26865455/+91-9871145498 Hauz Khas, New Delhi-16
Website: www.physicsbyfiziks.com
Email: fiziks.physics@gmail.com 9
fiziks
Institute for NET/JRF, GATE, IIT-JAM, JEST, TIFR and GRE in PHYSICAL SCIENCES

Q32. A particle of charge e and mass m is located at the midpoint of the line joining two fixed
collinear dipoles with unit charges as shown in the figure. (The particle is constrained to
move only along the line joining the dipoles). Assuming that the length of the dipoles is
much shorter than their separation, the natural frequency of oscillation of the particle is
R R

  e, m  

2d 2d
2 2 2
6e R 6e R 6e 2 d 2 6e 2 d
(a) (b) (c) (d)
 0 md 5  0 md 4  0 mR 5  0 mR 4

Q33. A current I is created by a narrow beam of protons moving in vacuum with constant
 
velocity u . The direction and magnitude, respectively, of the Poynting vector S outside
the beam at a radial distance r (much larger than the width of the beam) from the axis,
are
   I2    I2
(a) S  u and S   (b) S ||  u  and S  
4 2  0 u r 2 4 2  0 u r 4
   I2    I2
(c) S || u and S   (d) S || u and S  
4 2  0 u r 2 4 2  0 u r 4


Q34. If the electric and magnetic fields are unchanged when the potential A changes (in
  
suitable units) according to A  A  r̂ , where r  r t r̂ , then the scalar potential  must
simultaneously change to
(a)   r (b)   r (c)    r /  t (d)    r /  t

Head office Branch office


fiziks, H.No. 23, G.F, Jia Sarai, Anand Institute of Mathematics,
Near IIT, Hauz Khas, New Delhi-16 28-B/6, Jia Sarai, Near IIT
Phone: 011-26865455/+91-9871145498 Hauz Khas, New Delhi-16
Website: www.physicsbyfiziks.com
Email: fiziks.physics@gmail.com 10
fiziks
Institute for NET/JRF, GATE, IIT-JAM, JEST, TIFR and GRE in PHYSICAL SCIENCES

Q35. Consider an axially symmetric static charge distribution of the form,


2
r 
   0  0  e r / r0 cos 2 
r
The radial component of the dipole moment due to this charge distribution is
(a) 2 0 r04 (b)  0 r04 (c)  0 r04 (d)  0 r04 / 2

Q36. In a basis in which the z-component S z of the spin is diagonal, an electron is in a spin
state
 1  i  / 6 
   
 2 / 3 

The probabilities that a measurement of S z will yield the values  / 2 and   / 2 are,
respectively,
(a) 1/2 and 1/2 (b) 2/3 and 1/3 (c) 1/4 and 3/4 (d) 1/3 and 2/3

Q37. Consider the normalized state  of a particle in a one-dimensional harmonic oscillator:

  b1 0  b2 1

where 0 and 1 denote the ground and first excited states respectively, and b1 and b2

are real constants. The expectation value of the displacement x in the state  will be a

minimum when
1 1
(a) b2  0, b1  1 (b) b2  b1 (c) b2  b1 (d) b2  b1
2 2

Q38.  
A muon   from cosmic rays is trapped by a proton to form a hydrogen-like atom.
Given that a muon is approximately 200 times heavier than an electron, the longest
wavelength of the spectral line (in the analogue of the Lyman series) of such an atom will
be
o o o o
(a) 5.62 A (b) 6.67 A (c) 3.75 A (d) 13.3 A

Head office Branch office


fiziks, H.No. 23, G.F, Jia Sarai, Anand Institute of Mathematics,
Near IIT, Hauz Khas, New Delhi-16 28-B/6, Jia Sarai, Near IIT
Phone: 011-26865455/+91-9871145498 Hauz Khas, New Delhi-16
Website: www.physicsbyfiziks.com
Email: fiziks.physics@gmail.com 11
fiziks
Institute for NET/JRF, GATE, IIT-JAM, JEST, TIFR and GRE in PHYSICAL SCIENCES

Q39. The un-normalized wavefunction of a particle in a spherically symmetric potential is


given by

 r   zf r 
where f r  is a function of the radial variable r . The eigenvalue of the operator

L2 (namely the square of the orbital angular momentum) is
(a)  2 / 4 (b)  2 / 2 (c)  2 (d) 2 2

Q40. Given that



tn 2

 H n x 
n 0 n!
 e t 2 tx

the value of H 4 0 is


(a) 12 (b) 6 (c) 24 (d) – 6

Q41. A unit vector n̂ on the xy -plane is at an angle of 120° with respect to iˆ . The angle
 
between the vectors u  a iˆ  b nˆ and v  anˆ  b iˆ will be 60° if

(a) b  3a / 2 (b) b  2a / 3 (c) b  a / 2 (d) b  a

Q42. With z  x  iy, which of the following functions f  x, y  is NOT a (complex) analytic
function of z ?


(a) f x, y   x  iy  8 4  x 2  y 2  2ixy
3
 7

7 3
(b) f  x, y    x  iy  1  x  iy 


(c) f  x, y   x 2  y 2  2ixy  3 
5

4 6
(d) f  x, y   1  x  iy  2  x  iy 

Head office Branch office


fiziks, H.No. 23, G.F, Jia Sarai, Anand Institute of Mathematics,
Near IIT, Hauz Khas, New Delhi-16 28-B/6, Jia Sarai, Near IIT
Phone: 011-26865455/+91-9871145498 Hauz Khas, New Delhi-16
Website: www.physicsbyfiziks.com
Email: fiziks.physics@gmail.com 12
fiziks
Institute for NET/JRF, GATE, IIT-JAM, JEST, TIFR and GRE in PHYSICAL SCIENCES

Q43. A planet of mass m and an angular momentum L moves in a circular orbit in a potential,
V r   k / r , where k is a constant. If it is slightly perturbed radially, the angular
frequency of radial oscillations is
(a) mk 2 / 2 L3 (b) mk 2 / L3 (c) 2 mk 2 / L3 (d) 3mk 2 / L3

Q44. The Lagrangian of a particle of mass m moving in one dimension is given by


1
L mx 2  bx
2
where b is a positive constant. The coordinate of the particle xt  at time t is given by: (in
following c1 and c2 are constants)
b 2
(a)  t  c1t  c2 (b) c1t  c2
2m
 bt   bt   bt   bt 
(c) c1 cos   c 2 sin   (d) c1 cosh   c 2 sinh 
m m m m

Q45. A uniform cylinder of radius r and length l , and a uniform sphere of radius R are
released on an inclined plane when their centres of mass are at the same height. If they
roll down without slipping, and if the sphere reaches the bottom of the plane with a speed
V , then the speed of the cylinder when it reaches the bottom is:

14rl r 4V 14
(a) V (b) 4V (c) (d) V
15R 2 15R 15 15

Head office Branch office


fiziks, H.No. 23, G.F, Jia Sarai, Anand Institute of Mathematics,
Near IIT, Hauz Khas, New Delhi-16 28-B/6, Jia Sarai, Near IIT
Phone: 011-26865455/+91-9871145498 Hauz Khas, New Delhi-16
Website: www.physicsbyfiziks.com
Email: fiziks.physics@gmail.com 13
fiziks
Institute for NET/JRF, GATE, IIT-JAM, JEST, TIFR and GRE in PHYSICAL SCIENCES

PART-C

Q46. The components of a vector potential A   A0 , A1 , A2 , A3  are given by

A  k  xyz, yzt , zxt , xyt 

where k is a constant. The three components of the electric field are


(a) k  yz, zx, xy  (b) k  x, y, z  (c) 0, 0, 0  (d) k  xt , yt , zt 

Q47. In the Born approximation, the scattering amplitude f   for the Yukawa potential

e   r
V r  
r

is given by: (in the following b  2k sin , E   2 k 2 / 2m )
2
2m 2m 2m 2m
(a)  (b)  (c)  (d) 

2  2  b 
2 2 
  2  b2
2
 2  2  b2 
2  2  b2 
3

Q48. If  nlm denotes the eigenfunciton of the Hamiltonian with a potential V  V r  then the

expectation value of the operator L2x  L2y in the state

1
 
5

3 211   210  15 211 
is
(a) 39 2 / 25 (b) 13 2 / 25 (c) 2 2 (d) 26 2 / 25

Q49. An oscillating current I t   I 0 exp it  flows in the direction of the y-axis through a
thin metal sheet of area 1.0 cm2 kept in the xy -plane. The rate of total energy radiated per
unit area from the surfaces of the metal sheet at a distance of 100 m is

(a) I 0 / 12 0 c 3   
(b) I 02 2 / 12 0 c 3

(c) I 02  / 12 c 
0
3
(d) I 0 2 / 24 c 
0
3

Head office Branch office


fiziks, H.No. 23, G.F, Jia Sarai, Anand Institute of Mathematics,
Near IIT, Hauz Khas, New Delhi-16 28-B/6, Jia Sarai, Near IIT
Phone: 011-26865455/+91-9871145498 Hauz Khas, New Delhi-16
Website: www.physicsbyfiziks.com
Email: fiziks.physics@gmail.com 14
fiziks
Institute for NET/JRF, GATE, IIT-JAM, JEST, TIFR and GRE in PHYSICAL SCIENCES

Q50. Consider a two-dimensional infinite square well


0 0  x  a, 0 ya
V x , y   
 otherwise
Its normalized Eigenfunctions are

2  n xx   n y y 
 n x , n y  x, y   sin  sin 
a  a   a 

where n x , n y  1,2,3, …..If a perturbation

 a a
V 0 x , 0 y
H '  0 2 2
 0 otherwise

is applied, then the correction to the energy of the first excited state to order V0 is

V0 V0  64 
(a)
4
(b)
4 1  9 2 
 
V0  16  V0  32 
(c)
4 1  9 2  (d)
4 1  9 2 

Q51. Consider a system of two Ising spins S1 and S 2 taking values  1 with interaction energy
given by    JS1 S 2 , when it is in thermal equilibrium at temperature T . For large T ,
the average energy of the system varies as C / k B T , with C given by

(a)  2J 2 (b)  J 2 (c) J 2 (d) 4 J

Q52. Consider three particles A, B and C , each with an attribute S that can take two
values  1 . Let S A  1, S B  1 and S C  1 at a given instant. In the next instant, each S

value can change to  S with probability1 / 3 . The probability that S A  S B  S C remains


unchanged is
(a) 2 / 3 (b) 1 / 3 (c) 2 / 9 (d) 4 / 9

Head office Branch office


fiziks, H.No. 23, G.F, Jia Sarai, Anand Institute of Mathematics,
Near IIT, Hauz Khas, New Delhi-16 28-B/6, Jia Sarai, Near IIT
Phone: 011-26865455/+91-9871145498 Hauz Khas, New Delhi-16
Website: www.physicsbyfiziks.com
Email: fiziks.physics@gmail.com 15
fiziks
Institute for NET/JRF, GATE, IIT-JAM, JEST, TIFR and GRE in PHYSICAL SCIENCES

Q53. The bound on the ground state energy of the Hamiltonian with an attractive delta-
function potential, namely
2 d 2
H   a  x 
2m dx 2
using the variational principle with the trial wavefunction   x   A exp  bx 2 is 

 
 Note :  e t dt   a  1
t a

 0 
(a)  ma 2 / 4  2 (b)  ma 2 / 2  2

(c)  ma 2 /   2 (d)  ma 2 / 5  2
Q54. Consider two different systems each with three identical non-interacting particles. Both
have single particle states with energies  0 ,3 0 and 5 0 ,  0  0  . One system is populated

1
by spin  fermions and the other by bosons. What is the value of E F  E B where EB
2
and EB are the ground state energies of the fermionic and bosonic systems respectively?
(a) 6 0 (b) 2 0 (c) 4 0 (d)  0

Q55. The input to a lock-in amplifier has the form Vi t   Vi sin  t   i  where Vi ,  ,  i are the
amplitude, frequency and phase of the input signal respectively. This signal is multiplied
by a reference signal of the same frequency  , amplitude Vr and phase  r . If the
multiplied signal is fed to a low pass filter of cut-off frequency  , the final output signal
is
1
(a) ViVr cos i   r 
2
 1 
(b) ViVr cos i   r   cos  t   i   r 
 2 
(c) ViVr sin  i   r 

 1 
(d) ViVr cos i   r   cos  t   i   r 
 2 

Head office Branch office


fiziks, H.No. 23, G.F, Jia Sarai, Anand Institute of Mathematics,
Near IIT, Hauz Khas, New Delhi-16 28-B/6, Jia Sarai, Near IIT
Phone: 011-26865455/+91-9871145498 Hauz Khas, New Delhi-16
Website: www.physicsbyfiziks.com
Email: fiziks.physics@gmail.com 16
fiziks
Institute for NET/JRF, GATE, IIT-JAM, JEST, TIFR and GRE in PHYSICAL SCIENCES

Q56. The solution of the partial differential equation


2 2
u  x , t   u  x, t   0
t 2 x 2
satisfying the boundary conditions u 0, t   0  u L, t  and initial conditions

u x,0  sinx / L  and u  x, t  t 0  sin 2x / L  is
t
L
(a) sin  x / L  cos t / L   sin 2x / L  cos2 t / L 
2
(b) 2 sin x / L  cos t / L   sin x / L  cos2 t / L 
L
(c) sin x / L  cos2 t / L   sin 2x / L sin  t / L 

L
(d) sin x / L  cos t / L   sin 2x / L sin 2 t / L 
2

Q57. Consider the hydrogen-deuterium molecule HD. If the mean distance between the two
atoms is 0.08 nm and the mass of the hydrogen atom is 938 MeV / c 2 , then the energy
difference E between the two lowest rotational states is approximately
(a) 10 1 eV (b) 10 2 eV (c) 2  10 2 eV (d) 10 3 eV

Q58. Four digital outputs V , P, T and H monitor the speed v , tyre pressure p , temperature t
and relative humidity h of a car. These outputs switch from 0 to 1 when the values of the
parameters exceed 85 km/hr, 2 bar, 40 0 C and 50%, respectively. A logic circuit that is
used to switch ON a lamp at the output E is shown below.

P
E

H
Head office Branch office
fiziks, H.No. 23, G.F, Jia Sarai, Anand Institute of Mathematics,
Near IIT, Hauz Khas, New Delhi-16 28-B/6, Jia Sarai, Near IIT
Phone: 011-26865455/+91-9871145498 Hauz Khas, New Delhi-16
Website: www.physicsbyfiziks.com
Email: fiziks.physics@gmail.com 17
fiziks
Institute for NET/JRF, GATE, IIT-JAM, JEST, TIFR and GRE in PHYSICAL SCIENCES

Which of the following condition will switch the lamp ON?


(a) v  85km / hr, p  2 bar, t  40 0 C , h  50%

(b) v  85km / hr, p  2 bar, t  40 0 C , h  50%

(c) v  85km / hr, p  2 bar, t  40 0 C , h  50%

(d) v  85km / hr, p  2 bar, t  40 0 C , h  50%

Q59. The solution of the differential equation


dx
 x2
dt
with the initial condition x0  1 will blow up as t tends to
(a) 1 (b) 2 (c) ½ (d) 

Q60. Let u be a random variable uniformly distributed in the interval 0,1 and V  c ln u ,
where c is a real constant. If V is to be exponentially distributed in the interval
[0, ) with unit standard deviation, then the value of c should be
(a) ln 2 (b) ½ (c) 1 (d) -1

1
Q61. The inverse Laplace transforms of is
s s  1
2

1 2 t 1 2
(a) t e (b) t  1  e t
2 2
1 2
(c) t 1  e t (d)
2

t 1  e t 

Q62. The number of degrees of freedom of a rigid body in d space-dimensions is


(a) 2 d (b) 6 (c) d d  1 / 2 (d) d !

Head office Branch office


fiziks, H.No. 23, G.F, Jia Sarai, Anand Institute of Mathematics,
Near IIT, Hauz Khas, New Delhi-16 28-B/6, Jia Sarai, Near IIT
Phone: 011-26865455/+91-9871145498 Hauz Khas, New Delhi-16
Website: www.physicsbyfiziks.com
Email: fiziks.physics@gmail.com 18
fiziks
Institute for NET/JRF, GATE, IIT-JAM, JEST, TIFR and GRE in PHYSICAL SCIENCES

Q63. A particle of mass m is at the stable equilibrium position of its potential energy
V  x   ax  bx 3
where a, b are positive constants. The minimum velocity that has to be imparted to the
particle to render its motion unstable is

(a) 64a 3 / 9m 2 b  (b) 64a 3 / 279m 2 b 


1/ 4 1/ 4

(c) 16a 3 / 27m 2 b  (d) 3a 3 / 64m 2 b 


1/ 4 1/ 4

Q64. If the operators A and B satisfy the commutation relation  A, B  I , where I is the
identity operator, then
 
(a) e A , B  e A   
(b) e A , B  e B , A 
(c) e , B   e
A B
,A  (d) e , B   I
A

Q65. A system is governed by the Hamiltonian


1
H  p x  ay 2  1  p x  bx2
2 2
where a and b are constants and p x , p y are momenta conjugate to x and y respectively.

For what values of a and b will the quantities  p x  3 y  and  p y  2 x  be conserved?

(a) a  3, b  2 (b) a  3, b  2


(c) a  2, b  3 (d) a  2, b  3

Q66. Using the frequency-dependent Drude formula, what is the effective kinetic inductance of
a metallic wire that is to be used as a transmission line? [In the following, the electron
mass is m , density of electrons is n , and the length and cross-sectional area of the wire
 and A respectively.]


(a) mA / ne 2  (b) zero (c) m / ne 2 A  
(d) m A / ne 2  2 

Head office Branch office


fiziks, H.No. 23, G.F, Jia Sarai, Anand Institute of Mathematics,
Near IIT, Hauz Khas, New Delhi-16 28-B/6, Jia Sarai, Near IIT
Phone: 011-26865455/+91-9871145498 Hauz Khas, New Delhi-16
Website: www.physicsbyfiziks.com
Email: fiziks.physics@gmail.com 19
fiziks
Institute for NET/JRF, GATE, IIT-JAM, JEST, TIFR and GRE in PHYSICAL SCIENCES

Q67. The phonon dispersion for the following one-dimensional diatomic lattice with masses
M 1 and M 2 (as shown in the figure)
K

M1 M2 M1 M2
is given by

 1 1  4M 1 M 2  qa  
 2 q   K    1  1  sin 2   
 M 1 M 2   M 1  M 2  2
 2  

where a is the lattice parameter and K is the spring constant. The velocity of sound is

K M 1  M 2  K
(a) a (b) a
2M 1M 2 2 M 1  M 2 

K M 1  M 2  KM 1 M 2
(c) a (d) a
2 M 1  M 2 
3
M 1M 2

Q68. The binding energy of a light nucleus Z , A in MeV is given by the approximate formula

B A, Z   16 A  20 A 2/3 3
 Z 2 A 1 / 3  30
N  Z  2

4 A
where N  A  Z is the neutron number. The value of Z of the most stable isobar for a
given A is
1 1 1
A A2 / 3  A A A2 / 3  A A4 / 3 
(a) 1   (b) (c) 1   (d) 1  
2 160  2 2 120  2 64 
Q69. Muons are produced through the annihilation of particle a and its antiplarticle, namely the
process
a  a    
A muon has a rest mass of 105 MeV/c2 and its proper life time is 2  s . If the center of
mass energy of the collision is 2.1 GeV in the laboratory frame that coincides with the
center-of-mass frame, then the fraction of muons that will decay before they reach a
detector placed 6 km away from the interaction point is
(a) e 1 (b) 1  e 1 (c) 1  e 2 (d) e 10

Head office Branch office


fiziks, H.No. 23, G.F, Jia Sarai, Anand Institute of Mathematics,
Near IIT, Hauz Khas, New Delhi-16 28-B/6, Jia Sarai, Near IIT
Phone: 011-26865455/+91-9871145498 Hauz Khas, New Delhi-16
Website: www.physicsbyfiziks.com
Email: fiziks.physics@gmail.com 20
fiziks
Institute for NET/JRF, GATE, IIT-JAM, JEST, TIFR and GRE in PHYSICAL SCIENCES

Q70. The conductors in a 0.75 km long two-wire transmission line are separated by a centre-to-
centre distance of 0.2 m. If each conductor has a diameter of 4 cm, then the capacitance
of the line is
(a) 8.85 F (b) 88.5 nF (c) 8.85 pF (d) 8.85 nF

Q71. The electron dispersion relation for a one-dimensional metal is given by


 ka 1 2 
 k  2 0 sin 2  sin ka 
 2 6 
where k is the momentum, a is the lattice constant,  0 is a constant having dimensions of

energy and ka   . If the average number of electrons per atom in the conduction band

is 1/3, then the Fermi energy is


(a)  0 / 4 (b)  0 (c) 2 0 / 3 (d) 5 0 / 3

Q72. The electronic energy levels in a hydrogen atom are given by E n  13.6 / n 2 eV. If a

selective excitation to the n  100 level is to be made using a laser, the maximum
allowed frequency line-width of the laser is
(a) 6.5 MHz (b) 6.5 GHz (c) 6.5 Hz (d) 6.5 kHz

Q73. If the energy dispersion of a two-dimensional electron system is E  uk where u is the
velocity and k is the momentum, then the density of states DE  depends on the energy
as
(a) 1 / E (b) E (c) E (d) constant

Head office Branch office


fiziks, H.No. 23, G.F, Jia Sarai, Anand Institute of Mathematics,
Near IIT, Hauz Khas, New Delhi-16 28-B/6, Jia Sarai, Near IIT
Phone: 011-26865455/+91-9871145498 Hauz Khas, New Delhi-16
Website: www.physicsbyfiziks.com
Email: fiziks.physics@gmail.com 21
fiziks
Institute for NET/JRF, GATE, IIT-JAM, JEST, TIFR and GRE in PHYSICAL SCIENCES

Q74. Consider the laser resonator cavity shown in the figure.

R1  1 R2  R
l

M1 P M2

If I 1 is the intensity of the radiation at mirror M 1 and  is the gain coefficient of the
medium between the mirrors, then the energy density of photons in the plane P at a
distance x from M 1 is

(a) I 1 / c e x (b) I 1 / c ex


(c) I 1 / c  ex  e x  (d) I 1 / c e 2x

Q75. A spin-1/2 particle A undergoes the delay


A BC D
where it is known that B and C are also spin-1/2 particles. The complete set of allowed
values of the spin of the particle D is
1 3 5
(a) ,1, , 2, , 3, ... (b) 0, 1
2 2 2
1 1 3 5 7
(c) only (d) , , , ,....
2 2 2 2 2

Head office Branch office


fiziks, H.No. 23, G.F, Jia Sarai, Anand Institute of Mathematics,
Near IIT, Hauz Khas, New Delhi-16 28-B/6, Jia Sarai, Near IIT
Phone: 011-26865455/+91-9871145498 Hauz Khas, New Delhi-16
Website: www.physicsbyfiziks.com
Email: fiziks.physics@gmail.com 22
fiziks
Institute for NET/JRF, GATE, IIT-JAM, JEST, TIFR and GRE in PHYSICAL SCIENCES

JRF-NET(PHYSICS)-December-2013
Q1. A cylinder of radius 1 cm and height 1 cm is broken into three pieces. Which of the
following MUST be true?
(1) At least one piece has volume equal to 1 cm 3
(2) At least two pieces have equal volumes.
(3) At least one piece has volume less than 1 cm 3

(4) At least one piece has volume greater than 1 cm 3

2
Q2. For real numbers x and y , x 2   y  4   0 . Then the value of x  y is

(1) 0 (2) 2 (3) 2 (4) 4

Q3. Every time a ball falls to ground, it bounces back to half the height it fell from. A ball is
dropped from a height of 1024 cm . The maximum height from the ground to which it
can rise after the tenth bounce is
(1) 102.4 cm (2) 1.24 cm (3) 1 cm (4) 2 cm

Q4. A farmer gives 7 full, 7 half-full and 7 empty bottles of honey to his three sons and asks
them to share these among themselves such that each of them gets the same amount of
honey and the same number of bottles. In how many ways can this be done? (bottles
cannot be distinguished otherwise, they are sealed and cannot be broken).
(1) 0 (2) 1 (3) 2 (4) 3

Q5. A car is moving along a straight track. Its speed is changing with time as shown below.
Which of the following statements is correct?
(1) The speed is never zero.
speed

(2) The acceleration is zero once on the path.


(3) The distance covered initially increases and then decreases.
0
(4) The car comes back to its initial position once. 0 Time

Head office Branch office


fiziks, H.No. 23, G.F, Jia Sarai, Anand Institute of Mathematics,
Near IIT, Hauz Khas, New Delhi-16 28-B/6, Jia Sarai, Near IIT
Phone: 011-26865455/+91-9871145498 Hauz Khas, New Delhi-16
Website: www.physicsbyfiziks.com
Email: fiziks.physics@gmail.com 1
fiziks
Institute for NET/JRF, GATE, IIT-JAM, JEST, TIFR and GRE in PHYSICAL SCIENCES

Q6. If a  b  c  d  e  10 (all positive numbers), then the maximum value of


a  b  c  d  e is
(1) 12 (2) 32 (3) 48 (4) 72

Q7. How many nine-digit positive integers are there, the sum of squares of whose digits is 2?
(1) 8 (2) 9 (3) 10 (4) 11

Q8. A circle of radius 7 units lying in the fourth quadrant touches the x -axis at 10, 0  . The
centre of the circle has coordinates
(1) 7, 7  (2)  10, 7  (3) 10,  7  (4) 7,  7 

Q9. One of the four A, B, C and D committed a crime. A said, “I did it”, B said, “I didn’t”, C
said, “B did it’, D said, ‘A did it”. Who is lying?
(1) A (2) B (3) C (4) D

Q10. A circle circumscribes identical, close-packed circles of unit diameter as shown in the
figure. What is the total area of the shaded portion?

(1) 2 (2) 2 (3) 1 / 2 (4)  / 2

Q11. There are 2 hills, A and B, in a region. If hill A is located N30o E of hill B, what will be
the direction of hill B when observed from hill A? ( N30 o E means 30 o from north
towards east).
(1) S30 o W (2) S60 o W (3) S30 o E (4) S60 o E

Head office Branch office


fiziks, H.No. 23, G.F, Jia Sarai, Anand Institute of Mathematics,
Near IIT, Hauz Khas, New Delhi-16 28-B/6, Jia Sarai, Near IIT
Phone: 011-26865455/+91-9871145498 Hauz Khas, New Delhi-16
Website: www.physicsbyfiziks.com
Email: fiziks.physics@gmail.com 2
fiziks
Institute for NET/JRF, GATE, IIT-JAM, JEST, TIFR and GRE in PHYSICAL SCIENCES

Q12. What is the next number in the following sequence?


39, 42, 46, 50,….
(1) 52 (2) 53 (3) 54 (4) 55

Q13. What is the perimeter of the given figure, where adjacent sides are at right angles to each
other? 5 cm
4 cm

1 cm

(1) 20 cm (2) 18 cm (3) 21 cm (4) cannot be determined

Q14. Three fishermen caught fishes and went to sleep. One of them woke up, took away one
fish and 1 / 3rd of the remainder as his share, without others’ knowledge. Later, the three
of them divided the remainder equally. How many fishes were caught?
(1) 58 (2) 19 (3) 76 (4) 88

1 1 1 1 1
Q15. What is the arithmetic mean of , , , ,...., ?
1 2 2  3 3  4 4  5 100  101
1 1

1
(1) 0.01 (2) (3) 0.00111.... (4) 49  50 50  51
101 2

Q16. 25  5  3  2  4  16  4  3 


(1) 61 (2) 22 (3) 41/24 (4) 16

Head office Branch office


fiziks, H.No. 23, G.F, Jia Sarai, Anand Institute of Mathematics,
Near IIT, Hauz Khas, New Delhi-16 28-B/6, Jia Sarai, Near IIT
Phone: 011-26865455/+91-9871145498 Hauz Khas, New Delhi-16
Website: www.physicsbyfiziks.com
Email: fiziks.physics@gmail.com 3
fiziks
Institute for NET/JRF, GATE, IIT-JAM, JEST, TIFR and GRE in PHYSICAL SCIENCES

Q17. Consider the sequence of ordered sets of natural numbers:


1, 2, 3, 4, 5, 6,....
What is the last number in the 10 th set?
(1) 10 (2) 19 (3) 55 (4) 67
Q18. A student buys a book from an online shop at 20% discount. His friend buys another
copy of the same book in a book fair for Rs. 192 paying 20% less than his friend. What is
the full price of the book?
(1) Rs. 275 (2) Rs. 300 (3) Rs. 320 (4) Rs. 392

Q19. 366 players participate in a knock-out tournament. In each round all competing players
pair together and play a match, the winner of each match moving to the next round. If at
the end of a round there is an odd number of winners, the unpaired one moves to the next
round without playing a match. What is the total number of matches played?
(1) 366 (2) 282 (3) 365 (4) 416

Q20. What does the diagram below establish?

Note: The diagram is a circle inside a square.


(1)   3 (2)   2 2 (3)   4 (4)  is closer to 3 and 4.

Head office Branch office


fiziks, H.No. 23, G.F, Jia Sarai, Anand Institute of Mathematics,
Near IIT, Hauz Khas, New Delhi-16 28-B/6, Jia Sarai, Near IIT
Phone: 011-26865455/+91-9871145498 Hauz Khas, New Delhi-16
Website: www.physicsbyfiziks.com
Email: fiziks.physics@gmail.com 4
fiziks
Institute for NET/JRF, GATE, IIT-JAM, JEST, TIFR and GRE in PHYSICAL SCIENCES

PART ‘B’
Q21. A horizontal metal disc rotates about the vertical axis in a uniform magnetic field
pointing up as shown in the figure. A circuit is made by connecting one end A of a
resistor to the centre of the disc and the other end B to its edge through a sliding contact.
The circuit that flows through the resistor is
B
A B

(1) zero (2) DC from A to B


(3) DC from B to A (4) AC
1 1 1  i 
Q22. A spin - particle is in the state     in the eigenbasis of S 2 and S z . If we
2 11  3 
h h
measure S z , the probabilities of getting  and  , respectively are
2 2
1 1 2 9 1 3
(1) and (2) and (3) 0 and 1 (4) and
2 2 11 11 11 11
Q23. Which of the following functions cannot be the real part of a complex analytic function
of z  x  iy ?

(1) x 2 y (2) x 2  y 2 (3) x 3  3xy 2 (4) 3 x 2 y  y  y 3


Q24. The motion of a particle of mass m in one dimension is described by the
p2 1
Hamiltonian H   m 2 x 2  x . What is the difference between the (quantized)
2m 2
energies of the first two levels? (In the following, x is the expectation value of x in the

ground state.)
2
(1)    x (2)    x (3)   (4) 
2m 2

Head office Branch office


fiziks, H.No. 23, G.F, Jia Sarai, Anand Institute of Mathematics,
Near IIT, Hauz Khas, New Delhi-16 28-B/6, Jia Sarai, Near IIT
Phone: 011-26865455/+91-9871145498 Hauz Khas, New Delhi-16
Website: www.physicsbyfiziks.com
Email: fiziks.physics@gmail.com 5
fiziks
Institute for NET/JRF, GATE, IIT-JAM, JEST, TIFR and GRE in PHYSICAL SCIENCES

Q25. Let  nlm denote the eigenfunctions of a Hamiltonian for a spherically symmetric

potential V r  . The expectation value of Lz in the state


1
 
6

 200  5 210  10 211  20 211 is 
5 5 5
(1)   (2)  (3)  (4) 
18 6 18

1
Q26. Three identical spin- fermions are to be distributed in two non-degenerate distinct
2
energy levels. The number of ways this can be done is
(1) 8 (2) 4 (3) 3 (4) 2

Q27. Let A, B and C be functions of phase space variables (coordinates and momenta of a
mechanical system). If ,  represents the Poisson bracket, the value of

A, B, C  A, B, C is given by


(1) 0 (2) B, C , A (3) A, C , B (4) C, A, B

Q28. If A, B and C are non-zero Hermitian operators, which of the following relations must
be false?
(1)  A, B  C (2) AB  BA  C (3) ABA  C (4) A  B  C

Q29. The expression


 2 2 2 2  1
 2  2  2  2  2
 x
 1 x2 x3 x4  1 
 x  x2  x2  x2
2 3 4 
is proportional to
(1)   x1  x 2  x3  x4  (2)   x1  x 2  x3  x 4 


(3) x12  x 22  x32  x 42 
3 / 2

(4) x12  x22  x32  x42  2

Head office Branch office


fiziks, H.No. 23, G.F, Jia Sarai, Anand Institute of Mathematics,
Near IIT, Hauz Khas, New Delhi-16 28-B/6, Jia Sarai, Near IIT
Phone: 011-26865455/+91-9871145498 Hauz Khas, New Delhi-16
Website: www.physicsbyfiziks.com
Email: fiziks.physics@gmail.com 6
fiziks
Institute for NET/JRF, GATE, IIT-JAM, JEST, TIFR and GRE in PHYSICAL SCIENCES

 
dx  dx
Q30. Given that the integral  2  , the value of  y is
0 y  x
2
2y 0
2
 x2 
2

   
(1) (2) (3) (4)
y3 4 y3 8y 3 2 y3

Q31. The force between two long and parallel wires carrying currents I 1 and I 2 and separated
by a distance D is proportional to
(2) I 1  I 2  / D
2
(1) I 1 I 2 / D (3) I 1 I 2 / D  (4) I 1 I 2 / D 2

1 2 3 4 5 6
Q32. A loaded dice has the probabilities , , , , and of turning up 1, 2, 3, 4, 5
21 21 21 21 21 21
and 6, respectively. If it is thrown twice, what is the probability that the sum of the
numbers that turn up is even?
144 225 221 220
(1) (2) (3) (4)
441 441 441 441

z2
Q33. A particle moves in a potential V  x 2  y 2  . Which component(s) of the angular
2
momentum is/are constant(s) of motion?
(1) none (2) L x , L y and L z (3) only L x and Ly (4) only Lz

Q34. The Hamiltonian of a relativistic particle of rest mass m and momentum p is given

by H  p 2  m 2  V x  , in units in which the speed of light c  1 . The corresponding


Lagrangian is

(1) L  m 1  x 2  V  x  (2) L  m 1  x 2  V  x 
1 2
(3) L  1  mx 2  V  x  (4) L  mx  V  x 
2

Head office Branch office


fiziks, H.No. 23, G.F, Jia Sarai, Anand Institute of Mathematics,
Near IIT, Hauz Khas, New Delhi-16 28-B/6, Jia Sarai, Near IIT
Phone: 011-26865455/+91-9871145498 Hauz Khas, New Delhi-16
Website: www.physicsbyfiziks.com
Email: fiziks.physics@gmail.com 7
fiziks
Institute for NET/JRF, GATE, IIT-JAM, JEST, TIFR and GRE in PHYSICAL SCIENCES

Q35. A ring of mass m and radius R rolls (without slipping) down an inclined plane starting
from rest. If the centre of the ring is initially at a height h , the angular velocity when the
ring reaches the base is R

h

(1) g / h  R  tan  (2) g / h  R  (3) g h  R  / R 2 (4) 2 g / h  R 

Q36. Consider the op-amp circuit shown in the figure.


1 F

1K
1K
Vi  Vo

If the input is a sinusoidal wave Vi  5 sin 1000t  , then the amplitude of the output V0 is

5 5 2
(1) (2) 5 (3) (4) 5 2
2 2

Q37. If one of the inputs of a J-K flip flop is high and the other is low, then the outputs Q and

Q
(1) oscillate between low and high in race around condition
(2) toggle and the circuit acts like a T flip flop
(3) are opposite to the inputs
(4) follow the inputs and the circuit acts like an R  S flip flop

Head office Branch office


fiziks, H.No. 23, G.F, Jia Sarai, Anand Institute of Mathematics,
Near IIT, Hauz Khas, New Delhi-16 28-B/6, Jia Sarai, Near IIT
Phone: 011-26865455/+91-9871145498 Hauz Khas, New Delhi-16
Website: www.physicsbyfiziks.com
Email: fiziks.physics@gmail.com 8
fiziks
Institute for NET/JRF, GATE, IIT-JAM, JEST, TIFR and GRE in PHYSICAL SCIENCES

Q38. Two monochromatic sources, L1 and L2 emit light at 600 and 700 nm, respectively. If

their frequency bandwidths are 10 1 and 10 3 GHz, respectively, then the ratio of
linewidth of L1 to L2 is approximately
(1) 100 : 1 (2) 1 : 85 (3) 75 : 1 (4) 1 : 75

   
Q39. Let V , A and V , A denote two sets of scalar and vector potentials, and  a scalar
function. Which of the following transformations leave the electric ad magnetic fields
(and hence Maxwell’s equations) unchanged?
 
(1) A  A   and V   V  (2) A  A   and V   V  2
t t
 
(3) A  A   and V   V  (4) A  A   and V   V 
t t

Q40. Consider the melting transition of ice into water at constant pressure. Which of the
following thermodynamic quantities does not exhibit a discontinuous change across the
phase transition?
(1) Internal energy (2) Helmholtz free energy (3) Gibbs free energy (4) entropy

Q41. Two different thermodynamic systems are described by the following equations of state:
1 3RN 1 1 5 RN  2 
 and 
T 1 2U 1 T 2 2U  2 
where T 1, 2  , N 1, 2  and U 1, 2  are respectively, the temperatures, the mole numbers and

the internal energies of the two systems, and R is the gas constant. Let U tot denote the
total energy when these two systems are put in contact and attain thermal equilibrium.
U 1
The ratio is
U tot

5N 2 3 N 1 N 1 N 2


(1) (2) (3) (4)
3 N 1  5 N  2  3 N 1  5 N  2  N 1  N  2  N 1  N  2 

Head office Branch office


fiziks, H.No. 23, G.F, Jia Sarai, Anand Institute of Mathematics,
Near IIT, Hauz Khas, New Delhi-16 28-B/6, Jia Sarai, Near IIT
Phone: 011-26865455/+91-9871145498 Hauz Khas, New Delhi-16
Website: www.physicsbyfiziks.com
Email: fiziks.physics@gmail.com 9
fiziks
Institute for NET/JRF, GATE, IIT-JAM, JEST, TIFR and GRE in PHYSICAL SCIENCES

Q42. The speed v of the molecules of mass m of an ideal gas obeys Maxwell’s velocity
distribution law at an equilibrium temperature T . Let v x , v y v z  denote the components of

the velocity and k B the Boltzmann constant. The average value of v x   y  , where 
2

and  are constants, is

 
(1)  2   2 k B T / m  
(2)  2   2 k B T / m
2 2
(3)     k B T / m (4)     k B T / m

Q43. The entropy S of a thermodynamic system as a function of energy E is given by the


following graph S

C
B
A

E
The temperatures of the phases A, B and C , denoted by T A , TB and TC , respectively,
satisfy the following inequalities:
(1) TC  TB  T A (2) T A  TC  TB (3) TB  TC  T A (4) TB  T A  TC

Q44. The physical phenomenon that cannot be used for memory storage applications is
(1) large variation in magnetoresistance as a function of applied magnetic field
(2) variation in magnetization of a ferromagnet as a function of applied magnetic field
(3) variation in polarization of a ferroelectric as a function of applied electric field
(4) variation in resistance of a metal as a function of applied electric field

Head office Branch office


fiziks, H.No. 23, G.F, Jia Sarai, Anand Institute of Mathematics,
Near IIT, Hauz Khas, New Delhi-16 28-B/6, Jia Sarai, Near IIT
Phone: 011-26865455/+91-9871145498 Hauz Khas, New Delhi-16
Website: www.physicsbyfiziks.com
Email: fiziks.physics@gmail.com 10
fiziks
Institute for NET/JRF, GATE, IIT-JAM, JEST, TIFR and GRE in PHYSICAL SCIENCES

Q45. Two identical Zener diodes are placed back to back in series and are connected to a
variable DC power supply. The best representation of the I-V characteristics of the circuit
is
(1) I (2) I

V V

(3) I (4) I

V V

PART ‘C’
Q46. A pendulum consists of a ring of mass M and radius R suspended by a massless rigid
rod of length l attached to its rim. When the pendulum oscillates in the plane of the ring,
the time period of oscillation is

lR 2
(1) 2
g
(2) l 2
 R2 1/ 4

2 R 2  2 Rl  l 2 2
(3) 2
g R  l 
(4) 2 R 2
 2 Rl  l 2 
1/ 4

Q47. Spherical particles of a given material of density  are released from rest inside a liquid
medium of lower density. The viscous drag force may be approximated by the Stoke’s
law, i.e., Fd  6Rv , where  is the viscosity of the medium, R the radius of a particle

and v its instantaneous velocity. If  m  is the time taken by a particle of mass m to


reach half its terminal velocity, then the ratio  8m  /  m is
(1) 8 (2) 1 / 8 (3) 4 (4) 1 / 4

Head office Branch office


fiziks, H.No. 23, G.F, Jia Sarai, Anand Institute of Mathematics,
Near IIT, Hauz Khas, New Delhi-16 28-B/6, Jia Sarai, Near IIT
Phone: 011-26865455/+91-9871145498 Hauz Khas, New Delhi-16
Website: www.physicsbyfiziks.com
Email: fiziks.physics@gmail.com 11
fiziks
Institute for NET/JRF, GATE, IIT-JAM, JEST, TIFR and GRE in PHYSICAL SCIENCES

Q48. A system of N classical non-interacting particles, each of mass m , is at a temperature T


1 2
and is confined by the external potential V r   Ar (where A is a constant) in three
2
dimensions. The internal energy of the system is

3 3/ 2 A  k BT 
(1) 3 Nk B T (2) Nk B T (3) N 2mA k BT (4) N ln 
2 m  m 
Q49. Consider a particle of mass m attached to two identical springs each y
of length l and spring constant k (see the figure below). The
equilibrium configuration is the one where the springs are unstretched.
o
There are no other external forces on the system. If the particle is
given a small displacement along the x -axis, which of the following
describes the equation of motion for small oscillations? x
o
kx 3 kx 2
(1) mx  0 (2) mx  kx  0 (3) mx  2kx  0 (4) mx  0
l2 l


Q50. If   x   A exp  x 4  is the eigenfunction of a one dimensional Hamiltonian with
eiggenvalue E  0 , the potential V x  (in units where   2m  1 ) is

(1) 12x 2 (2) 16x 6 (3) 16 x 6  12x 2 (4) 16 x 6  12x 2

Q51. The electric field of an electromagnetic wave is given by



E  E0 cos 0.3x  0.4 y  1000t kˆ .

The associated magnetic field B is
(1) 10 3 E 0 cos 0.3 x  0.4 y  1000t kˆ

 
(2) 10 4 E 0 cos 0.3 x  0.4 y  1000t  4iˆ  3 ˆj

(3) E cos 0.3 x  0.4 y  1000t 0.3iˆ  0.4 ˆj 


0

(4) 10 E cos  0.3 x  0.4 y  1000t 3iˆ  4 ˆj 


2
0

Head office Branch office


fiziks, H.No. 23, G.F, Jia Sarai, Anand Institute of Mathematics,
Near IIT, Hauz Khas, New Delhi-16 28-B/6, Jia Sarai, Near IIT
Phone: 011-26865455/+91-9871145498 Hauz Khas, New Delhi-16
Website: www.physicsbyfiziks.com
Email: fiziks.physics@gmail.com 12
fiziks
Institute for NET/JRF, GATE, IIT-JAM, JEST, TIFR and GRE in PHYSICAL SCIENCES

Q52. The energy of an electron in a band as a function of its wave vector k is given
by E k   E 0  Bcos k x a  cos k y a  cos k z a , where E 0 B and a are constants. The

effective mass of the electron near the bottom of the band is


2 2 2 2 2
(1) (2) (3) (4)
3Ba 2 3Ba 2 2Ba 2 Ba 2

Q53. A DC voltage V is applied across a Josephson junction between two superconductors


with a phase difference  0 . If l 0 and k are constants that depend on the properties of the
junction, the current flowing through it has the form
 2eVt   2eVt 
(1) I 0 sin   0  (2) kV sin   0 
     
(3) kV sin  0 (4) I 0 sin  0  kV

Q54. Consider the following ratios of the partial decay widths

R1 

     0
and R
 

      p 

     0 
2

      n 
If the effects of electromagnetic and weak interactions are neglected, then R1 and R2 are
respectively,

(1) 1 and 2 (2) 1 and 2 (3) 2 and 1 (4) 1 and 1

A
Q55. The intrinsic electric dipole moment of a nucleus Z X
(1) increases with Z , but independent of A
(2) decreases with Z , but independent of A
(3) is always zero
(4) increases with Z and A

Head office Branch office


fiziks, H.No. 23, G.F, Jia Sarai, Anand Institute of Mathematics,
Near IIT, Hauz Khas, New Delhi-16 28-B/6, Jia Sarai, Near IIT
Phone: 011-26865455/+91-9871145498 Hauz Khas, New Delhi-16
Website: www.physicsbyfiziks.com
Email: fiziks.physics@gmail.com 13
fiziks
Institute for NET/JRF, GATE, IIT-JAM, JEST, TIFR and GRE in PHYSICAL SCIENCES

Q56. According to the shell model, the total angular momentum (in units of  ) and the parity
of the ground state of the 73 Li nucleus is

3 3
(1) with negative parity (2) with positive parity
2 2
1 7
(3) with positive parity (4) with negative parity
2 2
Q57. A point charge q is placed symmetrically at a distance d from two perpendicularly
placed grounded conducting infinite plates as shown in the figure. The net force on the
charge (in units of 1 / 4 0 ) is

q2 d q
(1)
8d 2
 
2 2  1 away from the corner

d
q2
(2)
8d 2
 
2 2  1 towards the corner

q2
(3) towards the corner
2 2d 2
3q 2
(4) away from the corner
8d 2
Q58. Let four point charges q,  q / 2, q and  q / 2 be placed at the vertices of a square of
side a . Let another point charge  q be placed at the cnetre of the square (see the figure).

q/2 q

q

q q/2

Let V r  be the electrostatic potential at a point P at a distance r  a from the centre

of the square. Then V 2r  / V r  is


1 1 1
(1) 1 (2) (3) (4)
2 4 8

Head office Branch office


fiziks, H.No. 23, G.F, Jia Sarai, Anand Institute of Mathematics,
Near IIT, Hauz Khas, New Delhi-16 28-B/6, Jia Sarai, Near IIT
Phone: 011-26865455/+91-9871145498 Hauz Khas, New Delhi-16
Website: www.physicsbyfiziks.com
Email: fiziks.physics@gmail.com 14
fiziks
Institute for NET/JRF, GATE, IIT-JAM, JEST, TIFR and GRE in PHYSICAL SCIENCES

Q59. Let A and B be two vectors in three-dimensional Euclidean space. Under rotation, the
tensor product Tij  Ai B j

(1) reduces to a direct sum of three 3-dimensional representations


(2) is an irreducible 9-dimensional representation
(3) reduces to a direct sum of a 1-dimensional, a 3-dimensional and a 5-dimensional
irreducible representations
(4) reduces to a direct sum of a 1-dimensional and an 8-dimensional irreducible
representations

Q60. The Fourier transform of the derivative of the Dirac  - function, namely  x  , is
proportional to
(1) 0 (2) 1 (3) sin k (4) ik

Q61. A particle is in the ground state of an infinite square well potential given by,
0 for  a  x  a
V x   
 otherwise

a a
The probability to find the particle in the interval between  and is
2 2
1 1 1 1 1 1
(1) (2)  (3)  (4)
2 2  2  
Q62. The expectation value of the x - component of the orbital angular momentum L x in the

1
state  
5

3 2 ,1, 1  5 2,1, 0  11 2,1, 1 
(where  nlm are the eigenfunctions in usual notation), is

 10  10
(1) 
25
 11  3 (2) 0 (3)
25
 11  3 (4)  2

Head office Branch office


fiziks, H.No. 23, G.F, Jia Sarai, Anand Institute of Mathematics,
Near IIT, Hauz Khas, New Delhi-16 28-B/6, Jia Sarai, Near IIT
Phone: 011-26865455/+91-9871145498 Hauz Khas, New Delhi-16
Website: www.physicsbyfiziks.com
Email: fiziks.physics@gmail.com 15
fiziks
Institute for NET/JRF, GATE, IIT-JAM, JEST, TIFR and GRE in PHYSICAL SCIENCES

Q63. A particle is prepared in a simultaneous eigenstate of L2 and Lz . If l   1 2 and m are

respectively the eigenvalues of L2 and Lz , then the expectation value L2x of the particle

in this state satisfies

(1) L2x  0 (2) 0  L2x   2  2

2   1 2  2 2   1 2


(3) 0  L x  (4)  Lx 
3 2 3

Q64. If the electrostatic potential V r ,  ,   in a charge free region has the form

V r       f r  cos , then the functional form of f r  (in the following a and b are
constants) is:
b b b r
(1) ar 2  (2) ar  (3) ar  (4) a ln 
r r2 r b

Q65. If A  iˆyz  ˆjxz  kˆxy , then the integral  A dl (where C is along the perimeter of a
C

rectangular area bounded by x  0, x  a and y  0, y  b ) is


1 3
(1)
2
a  b3  
(2)  ab 2  a 2 b  
(3)  a 3  b 3  (4) 0

Q66. Consider an n  nn  1 matrix A , in which Aij is the product of the indices i and j

(namely Aij  ij ). The matrix A

(1) has one degenerate eigevalue with degeneracy n  1


(2) has two degenerate eigenvalues with degeneracies 2 and n  2 
(3) has one degenerate eigenvalue with degeneracy n
(4) does not have any degenerate eigenvalue

Head office Branch office


fiziks, H.No. 23, G.F, Jia Sarai, Anand Institute of Mathematics,
Near IIT, Hauz Khas, New Delhi-16 28-B/6, Jia Sarai, Near IIT
Phone: 011-26865455/+91-9871145498 Hauz Khas, New Delhi-16
Website: www.physicsbyfiziks.com
Email: fiziks.physics@gmail.com 16
fiziks
Institute for NET/JRF, GATE, IIT-JAM, JEST, TIFR and GRE in PHYSICAL SCIENCES

Q67. A child makes a random walk on a square lattice of constant a taking a step in the north,
east, south or west directions with probabilities 0.255, 0.255, 0.245 and 0.245,
respectively. After a large number of steps, N , the expected position of the child with
respect to the starting point is at a distance
(1) 2  10 2 Na in the north-east direction
(2) 2 N  10 2 a in the north-east direction

(3) 2 2  10 2 Na in the south-west direction


(4) 0

Q68. A Carnot cycle operates as a heat engine between two bodies of equal heat capacity until
their temperatures become equal. If the initial temperatures of the bodies are T1 and T2 ,
respectively and T1  T2 then their common final temperature is
1
(1) T12 / T2 (2) T22 / T1 (3) T1T2 (4) T1  T2 
2

Q69. Three sets of data A, B and C from an experiment, represented by  , and  , are
plotted on a log-log scale. Each of these are fitted with straight lines as shown in the
figure. 1000

100
C
B
10

1 
 A


0.1 1 10 100 1000
0.1

The functional dependence y x  for the sets A, B and C are respectively


x 1 1
(1) x, x and x 2 (2)  , x and 2 x (3) , x and x 2 (4) , x and x 2
2 x2 x

Head office Branch office


fiziks, H.No. 23, G.F, Jia Sarai, Anand Institute of Mathematics,
Near IIT, Hauz Khas, New Delhi-16 28-B/6, Jia Sarai, Near IIT
Phone: 011-26865455/+91-9871145498 Hauz Khas, New Delhi-16
Website: www.physicsbyfiziks.com
Email: fiziks.physics@gmail.com 17
fiziks
Institute for NET/JRF, GATE, IIT-JAM, JEST, TIFR and GRE in PHYSICAL SCIENCES

Q70. A sample of Si has electron and hole mobilities of 0.13 and 0.05 m 2 /V- s respectively at
300 K. It is doped with P and Al with doping densities of 1.5  10 21 / m 3 and
2.5  10 21 / m 3 respectively. The conductivity of the doped Si sample at 300 K is
(1) 8  1 m 1 (2) 32  1 m 1

(3) 20.8  1 m 1 (4) 83.2  1 m 1

Q71. A 4-variable switching function is given by f   5, 7, 8, 10, 13, 15   d 0, 1, 2 , where

d is the do-not-care-condition. The minimized form of f in sum of products (SOP) form


is
(1) A C  B D (2) AB  CD
(3) AD  BC (4) B D  BD

Q72. A perturbation V pert  aL2 is added to the Hydrogen atom potential. The shift in the

energy level of the 2 P state, when the effects of spin are neglected up to second order in
a , is
(1) 0 (2) 2a 2  a 2  4
3 2 4
(3) 2 a 2 (4) a 2  a 
2

Q73. A gas laser cavity has been designed to operate at   0.5 m with a cavity length of1 m .
With this set-up, the frequency is found to be larger than the desired frequency by
100 Hz. The change in the effective length of the cavity required to retune the laser is
(1)  0.334  10 12 m (2) 0.334  10 12 m
(3) 0.167  10 12 m (4)  0.167  10 12 m

Head office Branch office


fiziks, H.No. 23, G.F, Jia Sarai, Anand Institute of Mathematics,
Near IIT, Hauz Khas, New Delhi-16 28-B/6, Jia Sarai, Near IIT
Phone: 011-26865455/+91-9871145498 Hauz Khas, New Delhi-16
Website: www.physicsbyfiziks.com
Email: fiziks.physics@gmail.com 18
fiziks
Institute for NET/JRF, GATE, IIT-JAM, JEST, TIFR and GRE in PHYSICAL SCIENCES

Q74. The spectroscopic symbol for the ground state of 13 Al is 2 P1/ 2 . Under the action of a
strong magnetic field (when L  S coupling can be neglected) the ground state energy
level will split into
(1) 3 levels (2) 4 levels (3) 5 levels (4) 6 levels

Q75. A uniform linear monoatomic chain is modeled by a spring-mass system of masses m


separated by nearest neighbour distance a and spring constant m 02 . The dispersion
relation for this system is
  ka    ka 
(1)  k   2 0 1  cos   (2)  k   20 sin 2  
  2   2

 ka   ka 
(3)  k   2 0 sin   (4)  k   2 0 tan  
 2  2 

Head office Branch office


fiziks, H.No. 23, G.F, Jia Sarai, Anand Institute of Mathematics,
Near IIT, Hauz Khas, New Delhi-16 28-B/6, Jia Sarai, Near IIT
Phone: 011-26865455/+91-9871145498 Hauz Khas, New Delhi-16
Website: www.physicsbyfiziks.com
Email: fiziks.physics@gmail.com 19
fiziks
Forum for CSIR-UGC JRF/NET, GATE, IIT-JAM, GRE in PHYSICAL SCIENCES

NET-DEC-2012
Part A
Q1. A granite block of 2m × 5m × 3m size is cut into 5 cm thick slabs of 2m × 5m size. These
slabs are laid over a 2m wide pavement. What is the length of the pavement that can be
covered with these slabs?
(a) 100 m (b) 200 m (c) 300 m (d) 500 m

Q2. Which is the least among the following?


0.33 0..33, 0.44 0.44, π-1/π, e-1/e
(a) 0.330.33 (b) 0.44 0.44 (c) π-1/π (d) e-1/e

Q3. What is the next number in this “see and tell” sequence?
1 11 21 1211 111221 _______
(a) 312211 (b) 1112221 (c) 1112222 (d) 1112131

Q4. A vertical pole of length a stands at the centre of a horizontal regular hexagonal ground
of side a. A rope that is fixed taut in between a vertex on the ground and the tip of the
pole has a length
(a) a (b) 2a (c) 3a (d) 6a

Q5. A peacock perched on the top of a 12 m high tree spots a snake moving towards its hole
at the base of the tree from a distance equal to thrice the height of the tree. The peacock
flies towards the snake in a straight line and they both move at the same speed. At what
distance from the base of the tree will the peacock catch the snake?
(a) 16 m (b) 18 m (c) 14 m (d) 12 m

fiziks c/o Anand Institute of mathematics, 28-B/6 Jia Sarai


Near IIT, Hauz Khas, New Delhi, PIN- 110016 (INDIA)
Phone: 011-32718565, +91-9871145498
Website: http://www.physicsbyfiziks.com 1
Email: fiziks.physics@gmail.com
fiziks
Forum for CSIR-UGC JRF/NET, GATE, IIT-JAM, GRE in PHYSICAL SCIENCES

Q6. The cities of a country are connected by intercity roads. If a city is directly connected to
an odd number of other cities, it is called an odd city. If a city is directly connected to an
even number of other cities, it is called an even city. Then which of the following is
impossible?
(a) There are an even number of odd cities (b) There are an odd number of odd cities
(c) There are an even number of even cities (d) There are an odd number of even cities

Q7. In the figure ABC   / 2 , AD = DE = EB


C
What is the ratio of the area of triangle ADC to that of triangle
CDB?
(a) 1:1 (b) 1:2
(c) 1:3 (d) 1:4
A D E B

Q8. A rectangular sheet ABCD is folded in such a way that vertex A D C

meets vertex C, thereby forming a line PQ. Assuming AB = 3 and


P
BC = 4, find PQ. Note that AP = PC and AQ = QC.
(a) 13/4 (b) 15/4 Q
(c) 17/4 (d) 19/4
A B

Q9. A string of diameter 1mm is kept on a table in the shape of a close flat spiral i.e. a spiral with
no gap between the turns. The area of the table occupied by the spiral is 1m2. Then the length of
the string is
(a) 10 m (b) 10 2 m (c) 103 m (d) 10 4 m

Q10. 25% of 25% of a quantity is x% of the quantity where x is


(a) 6.25 % (b) 12.5 % (c) 25 % (d) 50 %

fiziks c/o Anand Institute of mathematics, 28-B/6 Jia Sarai


Near IIT, Hauz Khas, New Delhi, PIN- 110016 (INDIA)
Phone: 011-32718565, +91-9871145498
Website: http://www.physicsbyfiziks.com 2
Email: fiziks.physics@gmail.com
fiziks
Forum for CSIR-UGC JRF/NET, GATE, IIT-JAM, GRE in PHYSICAL SCIENCES

Q11. In sequence {an}every term is equal to the sum of all previous terms. If a0 = 3, then
a n 1
lim is
n a
n

(a) 3 (b) 2 (c) 1 (d) e

Q12. In the figure given, angle ABC = π/2. I, II, III are the areas of A
semicircles on the sides opposite angles B, A and C, respectively. I
III
Which of the following is always true?
(a) II2 + III2 = I2 (b) II + III = I B II
C
(c) II2 + III2 > I2 (d) II + III < I

Q13. What is the minimum number of days between one Friday the 13th and the next Friday
the 13 th? (Assume that the year is a leap year).
(a) 28 (b) 56 (c) 91 (d) 84

Q14. Suppose a person A is at the North-East corner of a square (see A


the figure below). From that point he moves along the diagonal N

and after covering 1/3rd portion of the diagonal, he goes to his left
W E
and after sometime he stops, rotates 90° clockwise and moves
straight. After a few minutes he stops, rotates 180° anticlockwise. S
Towards which direction he is facing now?
(a) North-East (b) North-West (c) South-East (d) South-West

Q15. Cucumber contains 99% water. Ramesh buys 100 kg of cucumbers. After 30 days of
storing the cucumbers lose some water. They now contain 98% water. What is the total
weight of cucumbers now?
(a) 99 kg (b) 50 kg (c) 75 kg (d) 2 kg

fiziks c/o Anand Institute of mathematics, 28-B/6 Jia Sarai


Near IIT, Hauz Khas, New Delhi, PIN- 110016 (INDIA)
Phone: 011-32718565, +91-9871145498
Website: http://www.physicsbyfiziks.com 3
Email: fiziks.physics@gmail.com
fiziks
Forum for CSIR-UGC JRF/NET, GATE, IIT-JAM, GRE in PHYSICAL SCIENCES

Q16. In a museum there were old coins with their respective years engraved on them, as
follows:
(i) 1837 AD (ii) 1907 AD (iii) 1947 AD (iv) 200 BC
identify the fake coin(s)
(a) coin (i) (b) coin (iv) (c) coins (i) and (ii) (d) coin (iii)

Q17. A student observes the movement of four snails and plots the graphs of distance moved
as a function of time as given in figures (A), (B), (C) and (D).
Distance

Distance

Distance
Distance

Time Time Time Time

A  B C D


Which of the following is not correct?
(a) Graph (A) (b) Graph (B) (c) Graph (C) (d) Graph (D)

Q18. Find the missing letter:

A EGK C

? P

U R
Q V
B OJF D

(a) H (b) L (c) Z (d) Y

fiziks c/o Anand Institute of mathematics, 28-B/6 Jia Sarai


Near IIT, Hauz Khas, New Delhi, PIN- 110016 (INDIA)
Phone: 011-32718565, +91-9871145498
Website: http://www.physicsbyfiziks.com 4
Email: fiziks.physics@gmail.com
fiziks
Forum for CSIR-UGC JRF/NET, GATE, IIT-JAM, GRE in PHYSICAL SCIENCES

Q19. Consider the following equation


x 2  4 y 2  9 z 2  14 x  28 y  42 z  147
where x, y and z are real numbers. Then the value of x + 2y + 3z is
(a) 7 (b) 14 (c) 21 (d) not unique

Q20. The map given below shows a meandering river following a semi-circular path, along
which two villages are located at A and B. The distance between A and B along the east-
west direction in the map is 7 cm. What is the length of the river between A and B in the
ground?
N
A B

SCALE  1 : 50000

(a) 1.1 km (b) 3.5 km (c) 5.5 km (d) 11.0 km

fiziks c/o Anand Institute of mathematics, 28-B/6 Jia Sarai


Near IIT, Hauz Khas, New Delhi, PIN- 110016 (INDIA)
Phone: 011-32718565, +91-9871145498
Website: http://www.physicsbyfiziks.com 5
Email: fiziks.physics@gmail.com
fiziks
Forum for CSIR-UGC JRF/NET, GATE, IIT-JAM, GRE in PHYSICAL SCIENCES

Part B
Q21. A 2×2 matrix A has eigenvalues e i / 5 and ei / 6 . The smallest value of n such that An = I is
(a) 20 (b) 30 (c) 60 (d) 120

Q22. The graph of the function f(x) shown below is best described by
1.00
0.25
0.50
0.25
f (x)

0.00
 0.25
 0.50
 0.75
 1.00
0 1 2 3 4 5 6 7 8 9 10
x
(a) The Bessel function J0 (x) (b) cos x
1
(c) e  x cos x (d) cos x
x

Q23. In a series of five Cricket matches, one of the captains calls “Heads” every time when the
toss is taken. The probability that he will win 3 times and lose 2 times is
(a) 1/8 (b) 5/8 (c) 3/16 (d) 5/16

 a b c 
Q24. The unit normal vector at the point  , ,  on the surface of the
 3 3 3
x2 y2 z2
ellipsoid    1 , is
a2 b2 c2
bciˆ  caˆj  abkˆ aiˆ  bˆj  ckˆ
(a) (b)
a 2c 2  b 2c 2  a 2b 2 a 2  b2  c2

biˆ  cˆj  akˆ iˆ  ˆj  kˆ


(c) (d)
a 2  b2  c2 3

fiziks c/o Anand Institute of mathematics, 28-B/6 Jia Sarai


Near IIT, Hauz Khas, New Delhi, PIN- 110016 (INDIA)
Phone: 011-32718565, +91-9871145498
Website: http://www.physicsbyfiziks.com 6
Email: fiziks.physics@gmail.com
fiziks
Forum for CSIR-UGC JRF/NET, GATE, IIT-JAM, GRE in PHYSICAL SCIENCES

Q25. A solid cylinder of height H, radius R and density ρ, floats vertically on the surface of a
liquid of density  0 . The cylinder will be set into oscillatory motion when a small
instantaneous downward force is applied. The frequency of oscillation is

g  g g 0 g
(a) (b) (c) (d)
0 H 0 H 0H H

Q26. Three particles of equal mass m are connected by two identical massless springs of
stiffness constant k as shown in the figure:
If x1, x2 and x3 denote the horizontal displacements of the masses from their respective
equilibrium positions, the potential energy of the system is
k k
m m m
1 1
(a)
2

k x12  x 22  x32  (b)
2

k x12  x 22  x32  x2  x1  x3  
1 1
(c)
2

k x12  2 x22  x32  2 x2 x1  x3   (d)
2

k x12  2 x22  x32  2 x 2 x1  x3  

Q27. Let v, p and E denote the speed, the magnitude of the momentum, and the energy of a
free particle of rest mass m. Then

(a) dE  constant (b) p = mv


dp

(c) v  cp 2 2 2
(d) E = mc2
p m c

Q28. A binary star system consists of two stars S1 and S2, with masses m and 2m respectively
separated by a distance r. If both S1 and S2 individually follow circular orbits around the
centre of mass with instantaneous speeds v1 and v2 respectively, the speeds ratio v1/v2 is
(a) 2 (b) 1 (c) 1/2 (d) 2

fiziks c/o Anand Institute of mathematics, 28-B/6 Jia Sarai


Near IIT, Hauz Khas, New Delhi, PIN- 110016 (INDIA)
Phone: 011-32718565, +91-9871145498
Website: http://www.physicsbyfiziks.com 7
Email: fiziks.physics@gmail.com
fiziks
Forum for CSIR-UGC JRF/NET, GATE, IIT-JAM, GRE in PHYSICAL SCIENCES

Q29. Three charges are located on the circumference of a circle of radius R as shown in the
figure below. The two charges Q subtend an angle 90° at the centre
Q Q
of the circle. The charge q is symmetrically placed with respect to
the charges Q. If the electric field at the centre of the circle is zero,
what is the magnitude of Q?
(a) q / 2 (b) 2q (c) 2q (d) 4q q

Q30. Consider a hollow charged shell of inner radius a and outer radius b. The volume charge
k
density is  r   (k is constant) in the region a < r < b. The magnitude of the electric
r2
field produced at distance r > a is
k b  a 
(a) for all r > a
0r2
k b  a  kb
(b) 2
for a < r < b and for r > b
 0r  0r 2
k r  a  k b  a 
(c) 2
for a < r < b and for r > b
 0r 0r2
k r  a  k b  a 
(d) 2
for a < r < b and for r > b
 0a 0r2

Q31. Consider the interference of two coherent electromagnetic waves whose electric field
 
vectors are given by E1  iˆE 0 cos  t and E 2  ˆjE0 cos t    where  is the phase

0 2
difference. The intensity of the resulting wave is given by E , where E 2 is the
2
time average of E 2. The total intensity is
(a) 0 (b)  0 E 02 (c)  0 E 02 sin 2  (d)  0 E02 cos 2 

fiziks c/o Anand Institute of mathematics, 28-B/6 Jia Sarai


Near IIT, Hauz Khas, New Delhi, PIN- 110016 (INDIA)
Phone: 011-32718565, +91-9871145498
Website: http://www.physicsbyfiziks.com 8
Email: fiziks.physics@gmail.com
fiziks
Forum for CSIR-UGC JRF/NET, GATE, IIT-JAM, GRE in PHYSICAL SCIENCES

Q32. Four charges (two + q and two –q) are kept fixed at the four vertices of a square of side a
as shown
q q

a R P

q q

At the point P which is at a distance R from the centre (R >> a), the potential is
proportional to
(a) 1/R (b) 1/R2 (c) 1/R3 (d) 1/R4

Q33. A point charges q of mass m is kept at a distance d below a grounded infinite conducting
sheet which lies in the xy - plane. For what value of d will the charge remains stationary?
(a) q / 4 mg 0 (b) q / mg 0

(c) There is no finite value of d (d) mg 0 / q

Q34. The wave function of a state of the hydrogen atom is given by


   200  2 211  3 210  2 211

where  nlm is the normalized eigen function of the state with quantum numbers n, l and
m in the usual notation. The expectation value of Lz in the state  is
(a) 15 / 16 (b) 11 / 16 (c) 3 / 8 (d)  / 8

1
Q35. The energy eigenvalues of a particle in the potential V x   m 2 x 2  ax are
2

 1 a2  1 a2
(a) E n   n    (b) E n   n   
 2 2m 2  2 2m 2

 1 a2  1
(c) E n   n    (d) E n   n  
 2 m 2  2

fiziks c/o Anand Institute of mathematics, 28-B/6 Jia Sarai


Near IIT, Hauz Khas, New Delhi, PIN- 110016 (INDIA)
Phone: 011-32718565, +91-9871145498
Website: http://www.physicsbyfiziks.com 9
Email: fiziks.physics@gmail.com
fiziks
Forum for CSIR-UGC JRF/NET, GATE, IIT-JAM, GRE in PHYSICAL SCIENCES

Q36. If a particle is represented by the normalized wave function


 15 a 2  x 2

 for  a  x  a
 x   4a 5 / 2
0 otherwise

the uncertainty p in its momentum is

(a) 2 / 5a (b) 5 / 2a (c) 10 / a (d) 5 / 2 a

Q37. Given the usual canonical commutation relations, the commutator  A, B of

A  i xp y  yp x  and B   yp z  zp y  is

(a)  xp z  p x z  (b)   xp z  p x z 

(c)  xp z  p x z  (d)   xp z  p x z 

Q38. The entropy of a system, S, is related to the accessible phase space volume  by
S  k B ln E , N , V  where E, N and V are the energy, number of particles and volume
respectively. From this one can conclude that 
(a) does not change during evolution to equilibrium
(b) oscillates during evolution to equilibrium
(c) is a maximum at equilibrium
(d) is a minimum at equilibrium

Q39. Let W be the work done in a quasistatic reversible thermodynamic process. Which of
the following statements about W is correct?
(a) W is a perfect differential if the process is isothermal
(b) W is a perfect differential if the process is adiabatic
(c) W is always a perfect differential
(d) W cannot be a perfect differential

fiziks c/o Anand Institute of mathematics, 28-B/6 Jia Sarai


Near IIT, Hauz Khas, New Delhi, PIN- 110016 (INDIA)
Phone: 011-32718565, +91-9871145498
Website: http://www.physicsbyfiziks.com 10
Email: fiziks.physics@gmail.com
fiziks
Forum for CSIR-UGC JRF/NET, GATE, IIT-JAM, GRE in PHYSICAL SCIENCES

Q40. Consider a system of three spins S1, S2 and S3 each of which can take values +1 and -1.
The energy of the system is given by E   J S1 S 2  S 2 S 3  S 3 S1  where J is a positive
constant. The minimum energy and the corresponding number of spin configuration are,
respectively,
(a) J and 1 (b) -3J and 1 (c)-3J and 2 (d) -6J and 2

1
Q41. The minimum energy of a collection of 6 non-interacting electrons of spin - and mass
2
m placed in a one dimensional infinite square well potential of width L is
(a) 14 2  2 / mL2 (b) 91 2  2 / mL2 (c) 7 2  2 / mL2 (d) 3 2  2 / mL2

Q42. A live music broadcast consists of a radio-wave of frequency 7 MHz, amplitude-


modulated by a microphone output consisting of signals with a maximum frequency of
10 kHz. The spectrum of modulated output twill be zero outside the frequency band
(a) 7.00 MHz to 7.01 MHz (b) 6.99 MHz to 7.01 MHz
(c) 6.99 MHz to 7.00 MHz (d) 6.995 MHz to 7.005 MHz

Q43. In the op-amp circuit shown in the figure, Vi is a sinusoidal input signal of frequency
10 Hz and V0 is the output signal. The magnitude of the 0.01F
gain and the phase shift, respectively, close to the values
(a) 5 2 and  / 2 10K
1K
(b) 5 2 and   / 2 Vi
Vo
(c) 10 and zero
(d) 10 and π

fiziks c/o Anand Institute of mathematics, 28-B/6 Jia Sarai


Near IIT, Hauz Khas, New Delhi, PIN- 110016 (INDIA)
Phone: 011-32718565, +91-9871145498
Website: http://www.physicsbyfiziks.com 11
Email: fiziks.physics@gmail.com
fiziks
Forum for CSIR-UGC JRF/NET, GATE, IIT-JAM, GRE in PHYSICAL SCIENCES

Q44. The logic circuit shown in the figure below Implements the Boolean expression
A

HIGH y

(a) y  A  B (b) y  A  B (c) y  A  B (d) y  A  B

Q45. A diode D as shown in the circuit has an i-v relation that can be approximated by
v 2  2v D , for v D  0
iD   D
0, for v D  0

1

iD

10 V D vD

The value of v D in the circuit is


(a)  1 11 V  (b) 8 V (c) 5 V (d) 2 V

Q46. The Taylor expansion of the function ln(cosh x), where x is real, about the point x = 0
starts with the following terms:
1 2 1 4 1 2 1 4
(a)  x  x  .... (b) x  x  ....
2 12 2 12
1 2 1 4 1 2 1 4
(c)  x  x  .... (d) x  x  ....
2 6 2 6

fiziks c/o Anand Institute of mathematics, 28-B/6 Jia Sarai


Near IIT, Hauz Khas, New Delhi, PIN- 110016 (INDIA)
Phone: 011-32718565, +91-9871145498
Website: http://www.physicsbyfiziks.com 12
Email: fiziks.physics@gmail.com
fiziks
Forum for CSIR-UGC JRF/NET, GATE, IIT-JAM, GRE in PHYSICAL SCIENCES

Q47. Given a 2×2 unitary matrix U satisfying U†U = UU† = 1 with det U  e i , one can
 
construct a unitary matrix V V †V  VV †  1 with det V = 1 from it by

(a) multiplying U by e i / 2


(b) multiplying any single element of U by e  i
(c) multiplying any row or column of U by e i / 2
(d) multiplying U by e i

z 3 dz
Q48. The value of the integral  , where C is a closed contour defined by the
C z 2  5z  6

equation 2 z  5  0, traversed in the anti-clockwise direction, is

(a)  16 i (b) 16 i (c) 8 i (d) 2 i

d2 f
Q49. The function f(x) obeys the differential equation  3  2i  f  0 and satisfies the
dx 2
conditions f(0) = 1 and f(x) → 0 as x → ∞. The value of f(π) is
(a) e 2 (b) e 2 (c)  e 2 (d)  e 2 i

Q50. A planet of mass m moves in the gravitational field of the Sun (mass M). If the semi-
major and semi-minor axes of the orbit are a and b respectively, the angular momentum
of the planet is

(a) 2GMm 2 a  b (b) 2GMm 2 a  b 

2GMm 2 ab 2GMm 2 ab
(c) (d)
ab ab

fiziks c/o Anand Institute of mathematics, 28-B/6 Jia Sarai


Near IIT, Hauz Khas, New Delhi, PIN- 110016 (INDIA)
Phone: 011-32718565, +91-9871145498
Website: http://www.physicsbyfiziks.com 13
Email: fiziks.physics@gmail.com
fiziks
Forum for CSIR-UGC JRF/NET, GATE, IIT-JAM, GRE in PHYSICAL SCIENCES

Q51. The Hamiltonian of a simple pendulum consisting of a mass m attached to a massless


p2
string of length l is H   mgl 1  cos  . If L denotes the Lagrangian, the value of
2 ml 2
dL
is:
dt
2g g
(a)  p sin  (b)  p sin 2
l l
g
(c) p cos (d) lp2 cos
l

Q52. Which of the following set of phase-space trajectories is not possible for a particle
obeying Hamilton’s equations of motion?
(a) (b)
P P

x x
(c) (d)
P P

x x

Q53. Two bodies of equal mass m are connected by a massless rigid rod of length l lying in the
xy-plane with the centre of the rod at the origin. If this system is rotating about the z-axis
with a frequency ω, its angular momentum is
(a) ml 2 / 4 (b) ml 2 / 2 (c) ml 2 (d) 2ml 2

fiziks c/o Anand Institute of mathematics, 28-B/6 Jia Sarai


Near IIT, Hauz Khas, New Delhi, PIN- 110016 (INDIA)
Phone: 011-32718565, +91-9871145498
Website: http://www.physicsbyfiziks.com 14
Email: fiziks.physics@gmail.com
fiziks
Forum for CSIR-UGC JRF/NET, GATE, IIT-JAM, GRE in PHYSICAL SCIENCES

Q54. An infinite solenoid with its axis of symmetry along the z-direction carries a steady
current I.
 ẑ
The vector potential A at a distance R from the axis
(a) is constant inside and varies as R outside the solenoid
R
(b) varies as R inside and is constant outside the solenoid
1
(c) varies as inside and as R outside the solenoid
R
1
(d) varies as R inside and as outside the solenoid
R

Q55. Consider an infinite conducting sheet in the xy-plane with a time dependent current
density Kt iˆ , where K is a constant. The vector potential at (x, y, z) is given
 K 
by A  0 ct  z  iˆ . The magnetic field B is
2

4c
 0 Kt ˆ  0 Kz ˆ
(a) j (b)  j
2 2c
0 K 0 K
(c)  ct  z iˆ (d)  ct  z  ˆj
2c 2c


Q56. When a charged particle emits electromagnetic radiation, the electric field E and the
 1   1
Poynting vector S  E  B at a larger distance r from emitter vary as n and
0 r

1
respectively. Which of the following choices for n and m are correct?
rm
(a) n = 1 and m = 1 (b) n = 2 and m = 2
(c) n = 1 and m = 2 (d) n = 2 and m = 4

fiziks c/o Anand Institute of mathematics, 28-B/6 Jia Sarai


Near IIT, Hauz Khas, New Delhi, PIN- 110016 (INDIA)
Phone: 011-32718565, +91-9871145498
Website: http://www.physicsbyfiziks.com 15
Email: fiziks.physics@gmail.com
fiziks
Forum for CSIR-UGC JRF/NET, GATE, IIT-JAM, GRE in PHYSICAL SCIENCES

1
Q57. The energies in the ground state and first excited state of a particle of mass m  in a
2
potential V x  are -4 and -1, respectively, (in units in which ħ = 1). If the corresponding
wavefunctions are related by  1 x    0  x sinh x, then the ground state eigenfunction is

(a)  0 x   sec hx (b)  0 x   sec hx

(c)  0 x   sec h 2 x (d)  0 x   sec h 3 x

Q58. The perturbation


ba  x , a  x  a
H ' 
0, otherwise
acts on a particle of mass m confined in an infinite square well potential
0, a  x  a
V x   
, otherwise
The first order correction to the ground state energy of the particle is
ba ba
(a) (b) (c) 2ba (d) ba
2 2

Q59. Let 0 and 1 denote the normalized eigenstates corresponding to the ground and the

first excited states of a one-dimensional harmonic oscillator. The uncertainty x in the


1
state 0  1  is
2

(a) x   / 2m (b) x   / m

(c) x  2 / m (d) x  4 / m

fiziks c/o Anand Institute of mathematics, 28-B/6 Jia Sarai


Near IIT, Hauz Khas, New Delhi, PIN- 110016 (INDIA)
Phone: 011-32718565, +91-9871145498
Website: http://www.physicsbyfiziks.com 16
Email: fiziks.physics@gmail.com
fiziks
Forum for CSIR-UGC JRF/NET, GATE, IIT-JAM, GRE in PHYSICAL SCIENCES

Q60. What would be the ground state energy of the Hamiltonian


2 d 2
H     x 
2m dx 2
2
if variational principle is used to estimate it with the trial wavefunction   x   Ae bx
with b as the variational parameter?
 1
2 n  1
 x e dx  2b 
2 n  2bx
[Hint: 2  n   ]
  2

(a)  m 2 / 2 2 (b)  2m 2 /   2 (c)  m 2 /   2 (d) m 2 /   2

Q61. The free energy difference between the superconducting and the normal states of a
2  4
material is given by F  FS  FN      , where  is an order parameter and α
2
and β are constants such that   0 in the normal and   0 in the superconducting state,
while   0 always. The minimum value of F is

(a)   2 /  (b)   2 / 2  (c)  3 2 / 2  (d)  5 2 / 2 

Q62. A given quantity of gas is taken from the state A → C reversibly, by two paths, A → C
directly and A → B → C as shown in the figure.
P
During the process A → C the work done by the gas is 100 J and the A

heat absorbed is 150 J. If during the process A → B → C the work


done by the gas is 30 J, the heat absorbed is
B C
(a) 20 J (b) 80 J
V
(c) 220 J (d) 280 J

fiziks c/o Anand Institute of mathematics, 28-B/6 Jia Sarai


Near IIT, Hauz Khas, New Delhi, PIN- 110016 (INDIA)
Phone: 011-32718565, +91-9871145498
Website: http://www.physicsbyfiziks.com 17
Email: fiziks.physics@gmail.com
fiziks
Forum for CSIR-UGC JRF/NET, GATE, IIT-JAM, GRE in PHYSICAL SCIENCES

Q63. Consider a one-dimensional Ising model with N spins, at very low temperatures when
almost all spins are aligned parallel to each other. There will be a few spin flips with each
flip costing an energy 2J. In a configuration with r spin flips, the energy of the system is
N
E = - NJ + 2rJ and the number of configuration is C r ; r varies from 0 to N. The
partition function is
N
 J 
(a)   (b) e  NJ / kBT
 k BT 
N N
 J   J 
(c)  sinh  (d)  cosh 
 k B T   k B T 

Q64. A magnetic field sensor based on the Hall Effect is to be fabricated by implanting As into
a Si film of thickness 1 µm. The specifications require a magnetic field sensitivity of
500 mV/Tesla at an excitation current of 1 mA. The implantation dose is to be adjusted
such that the average carrier density, after activation, is
(a) 1.25 × 1026 m-3 (b) 1.25 × 1022 m-3
(c) 4.1 × 10 21 m-3 (d) 4.1 × 10 20 m-3

Q65. Band-pass and band-reject filters can be implemented by combining a low pass and a
high pass filter in series and in parallel, respectively. If the cut-off frequencies of the low
pass and high pass filters are  0LP and  0HP , respectively, the condition required to
implement the band-pass and band-reject filters are, respectively,
(a)  0HP   0LP and  0HP   0LP (b) 0HP  0LP and  0HP   0LP

(c)  0HP   0LP and 0HP  0LP (d)  0HP   0LP and  0HP  0LP

fiziks c/o Anand Institute of mathematics, 28-B/6 Jia Sarai


Near IIT, Hauz Khas, New Delhi, PIN- 110016 (INDIA)
Phone: 011-32718565, +91-9871145498
Website: http://www.physicsbyfiziks.com 18
Email: fiziks.physics@gmail.com
fiziks
Forum for CSIR-UGC JRF/NET, GATE, IIT-JAM, GRE in PHYSICAL SCIENCES

Q66. The output characteristics of a solar panel at a certain level of irradiance is shown in the
figure below.

3
I 2

1
0
0 5 10 15 20
V
If the solar cell is to power a load of 5 Ω, the power drawn by the load is
(a) 97 W (b) 73 W (c) 50 W (d) 45 W

Q67. Consider the energy level diagram shown below, which corresponds to the molecular
nitrogen laser. 2
R  21

1
1
0
20 -3 -1
If the pump rate R is 10 atoms cm s and the decay routes are as shown with
 21  20 ns and  1  1s , the equilibrium populations of states 2 and 1 are, respectively,

(a) 1014 cm-3 and 2  1012 cm-3 (b) 21012 cm-3 and 1014 cm-3 .
(c) 21012 cm-3 and 2  10 6 cm-3 (d) zero and 1020 cm-3

Q68. Consider a hydrogen atom undergoing a 2 P  1S transition. The lifetime tsp of the 2P
state for spontaneous emission is 1.6 ns and the energy difference between the levels is
10.2 eV. Assuming that the refractive index of the medium n0 = 1, the ratio of Einstein
coefficients for stimulated and spontaneous emission B21   / A21   is given by
(a) 0.683 × 1012 m3J-1s-1 (b) 0.146 × 10-12 Jsm-3 .
(c) 6.83 × 1012 m3J -1s -1 (d) 1.463 × 10-12 Jsm-3 .

fiziks c/o Anand Institute of mathematics, 28-B/6 Jia Sarai


Near IIT, Hauz Khas, New Delhi, PIN- 110016 (INDIA)
Phone: 011-32718565, +91-9871145498
Website: http://www.physicsbyfiziks.com 19
Email: fiziks.physics@gmail.com
fiziks
Forum for CSIR-UGC JRF/NET, GATE, IIT-JAM, GRE in PHYSICAL SCIENCES

Q69. Consider a He-Ne laser cavity consisting of two mirrors of reflectivities R1 = 1 and
R2 = 0.98. The mirrors are separated by a distance d = 20 cm and the medium in between
has a refractive index n0 = 1 and absorption coefficient α = 0. The values of the
separation between the modes δv and the width v p of each mode of the laser cavity are:

(a) v  75kHz, v p  24kHz (b) v  100kHz, v p  100kHz

(c) v  750MHz, v p  2.4 MHz (d) v  2.4 MHz, v p  750MHz

Q70. Non-interacting bosons undergo Bose-Einstein Condensation (BEC) when trapped in a


three dimensional isotropic simple harmonic potential. For BEC to occur, the chemical
potential must be equal to
(a)  / 2 (b)  (c) 3 / 2 (d) 0

Q71. In a band structure calculation, the dispersion relation for electrons is found to be
 k   cos k x a  cos k y a  cos k z a ,

where β is a constant and a is the lattice constant. The effective mass at the boundary of
the first Brillouin zone is
2 2 4 2 2 2
(a) (b) (c) (d)
5 a 2 5 a 2 2 a 2 3 a 2

Q72. The radius of the Fermi sphere of free electrons in a monovalent metal with an fcc
structure, in which the volume of the unit cell is a3, is
1/ 3 1/ 3 1/ 3
 12 2   3 2   2  1
(a)  3  (b)  3  (c)  3  (d)
 a   a  a  a

Q73. The muon has mass 105 MeV/c2 and mean lifetime 2.2 µs in its rest frame. The mean
distance traversed by a muon of energy 315 MeV/c2 before decaying is approximately
(a) 3 × 10 5 km (b) 2.2 cm (c) 6.6 µm (d) 1.98 km

fiziks c/o Anand Institute of mathematics, 28-B/6 Jia Sarai


Near IIT, Hauz Khas, New Delhi, PIN- 110016 (INDIA)
Phone: 011-32718565, +91-9871145498
Website: http://www.physicsbyfiziks.com 20
Email: fiziks.physics@gmail.com
fiziks
Forum for CSIR-UGC JRF/NET, GATE, IIT-JAM, GRE in PHYSICAL SCIENCES

Q74. Consider the following particles: the proton p, the neutron n, the neutral pion π0 and the
delta resonance Δ+. When ordered in terms of decreasing lifetime, the correct
arrangement is as follows:
(a) π0, n, p, Δ+ . (b) p, n, Δ+, π0.
(c) p, n, π0, Δ+. (d) Δ+, n, π0, p

Q75. The single particle energy difference between the p-orbitals (i.e. P3/2 and P1/2) of the
114
nucleus 50 Sn is 3 MeV. The energy difference between the states in its 1f orbitals is
(a) -7 MeV (b) 7 MeV (c) 5 MeV (d) -5 MeV

fiziks c/o Anand Institute of mathematics, 28-B/6 Jia Sarai


Near IIT, Hauz Khas, New Delhi, PIN- 110016 (INDIA)
Phone: 011-32718565, +91-9871145498
Website: http://www.physicsbyfiziks.com 21
Email: fiziks.physics@gmail.com

S-ar putea să vă placă și